Tải bản đầy đủ (.docx) (54 trang)

Tài liệu tham khảo Toán học cấp 2

Bạn đang xem bản rút gọn của tài liệu. Xem và tải ngay bản đầy đủ của tài liệu tại đây (1.69 MB, 54 trang )

<span class='text_page_counter'>(1)</span><div class='page_container' data-page=1>

<b>Bài 1: </b><i>( HSG TỈNH PHÚ YÊN NĂM HỌC 2011 – 2012)</i>


<i>a) Xác định a, b để </i>

(

)



4 3 2


16<i>x</i> - 32<i>x</i> +24<i>x</i> +<i>ax b</i>+ M

(

2

)


4<i>x</i> - 8<i>x</i>+5


<i>.</i>
Vì 16<i>x</i>4- 32<i>x</i>3+24<i>x</i>2+ +<i>ax b</i>chia hết cho 4<i>x</i>2- 8<i>x</i>+5nên ta có:


4 3 2 2 2


16<i>x</i> - 32<i>x</i> +24<i>x</i> +<i>ax b</i>+ =(4<i>x</i> - 8<i>x</i>+5)(4<i>x</i> + +<i>cx d</i>)


=16<i>x</i>4+(4<i>c</i>- 32)<i>x</i>3+(4<i>d</i>- 8<i>c</i>+20)<i>x</i>2+ -( 8<i>d</i>+5 )<i>c x</i>+5<i>d</i>
Suy ra:


<i> </i>


4 32 32 8


4 8 20 24 5


-8 5 0


5 1


<i>c</i> <i>a</i>



<i>d</i> <i>c</i> <i>b</i>


<i>d</i> <i>c</i> <i>a</i> <i>c</i>


<i>d</i> <i>b</i> <i>d</i>


ì - =- ì
=-ï ï
ï ï
ï ï
ï - + = ï =
ï <sub>Û</sub> ï
í í
ï + = ï =
ï ï
ï ï
ï <sub>=</sub> ï <sub>=</sub>
ï ï
ỵ ỵ <i>.</i>


Vậy<i> a</i> = <i>-</i>8, <i>b</i> = 5 thì u cầu bài tốn được thỏa.


<i>b) Chứng minh </i>


1
4
<i>n</i>
<i>S</i> <


<i>.</i>



Theo a) 16<i>n</i>4- 32<i>n</i>3+24<i>n</i>2- 8<i>n</i>+ =5 (4<i>n</i>2+1)(4<i>n</i>2- 8<i>n</i>+5)


Do đó: 4 3 2 2 2


2 1 2 1


16 32 24 8 5 (4 1)(4 8 5)


<i>n</i> <i>n</i>


<i>n</i> <i>n</i> <i>n</i> <i>n</i> <i>n</i> <i>n</i> <i>n</i>


-


-=


- + - + + - + 2 2


1 1 1


4 4<i>n</i> 8<i>n</i> 5 4<i>n</i> 1


ổ ử<sub>ữ</sub>




= ỗ<sub>ỗố</sub> <sub>-</sub> <sub>+</sub> - <sub>+</sub> ữ<sub>ữ</sub><sub>ứ</sub>
.
Vỡ thế :



n 4 3 2


1 3 2 1


S ...


5 85 16 32 24 8 5


<i>n</i>


<i>n</i> <i>n</i> <i>n</i> <i>n</i>




-= + + +


- + - + <sub> </sub>


2 2


1 1 1 1 1 1 1


...


4 1 5 5 17 4<i>n</i> 8<i>n</i> 5 4<i>n</i> 1


ổ ử<sub>ữ</sub>





= ỗ<sub>ỗố</sub> - + - + + <sub>-</sub> <sub>+</sub> - <sub>+</sub> ÷<sub>÷</sub><sub>ø</sub>




2 2


2 2 2


1 1 1


1


4 4 1 4 1 4 4


<i>n</i> <i>n</i>


<i>n</i> <i>n</i> <i>n</i>


ổ ử<sub>ữ</sub>




= ỗ<sub>ỗố</sub>- <sub>+</sub> ữ<sub>ữ</sub><sub>ứ</sub>= <sub>+</sub> < =


(pcm).


<b>Bi 2: </b><i>( HSG TỈNH PHÚ YÊN NĂM HỌC 2014 – 2015)</i>


<b>*Cách 1:</b> Gọi ( )<i>S n</i> là tổng các chữ số của số tự nhiên <i>n</i>. Hãy tìm số tự nhiên <i>n</i> biết


2


( ) 2015 8


<i>S n</i> <i>n</i>  <i>n</i> <sub> và 0</sub><i>S n</i>( )<i>n</i><sub>.</sub>


Ta có:


2
2


2015 8 0
0 ( )


2015 8


<i>n</i> <i>n</i>


<i>S n</i> <i>n</i>


<i>n</i> <i>n</i> <i>n</i>


   

 <sub>  </sub>
  



2


2


2015 2014 0
2016 0
<i>n</i> <i>n</i>
<i>n</i> <i>n</i>
   

 
 

 <sub> </sub>

 




1 2014 0


2016 0
<i>n</i> <i>n</i>
<i>n n</i>
  


 
 



 2014<i>n</i>2016 <i>n</i>2015<sub>.</sub>


Thử lại với <i>n </i>= 2015, thấy thỏa <i>S n</i>( )<i>n</i>2 2015<i>n</i>8.
Vậy số tự nhiên cần tìm là <i>n</i> = 2015.


<b>*Cách 2:</b>


</div>
<span class='text_page_counter'>(2)</span><div class='page_container' data-page=2>

( thỏa mãn )



* Nếu n > 2015 thì S(n) = n

2

<sub> – 2015n + 8 = n(n-2015) + 8 </sub>


Vì n > 2015 nên n – 2015

1

 <i>n n</i>

 2015

 8 <i>n</i>

hay S(n) > n


Nên

 <i>n</i> 2015

<sub> khơng có giá trị n thỏa mãn yêu cầu bài toán.</sub>


*Nếu n < 2015:



S(n) = n

2

<sub> – 2015n + 8 = n</sub>

2

<sub> – 1 – 2015n + 2015 – 2006</sub>



= (n -1)(n + 1) – 2015( n -1) – 2006 = (n -1)(n -2014) – 2006


+ n = 0 : S(n) = 8 > 0 ( = n) ( không thỏa mãn)



+ n = 2014: S(n) = - 2006 < 0 ( không thỏa mãn)


+ 0 < n < 2014 :

<i>n</i>1 0; n - 2014 < 0


nên S(n) = (n -1)(n -2014) – 2006 < - 2006 <0



Do đó,

0 <i>n</i> 2015 <i>S n</i>

 

0

nên khơng có giá trị n thỏa mãn yêu cầu bài toán.


Giá trị n thỏa mãn : S(n) = n

2

<sub> – 2015n + 8 và 0 < S(n) </sub>

<sub></sub><i><sub>n</sub></i>

<sub> là n = 2015.</sub>



<b>Bài 3: </b><i>( CHUYÊN LƯƠNG VĂN CHÁNH TỈNH PHÚ YÊN NĂM HỌC 2016 – 2017)</i>


Giả sử <i>x</i>2 6<i>x</i> 6<i>y y</i>2(  )



2 <sub>2</sub>



3 15


<i>x</i> <i>y</i>


   


<i>x</i> 3<i>y x</i>

 

 3 <i>y</i>

15.
Chú ý rằng <i>x</i> 3<i>y x</i>  3 <i>y</i> nên ta có bảng giá trị sau:


3


<i>x</i> <i>y</i> 15 -1 5 -3


3


<i>x</i>  <i>y</i> 1 -15 3 -5


3


<i>x</i> 8 -8 4 -4


<i>x</i> <sub>11</sub> <sub>-5</sub> <sub>7</sub> <sub>-1</sub>


Vậy <i>x</i> 

5; 1;7;11

thì <i>N</i> <i>x</i>2 6<i>x</i> 6<sub> là số chính phương.</sub>


<b>Bài 4: </b><i>( HSG TỈNH PHÚ YÊN NĂM HỌC 2017 – 2018)</i>
Giả sử 7<i>p</i> 1 <i>m m</i>3

 

, mà <i>p</i> 2 <i>m</i>3


khi đó




3 2


7<i>p m</i> 1 <i>m</i>1 <i>m</i> <i>m</i>1
(*)
Vì 7, <i>p</i> là các số nguyên tố, <i>m</i>1 1, <i>m</i>2<i>m</i> 1 1
nên từ (*) suy ra <i>m</i>1 7 <sub> hoặc </sub><i>m</i>2<i>m</i> 1 7<sub>.</sub>


3


) 1 7 8 73; 512 7.73 1


<i>a m</i>   <i>m</i>  <i>p</i> <i>m</i>    <sub>, đúng.</sub>


2 2


b)<i>m</i> <i>m</i>  1 7 <i>m</i> <i>m</i> 6 0 <sub>. </sub>


Giải ra ta được <i>m</i> = 2 hoặc <i>m</i> = -3 đều không thỏa mãn điều kiện <i>m</i>3<sub>.</sub>
Vậy chỉ có số nguyên tố <i>p </i>= 73 là số cần tìm.


<b>Bài 5: </b><i>( HSG TỈNH PHÚ YÊN NĂM HỌC 2018 – 2019)</i>


a) Ta có

 



2 <sub>8</sub> <sub>1</sub> <sub>8</sub> 2 <sub>1</sub> <sub>1</sub> <sub>1 .</sub>
<i>p</i>  <i>q</i>  <i>q</i><i>p</i>   <i>p</i> <i>p</i>


(1)
Do 8q + 1 lẻ nên <i>p</i>2 lẻ  <i>p</i> lẻ



</div>
<span class='text_page_counter'>(3)</span><div class='page_container' data-page=3>

Nếu <i>q</i> 2 4<i>k k</i>

1

 khơng tìm được k
Vậy <i>q</i>2, vì <i>q P q</i> ,  2

2;<i>q</i>

1


Từ (3) suy ra k = 2 và q = k + 1 suy ra k = 2 và q = 3.
Thay kết quả trên vào (2) ta có : <i>p</i>2.2 1 5 
hoặc q = k và k + 1 = 2


1
1
<i>k</i>
<i>q</i>




 




 <sub> (không thỏa mãn)</sub>
Vậy cặp số (p ;q) là (5 ;3) là cặp số cần tìm.


b) Ta có n5 <sub>- n = n(n</sub>4<sub> - 1) = n(n - 1)(n + 1)(n</sub>2<sub> + 1) = (n - 1).n.(n + 1)(n</sub>2<sub> + 1)</sub>
chia hết cho 6 (1)
Vì (n - 1).n.(n+1) là tích của ba số tự nhiên liên tiếp nên chia hết cho 2 và 3
Mặt khác n5<sub> - n = n(n</sub>2<sub> - 1)(n</sub>2<sub> + 1) = n(n</sub>2<sub> - 1).(n</sub>2<sub> - 4 + 5) </sub>


= n(n2<sub> - 1).(n</sub>2<sub> - 4 ) + 5n(n</sub>2<sub> - 1)</sub>


= (n - 2)(n - 1)n(n + 1)(n + 2) + 5n(n2<sub> - 1)</sub>



Vì (n - 2)(n - 1)n(n + 1)(n + 2) là tích của 5 số nguyên liên tiếp nên chia hết cho 5 và 5n(n2<sub> - 1) </sub>
chia hết cho 5. Suy ra n5<sub> – n chia hết cho 5 (2)</sub>


Vậy theo (1) và (2) suy ra n5<sub> – n chia hết cho 30.</sub>


<b>Bài 6: </b><i>( CHUYÊN LƯƠNG VĂN CHÁNH TỈNH PHÚ YÊN NĂM HỌC 2018 – 2019)</i>


a) Chứng minh rằng: <i>a</i>0<sub>, từ đó suy ra </sub>


1 1 1
0
<i>x</i><i>y</i><i>z</i> 


Ta có<i>a x x y</i>

<i>y y z</i>

<i>z z x</i>

(1).


Do <i>x</i>, <i>y</i>, <i>z</i> khác 0, đôi một khác nhau suy ra <i>a</i>0<sub>.</sub>


Từ (1) ta có ; ;


<i>a</i> <i>a</i> <i>a</i>


<i>x y</i> <i>y z</i> <i>z x</i>


<i>x</i>   <i>y</i>   <i>z</i>  


Suy ra


1 1 1



0


<i>a</i> <i>x y y z z x</i>


<i>x</i> <i>y</i> <i>z</i>


 


        


 


  <sub>. Mà </sub><i>a</i>0<sub> nên </sub>


1 1 1
0
<i>x</i><i>y</i><i>z</i>  <sub>.</sub>


b) Chứng minh rằng: 3


<i>x</i> <i>z</i> <i>y</i>
<i>z</i> <i>y</i> <i>x</i> 


Từ


1 1 1
0


<i>x</i><i>y</i><i>z</i>  <sub> suy ra</sub><i>xy yz zx</i>   0 <i>xy</i> <i>yz zx</i> (2).



Vì thế


2 2 2


<i>x</i> <i>z</i> <i>y</i> <i>x y y z z x</i>


<i>z</i> <i>y</i> <i>x</i> <i>xyz</i>


 


  



2 2
2 2


(3)
<i>xyz z x</i> <i>y</i> <i>zx</i>
<i>x yz zx</i> <i>y z z x</i>


<i>xyz</i> <i>xyz</i>


   


   


  


Theo giả thiết <i>x</i>2 <i>xy</i><i>y</i>2 <i>yz</i> <i>x</i>2 <i>y</i>2 <i>xy yz</i> (4).


Kết hợp (2), (3) và (4):



<sub>3</sub>


<i>xyz z xy yz zx</i> <i>xyz z xy xy</i>
<i>x z y</i>


<i>z y x</i> <i>xyz</i> <i>xyz</i>


      


     


<b>Bài 7: </b><i>( CHUYÊN LƯƠNG VĂN CHÁNH TỈNH PHÚ YÊN NĂM HỌC 2019 – 2020)</i>


</div>
<span class='text_page_counter'>(4)</span><div class='page_container' data-page=4>

2 2 2


1
?
2019


<i>a</i> <i>b</i> <i>c</i>


<i>b</i>  <i>ac</i> <i>c</i>  <i>ab</i> <i>a</i>  <i>bc</i> 


Giả sử tồn tại ba số thực , ,<i>a b c</i>thỏa yêu cầu bài tốn. Khi đó,




2 2 2



0


4.1


0, 0, 0


<i>abc</i>


<i>b</i> <i>ac</i> <i>a</i> <i>bc</i> <i>c</i> <i>ab</i>






     




Khi đó, ta có


2 <sub>2</sub> <sub>2</sub> <sub>2</sub> <sub>2</sub>


2 2 2 2


2


2 2 2 2


2



1


2019 <sub>2019</sub> <sub>2019</sub>


1
2019 2019
2019
2019 2019
1
2019
<i>a</i>


<i>b</i> <i>ac</i> <i><sub>b</sub></i> <i><sub>ac</sub></i> <i><sub>a</sub></i> <i><sub>b a a c</sub></i> <i><sub>a</sub></i>


<i>b</i>


<i>c</i> <i>ab</i> <i>b</i> <i>c b ab</i> <i>b</i>


<i>c</i> <i>ab</i>


<i>a</i> <i>bc</i> <i>c</i> <i>a c bc</i> <i>c</i>


<i>c</i>
<i>a</i> <i>bc</i>


 <sub></sub>
     

 



      
  

  
   
 


 <sub></sub>

Suy ra,


2 2 2

 

2 2

 

2 2

 

2 2



2019 0


0.


<i>a</i> <i>b</i> <i>c</i> <i>b a a c</i> <i>c b ab</i> <i>a c bc</i>
<i>a b c</i>


        


   


Điều này mâu thuẫn với

4.1 .

Vậy không tồn tại ba số thực , ,<i>a b c</i> thỏa u cầu bài tốn.


b) Tìm tất cả các cặp số nguyên dương

<i>x y</i>,

thỏa mãn



2 2 <sub>85</sub>


.
13
<i>x</i> <i>y</i>
<i>x y</i>




Gọi

<i>x y</i>,

là cặp số nguyên thỏa yêu cầu bài toán, rõ ràng <i>x y</i> 0.


Khi đó, ta có


2 2

2


2
170


.
13


<i>x</i> <i>y</i> <i><sub>x y</sub></i>


<i>x y</i>


<i>x y</i> <i>x y</i>


 



   


  <sub> Suy ra </sub>0 <i>x y</i>13

4.2 .



Mặt khác, ta có






2 2


85 13


13 4.3 .
85,13 1


<i>x y</i> <i>x</i> <i>y</i>


<i>x y</i>
 <sub></sub> <sub></sub> <sub></sub>

 






Từ (4.2) và (4.3) suy ra <i>x y</i> 13. Kết hợp với giả thiết, ta nhận được hệ phương trình



2 2
13
4.4 .
85
<i>x y</i>
<i>x</i> <i>y</i>
 


 


 <sub> Giải (4.4), ta thu được hai cặp nghiệm </sub>

6;7 , 7;6

 

<sub>.</sub>


<b>Bài 8: </b><i>( HSG TỈNH PHÚ YÊN NĂM HC 2012 2013- D PHềNG)</i>
Để <i>n</i>18 và <i>n</i> 41 là hai số chính phơng


2
18


<i>n</i> <i>p</i>


<sub>và</sub><i>n</i> 41<i>q p q</i>2

, <b>N</b>

 <i>p</i>2 <i>q</i>2 

<i>n</i>18

 

 <i>n</i> 41

59

<i>p q p q</i>

 

59
Nhng 59 là số nguyên tố, nên:


1 30


59 29


<i>p q</i> <i>p</i>



<i>p q</i> <i>q</i>


  
 

 
  
 

2 2


18 30 900


<i>n</i> <i>p</i>   <sub> suy ra </sub><i><sub>n</sub></i><sub></sub><sub>882</sub>


Thay vào <i>n</i> 41, ta đợc 882 41 841 29   2 <i>q</i>2.


VËy víi <i>n</i>882 thì <i>n</i>18 và <i>n</i> 41 là hai số chính ph¬ng


<b>Bài 9: </b><i>( HSG TỈNH BÀ RỊA - VŨNG TÀU NĂM HỌC 2008 – 2009)</i>


Ta có: <i>abc</i>100<i>a</i>10<i>b c n</i>  21 (1)


</div>
<span class='text_page_counter'>(5)</span><div class='page_container' data-page=5>

Mặt khác: 100<i>n</i>2 1 999101<i>n</i>2 100011 <i>n</i> 31
 39 4 <i>n</i> 5 119 (4). Từ (3) và (4) suy ra n = 26.
Vậy <i>abc</i>675<sub>.</sub>


<b>Bài 10:</b><i> ( HSG TỈNH BÀ RỊA - VŨNG TÀU NĂM HỌC 2008 – 2009)</i>



Ta có kết quả quen thuộc sau đây:


1 1 1


... 2 2


2 3


<i>A</i> <i>n</i>


<i>n</i>


     


Thật vậy: Từ



1 2 1


2 1


2 1 <i>k</i> <i>k</i>


<i>k</i>  <i>k</i>  <i>k</i>  <i>k</i>    <sub>, suy ra:</sub>


2 ( 2 1) ( 3 2) ... ( 1) 2( 1) 2 2


<i>A</i>       <i>n</i>  <i>n</i>   <i>n</i>  <i>n</i>


  <sub> (*)</sub>



Giả sử trong 100 số tự nhiện đã cho khơng có hai số nào bằng nhau. Khơng mất tính tổng qt, giả
sử: <i>a</i>1<i>a</i>2...<i>a</i>100 <i>a</i>11,<i>a</i>2 2,...<i>an</i> 100


Thế thì: 1 2 100


1 1 1 1 1 1


... ...


1 2 100


<i>a</i>  <i>a</i>   <i>a</i>     <sub></sub><sub>2 100 1 19</sub><sub></sub> <sub></sub>


(áp dụng (*))
Kết quả này trái với giả thiết. Vậy tồn tại ít nhất hai số bằng nhau trong 100 số đã cho.


<b>Bài 11:</b><i> ( HSG TỈNH BẮC GIANG NĂM HỌC 2009 – 2010)</i>


a) + Xét <i>p</i> = 3 thì 2<i>p</i> + 1 = 7; 4<i>p</i> + 1 = 13 đều là số nguyên tố.


+ XÐt <i>p</i> ¿ <sub> 3: do </sub><i><sub>p</sub></i><sub> là số nguyên tố nên </sub><i><sub>p</sub></i><sub> = 3</sub><i><sub>k</sub></i><sub> + 1 (</sub><i><sub>k</sub></i><sub></sub><sub> 1) hoăc </sub><i><sub>p</sub></i><sub> = 3</sub><i><sub>k</sub></i><sub> + 2</sub><i><sub>(k</sub></i><sub></sub><sub> 0)</sub>


<i>p</i> = 3<i>k</i> + 1  2<i>p</i> + 1 = 2(3<i>k</i> + 1) + 1 = 3(2<i>k</i> + 1) là hợp số
<i>p</i> = 3k + 2  4<i>p</i> + 1 = 4(3<i>k</i> + 2) + 1 = 3(<i>4k</i> + 3) là hợp số.
Tóm lại <i>p</i> = 3 là số cần tìm.


b) Do <i>p</i> là số nguyên tố nên các ớc cđa <i>p</i>4<sub> lµ 1; </sub><i><sub>p</sub></i><sub>; </sub><i><sub>p</sub></i>2<sub>; </sub><i><sub>p</sub></i>3<sub>; </sub><i><sub>p</sub></i>4


Theo bµi ta cã <i>n</i>2<sub> = </sub><i><sub>p</sub></i>4<sub> + </sub><i><sub>p</sub></i>3<sub> +</sub><i><sub> p</sub></i>2<sub> + </sub><i><sub>p</sub></i><sub> + 1</sub>



Do đó 4<i>n</i>2<sub> = 4</sub><i><sub>p</sub></i>4<sub> + 4</sub><i><sub>p</sub></i>3<sub> + 4</sub><i><sub>p</sub></i>2<sub> + 4</sub><i><sub>p</sub></i><sub> + 4 </sub>


= (2<i>p</i>2<sub> + </sub><i><sub>p</sub></i><sub>)</sub>2<sub> + 3</sub><i><sub>p</sub></i>2<sub> + 4</sub><i><sub>p</sub></i><sub> + 4 > (2</sub><i><sub>p</sub></i>2<sub> + </sub><i><sub>p</sub></i><sub>)</sub>2


do 3<i>p</i>2<sub> + 4</sub><i><sub>p</sub></i><sub> + 4 > 0 víi mäi sè nguyªn tè </sub><i><sub>p</sub></i><sub>.</sub>


 2<i>n</i> > 2<i>p</i>2<sub> + </sub><i><sub>p </sub></i><sub> (3)</sub>


Mặt khác 4<i>n</i>2<sub> = 4</sub><i><sub>p</sub></i>4<sub> + 4</sub><i><sub>p</sub></i>3<sub> + 4</sub><i><sub>p</sub></i>2<sub> + 4</sub><i><sub>p</sub></i><sub> + 4</sub>


= (2<i>p</i>2<sub> + </sub><i><sub>p</sub></i><sub> + 1)</sub>2<sub> - </sub><i><sub>p</sub></i>2<sub> + 2</sub><i><sub>p</sub></i><sub> + 3 </sub>


= (2<i>p</i>2<sub> + </sub><i><sub>p</sub></i><sub> + 1)</sub>2<sub> - (</sub><i><sub>p</sub></i><sub> - 1)</sub>2<sub> + 4 </sub>


XÐt <i>p</i> > 3 th× - (<i>p</i> - 1)2<sub> + 4 < 0</sub>


 4<i>n</i>2<sub> < (2</sub><i><sub>p</sub></i>2<sub> + </sub><i><sub>p</sub></i><sub> + 1)</sub>2<sub></sub><sub> 2</sub><i><sub>n</sub></i><sub> < </sub><i><sub>2p</sub>2</i><sub> + </sub><i><sub>p</sub></i><sub> + 1 (4)</sub>


Tõ (3) vµ (4) suy ra 2<i>p</i>2<sub> + </sub><i><sub>p</sub></i><sub> < 2</sub><i><sub>n</sub></i><sub> < 2</sub><i><sub>p</sub></i>2<sub> + </sub><i><sub>p</sub></i><sub> + 1 (v« lÝ)</sub>


Suy ra <i>p</i> 3. Do <i>p</i> là số nguyên tố nên <i>p</i> = 2 hoặc <i>p</i> = 3.
Thử lại đợc <i>p</i> = 3 thỏa mãn yêu cầu bài toán.


<b>Bài 12:</b><i> ( HSG TỈNH BẮC GIANG NĂM HỌC 2009 – 2010)</i>


a)Xét <i>( a2<sub> + b</sub>2<sub> + c</sub>2<sub> + d</sub>2<sub> ) - ( a + b + c + d) </sub></i>
<i> = a(a -1) + b( b -1) + c( c – 1) + d( d – 1)</i>


Vì a là số nguyên dương nên a, (a – 1) là hai số tự nhiên liên tiếp
 <i><sub>a(a-1)</sub></i>2<sub> tương tự ta có </sub><i><sub>b(b-1); c(c-1); d(d-1)</sub></i><sub> đều chia hết cho 2</sub>


 <i><sub>a(a -1) + b( b -1) + c( c – 1) + d( d – 1)</sub></i><sub> là số chẵn</sub>


Lại có <i>a2<sub> + c</sub>2<sub> = b</sub>2<sub> + d</sub>2</i> <sub></sub> <i><sub> a</sub>2<sub> + b</sub>2<sub> + c</sub>2<sub> + d</sub>2<sub> = 2( b</sub>2<sub> + d</sub>2<sub>)</sub></i><sub> là số chẵn.</sub>
Do đó <i>a + b + c + d</i> là số chẵn mà <i>a + b + c + d > 2</i>


 <i><sub>a + b + c + d</sub></i><sub> là hợp số.</sub>
b) (<i>x</i>2 3) (<i>xy</i>3)(1)


</div>
<span class='text_page_counter'>(6)</span><div class='page_container' data-page=6>

 <i><sub>x(xy +3) – 3(x+y) </sub></i>(<i>xy</i>3)  <i><sub>3(x+y) </sub></i>(<i>xy</i>3)
<i> </i> <i><sub> 3( x+y) = k(xy+3) ( k</sub></i><i>N</i>*<i><sub>) (2)</sub></i>


+Nếu <i>k</i>3<sub>thì </sub>3( <i>x y</i> ) (<i>k xy</i>3) 3( <i>xy</i>3) <i>x y xy</i>   3

<i>x</i>1

 

<i>y</i>1

 2 0
(Vơ lí vì <i>x, y</i> ngun dương)


+Nếu k = 1 thì từ (2)  <i><sub>(x-3)(y-3) =6</sub></i><sub>, mà x, y là các số nguyên dương nên </sub><i><sub>x = 6</sub></i><sub> và </sub>
<i>y = 5</i> hoặc <i>x = 5</i> và <i>y= 6</i> hoặc <i>x=4</i> và <i>y=9</i> hoặc <i>x=9</i> và <i>y=4.</i>


Thử lại thấy <i>x = 6</i> và <i>y = 5</i> hoặc <i>x=9</i> và <i>y=4</i> thỏa mãn (1).


+Nếu k=2 thì từ (2) ta có: <i>3( x+y) = 2(xy+3) </i>suy ra xy chia hết cho 3 (*)


mặt khác<i> 3( x+y) = 2(xy+3)</i>  <i><sub>y(x-3)+x(y-3)+6=0</sub></i><sub> suy ra </sub><i><sub>x ></sub></i><sub>3 và </sub><i><sub>y ></sub></i><sub>3 vơ lý. (**)</sub>
Từ (*) và (**) ta có

x; y

1;3 , 3;1 .

 

Thử lại vào (1) ta được <i>(x;y)</i>=(3;1).
Vậy

, <i>x y</i>

6;5 ; 9; 4 ; 3;1 .

 

 



<b>Bài 13:</b><i> ( HSG TỈNH BẮC GIANG NĂM HỌC 2012 – 2013)</i>


+ Từ giả thiết suy ra:


2 1 1 1 7



3<i>a b c</i>  10<sub>. Khơng giảm tính tổng qt giả sử </sub><i>a b c</i>  1<sub>. Suy </sub>
ra


2 3


2 9


3<i>c</i> <i>c</i>
Do đó <i>c</i>{2;3}
+ Với <i>c</i>2<sub> suy ra </sub>


2 1 1 1 7 1 1 1 1 1 2 1 1


(1) và


32<i>a b</i> 10 6<i>a b</i> 5  6<i>b</i> <i>b</i>12
Do đó <i>b</i>{7;11}


+ Với <i>b</i>7<sub> từ (1) suy ra </sub>


1 1 2


{19; 23; 29;31;37;41}
42<i>a</i> 35 <i>a</i>


+ Với <i>b</i>11<sub> từ (1) suy ra </sub>


5 1 6



13


66<i>a</i>55 <i>a</i> <sub> ( do a > b)</sub>
+ Với <i>c</i>3<sub> từ giả thiết suy ra </sub>


1 1 1 11 1 2


(*) 6 5


3<i>a b</i> 30  3<i>b</i>  <i>b</i>  <i>b</i> <sub> ( do b>c)</sub>
Thay <i>b</i>5<sub> vào (*) được </sub>


15


6 7


2


<i>a</i> <i>a</i>


   


.
Vậy có 48 bộ ba (a;b;c) thoả mãn:


(19;7;2),(23;7;2),(29;7; 2),(31;7;2),(37; 7;2),(41;7;2),(13;11; 2), (7;5;3) và các hốn vị của nó.


<b>Bài 14:</b><i> ( HSG TỈNH BẮC GIANG NĂM HỌC 2016 – 2017)</i>


Ta có:

  




2


27 2016 27 1989 27


4 4 4<i>n</i> 2 1 4 4<i>n</i>


<i>A</i> 


     


Vì A và


2
27
2


là số chính phương nên 1 41989 4<i>n</i>27


  <sub> là số chính phương</sub>
Ta có 1 4<sub></sub> 1989<sub></sub>4<i>n</i>27


>4<i>n</i>27 (2<i>n</i>27 2)
Mà 1 41989 4<i>n</i>27


  <sub> là số chính phương nên ta có </sub>
1989 27


1 4<sub></sub> <sub></sub>4<i>n</i>



2


27
2<i>n</i> 1


  <sub>2</sub><i>n</i>27 <sub>2</sub>3977 <i><sub>n</sub></i> <sub>4004</sub>


</div>
<span class='text_page_counter'>(7)</span><div class='page_container' data-page=7>

Với n = 4004 ta có


2
27 2016 4004 27 4004


4 4 4 2 2


<i>A</i>    


là số chính phương
Vậy n = 4004 thì A = 427 <sub>+ 4</sub>2016 <sub>+ 4</sub>n <sub>là số chính phương.</sub>


<b>Bài 15:</b><i> ( HSG TỈNH BẮC GIANG NĂM HỌC 2017 – 2018)</i>


a) Đặt


2 2


3; 2 3;


<i>a x</i> <i>b x</i> <i>c x</i>


<i>x</i>


     



với , ,<i>a b c Z</i>


Từ <i>a x</i>  3 <i>x a</i>  3; từ <i>b x</i> 22 3 <i>x</i>2  <i>b</i> 2 3, nên ta có

<i><sub>a</sub></i> <sub>3</sub>

2 <i><sub>b</sub></i> <sub>2 3</sub> <i><sub>a</sub></i>2 <sub>2 3</sub><i><sub>a</sub></i> <sub>3</sub> <i><sub>b</sub></i> <sub>2 3</sub> <sub>2 3</sub>

<i><sub>a</sub></i> <sub>1</sub>

<i><sub>b a</sub></i>2 <sub>3</sub>


            


-Nếu a + 1<sub>0</sub>


2 <sub>3</sub>
1 2 3


1
<i>b a</i>
<i>a</i>


<i>a</i>
 


   


 <sub>, vì </sub>


2 <sub>3</sub>


, 2 3


1
<i>b a</i>



<i>a b Z</i> <i>Q</i> <i>Q</i>


<i>a</i>
 


     


 <sub>Vô lý</sub>


Vậy a + 1=0 nên ta có 2


1 0 1


4
3 0


<i>a</i> <i>a</i>


<i>b</i>
<i>b a</i>


  


 


 


 





   <sub></sub>


<i>x</i>

3 1



Với <i>x</i> 3 1 <sub> ta có </sub><i>a</i>1;<i>b</i>4<sub> và </sub><i><sub>c</sub></i><sub></sub><sub>2</sub><sub> nguyên, thỏa mãn đầu bài.</sub>


b) Vì 4<i>x</i>314<i>x</i>29<i>x</i> 6<sub> là số chính phương, nên ta có </sub>4<i>x</i>314<i>x</i>29<i>x</i> 6<sub>=k</sub>2<sub> với </sub><i>k</i><sub></sub><sub>N</sub>
Ta có 4<i>x</i>314<i>x</i>29<i>x</i> 6<sub>=…=</sub>



2


2 4 6 3


<i>x</i> <i>x</i>  <i>x</i>


nên ta có



2


2 4 6 3


<i>x</i> <i>x</i>  <i>x</i>


=<i>k</i>2


Đặt



2



2, 4 6 3


<i>x</i> <i>x</i>  <i>x</i> <i>d</i>


với d<i>N</i>*


Ta có <i>x</i>2<i>d</i> 

<i>x</i>2 4

 

<i>x</i> 2

<i>d</i> 4<i>x</i>6<i>x</i> 4<i>d</i>


Ta lại có

 



2 2 2


4<i>x</i> 6<i>x</i> 3<i>d</i> 4<i>x</i> 6<i>x</i> 3  4<i>x</i> 6<i>x</i> 4 1<i>d</i> <i>d</i> 1


Vậy



2


2, 4 6 3 1


<i>x</i> <i>x</i>  <i>x</i> 




2


2 4 6 3


<i>x</i> <i>x</i>  <i>x</i>



=<i>k</i>2 nên ta có


x + 2 và 4<i>x</i>26<i>x</i> 3<sub> là số chính phương</sub> <i>x</i> 2 <i>a v</i>2 à 4x26<i>x</i> 3<i>b</i>2<sub> với a,b</sub><i>N</i>*


Vì x > 0 nên ta có



2 2


2 2 2 2


4<i>x</i> <i>b</i> 4<i>x</i> 12<i>x</i> 9 2<i>x</i> <i>b</i>  2<i>x</i>3
Vì b lẻ nên



2


2 <sub>2</sub> <sub>1</sub> <sub>4</sub> 2 <sub>6</sub> <sub>3 4</sub> 2 <sub>4</sub> <sub>1</sub> <sub>2</sub>
<i>b</i>  <i>x</i>  <i>x</i>  <i>x</i>  <i>x</i>  <i>x</i>  <i>x</i>
Với x=2 ta có 4<i>x</i>314<i>x</i>29<i>x</i> 6<sub>=100=10</sub>2<sub> là số chính phương.</sub>


c) -Nếu n là lũy thừa bậc 2 của 1 số tự nhiên bài toán chứng minh xong


-Nếu n không là lũy thừa bậc 2 của 1 số tự nhiên, ta ln tìm được 1 số ngun dương k sao cho


2


2 <sub>1</sub>


<i>k</i> <i>n</i> <i>k</i>


.Vì n nguyên dương và <i>n k</i> 2  <i>n k</i> 21<sub>, vậy ta có:</sub>



2 2

2 2

2


2<i>n</i> <i>k</i>1 2(<i>k</i> 1) <i>k</i>1  ... <i>k</i>  2<i>k</i> 1 <i>k</i>1 0
Vậy mọi k nguyên dương , nên ta có



2


2 <sub>1</sub>


<i>k</i> <i>n</i> <i>k</i> <sub></sub><sub>2n</sub>


Vậy trong dãy ln có ít nhất một lũy thừa bậc 2 của 1 số tự nhiên.


<b>Bài 16:</b><i> ( HSG TỈNH BẮC GIANG NĂM HỌC 2018 – 2019)</i>


</div>
<span class='text_page_counter'>(8)</span><div class='page_container' data-page=8>

Giả sử hai số trong 12 số đó khi chia cho 11 có cùng số dư là <i>abc mnp</i>;
Suy ra

(

<i>abc mnp</i>-

)

M11.


Khi ghép chúng cạnh nhau ta được số có sáu chữ số <i>abcmnp</i>.
Ta có


(

)

(

)



1000. 1001. 11.91.


<i>abcmnp</i>= <i>abc mnp</i>+ = <i>abc</i>+ <i>mnp abc</i>- = <i>abc</i>+ <i>mnp abc</i>


-Suy ra <i>abcmnp</i>M11. Điều phải chứng minh.



<b>Bài 17:</b><i> ( HSG TỈNH BẮC NINH NĂM HỌC 2018 – 2019)</i>


Đặt <i>p</i>3- 4<i>p</i>+ =9 <i>t t N</i>2( Ỵ )
Biến đổi thành


(

2 <sub>4</sub>

)

<sub>(</sub> <sub>3)(</sub> <sub>3) (1)</sub> <sub>|</sub>

(

<sub>3</sub>

)

<sub>|</sub>

(

<sub>3</sub>

)



<i>p p</i> - = -<i>t</i> <i>t</i>+ Þ <i>p t</i>- Ú<i>p t</i>+


<i>Trường hợp</i> 1: Nếu <i>p t</i>| - 3
Đặt <i>t</i>- 3=<i>pk k N</i>( Ỵ )
Khi đó thay vào (1) ta có:


(

2 <sub>4</sub>

)

<sub>(</sub> <sub>6)</sub> 2 2 <sub>6</sub> <sub>4</sub> <sub>0</sub>


<i>p p</i> - =<i>pk pk</i>+ Û <i>p</i> - <i>pk</i> - <i>k</i>- =


Coi đây là phương trình bậc hai ẩn <i>p</i><sub> điều kiện cần để tồn tại nghiệm của phương trình là:</sub>


(

)



4 <sub>4</sub> <sub>6</sub><i><sub>k</sub></i> <sub>4</sub> 4 <sub>24</sub><i><sub>k</sub></i> <sub>16</sub>


<i>k</i> <i>k</i>


D = + + = + +


là một số chính phương.


Mặt khác với <i>k</i>>3ta dễ chứng minh được

( )

(

)




2 2


2 4 <sub>24</sub> <sub>16</sub> 2 <sub>4</sub>


<i>k</i> <<i>k</i> + <i>k</i>+ < <i>k</i> +


Suy ra các trường hợp:


(

)

2


4 <sub>24</sub> <sub>16</sub> 2 <sub>1</sub> <sub>2</sub> 2 <sub>24</sub> <sub>15</sub> <sub>0</sub>


<i>k</i> + <i>k</i>+ = <i>k</i> + Û <i>k</i> - <i>k</i>- =


(loại)


(

)

2


4 <sub>24</sub> <sub>16</sub> 2 <sub>2</sub> 2 <sub>6</sub> <sub>3</sub> <sub>0</sub>


<i>k</i> + <i>k</i>+ = <i>k</i> + Û <i>k</i> - <i>k</i>- =


(loại)


(

)

2


2 <sub>24</sub> <sub>16</sub> 2 <sub>3</sub> <sub>6</sub> 2 <sub>24</sub> <sub>7</sub> <sub>0</sub>


<i>k</i> + <i>k</i>+ = <i>k</i> + Û <i>k</i> - <i>k</i>- =



(loại)
Do đó phải có<i>k</i>£ 3. Thử trực tiếp được <i>k</i>=3 thỏa mãn.
Từ đó ta có <i>t</i>=36;<i>p</i>=11.


<b>Lưu ý:</b> HS có thể làm như sau khi thay vào

( )

1


(

2 <sub>4</sub>

)

<sub>(</sub> <sub>3)</sub> <sub>(</sub> <sub>3)</sub> 2 <sub>4</sub> 2 <sub>3</sub> <sub>4</sub>


<i>p p</i> - =<i>pk t</i>+ Û <i>k t</i>+ =<i>p</i> - Þ <i>p</i> =<i>kt</i>+ <i>k</i>+
Mặt khác ta có(<i>t</i>- 3)2=<i>p k</i>2 2 Þ <i>t</i>2- 6<i>t</i>+ =9 <i>k kt</i>2( +3<i>k</i>+4)


(

)



2 <sub>6</sub> 3 <sub>9 3</sub> 3 <sub>4</sub> 2 <sub>0</sub>


<i>t</i> <i>t</i> <i>k</i> <i>k</i> <i>k</i>


Û - + + - - =


Coi đây là phương trình bậc hai ẩn <i>n</i>điều kiện cần để tồn tại nghiệm của phương trình là:


(

)

2

(

)

(

)



3 3 2 6 3 2 2 4


6 <i>k</i> 4 9 3<i>k</i> 4<i>k</i> <i>k</i> 24<i>k</i> 16<i>k</i> <i>k k</i> 24<i>k</i> 16


D = + - - - = + + = + +



là một số chính
phương. Muốn vậy thì <i>k</i>4+24<i>k</i>+16 phải là một số chính phương.


Sau đó cách làm giống như trên.


<b>Bài 18:</b><i> ( HSG TỈNH BÌNH ĐỊNH NĂM HỌC 2009 – 2010)</i>


</div>
<span class='text_page_counter'>(9)</span><div class='page_container' data-page=9>

1

 

2

 

3

 

4



( )

( ).

.

.

.

2003



<i>P x</i>

<i>h x x x</i>

<i>x x</i>

<i>x x</i>

<i>x x</i>



. Trong đó: h(x) là đa thức bậc nhất với
hệ số nguyên ( chẳng hạn: h(x) = ax + b; a, b nguyên);

<i>x Z</i>

<i>i</i>

;

 

<i>i</i>

1,4

<sub>. </sub>


Ta sẽ dùng phản chứng, giả sử rằng tại x = x0 nguyên ta có: P(x0) = 2010 suy ra:




0 0


2010

<i>h x</i>

( ).

<i>x</i>

<i>x</i>

<i><sub>i</sub></i>

2003;

 

<i>i</i>

1,4

<i>h x</i>

( ).

<sub>0</sub>

<i>x</i>

<sub>0</sub>

<i>x</i>

<i><sub>i</sub></i>

7;

 

<i>i</i>

1,4

 

*



<i>x</i>

0

<i>x</i>

<i>i</i>

;

 

<i>i</i>

1,4

<sub> nên vế trái của (*) là tích của ít nhất 4 số ngun khác nhau và khác khơng.</sub>
Cịn VP = 7 chỉ có thể phân tích 1.7 hoặc (-1).(-7) là tích hai số nguyên khác nhau. Nên đẳng thức (*)
không thể xảy ra. Hay giả sử trên là sai. Bài toán được chứng minh.


<b>Bài 19:</b><i> ( HSG TỈNH BÌNH ĐỊNH NĂM HỌC 2011 – 2012).</i>



a) n3<sub> – n = n(n</sub>2<sub> – 1) = n(n – 1)(n + 1).</sub>


Với n lẻ thì n – 1 và n + 1 là hai số chẵn liên tiếp nên có một số chia hết cho 2 , số còn lại chia hết
cho 4. Do đó tích (n – 1)(n + 1) chia hết cho 8.


Mặt khác, tích n(n – 1)(n + 1) là tích của ba số tự nhiên liên tiếp nên chia hết cho 3.
Mà (3; 8) = 1 nên n(n – 1)(n + 1) chia hết cho 3.8 = 24.


Vậy : n3<sub> – n chia hết cho 24 với n lẻ.</sub>


b) Từ điều kiện : a2<sub> + b</sub>2<sub> + c</sub>2<sub> = (a –b)</sub>2<sub> + (b- c)</sub>2<sub> + ( c – a)</sub>2
Suy ra : a2<sub> + b</sub>2<sub> + c</sub>2<sub> = 2(a</sub>2<sub> + b</sub>2<sub> + c</sub>2<sub>) – 2(ab + bc + ca)</sub>
 <sub> a</sub>2<sub> + b</sub>2<sub> + c</sub>2<sub> = 2(ab + bc + ca)</sub>


* Cách 1 :


Nếu c <sub> a và c </sub><sub> b thì a</sub>2<sub> + b</sub>2<sub> + c</sub>2<sub> = 2ab + 2bc + 2ca </sub><sub></sub><sub> 2ab + 2b</sub>2<sub> + 2a</sub>2
 <sub> c</sub>2<sub> </sub><sub></sub><sub> ( a + b)</sub>2<sub> </sub><sub></sub> <sub> c </sub><sub></sub><sub> a + b ( vì a, b, c > 0 )</sub>


* Cách 2 :


Từ a2<sub> + b</sub>2<sub> + c</sub>2<sub> = 2ab + 2bc + 2ca </sub><sub></sub> <sub> a</sub>2<sub> + b</sub>2<sub> + c</sub>2<sub> - 2ab - 2bc - 2ca = 0 </sub><sub></sub> <sub> ( a – b + c )</sub>2<sub> = 4ca . </sub>
Nếu c <sub> a thì ( a – b + c )</sub>2<sub> = 4ca </sub><sub></sub><sub> 4a</sub>2<sub> </sub><sub></sub> <sub> ( a – b + c )</sub>2<sub> - 4a</sub>2 <sub></sub><sub> 0 </sub>


 <sub> (a – b + c – 2a )(a – b + c + 2a ) </sub><sub> 0 </sub> <sub> ( c – a – b )(3a – b + c) </sub><sub> 0</sub>
Vì a > 0 và nếu c <sub> b thì 3a – b + c > 0 </sub> <sub> c – a – b </sub><sub> 0 </sub> <sub> c </sub><sub> a + b .</sub>


<b>Bài 20:</b><i> ( HSG TỈNH BÌNH ĐỊNH NĂM HỌC 2016 – 2017).</i>


Ta có: a + b + c <sub> 4 (a, b, c </sub><sub> Z)</sub>



Đặt a + b + c = 4k (k <sub> Z) </sub> <sub>a + b = 4k – c ; b + c = 4k – a ; a + c = 4k – b </sub>
Ta có: P = (a + b)(b + c)(c + a) – abc = (4k – c)(4k – a)(4k – b) – abc


=



2


16<i>k</i>  4<i>ak</i>  <i>ack</i>  <i>ac</i> 4<i>k</i>  <i>b</i>  <i>abc</i>


= 64<i>k</i>3  16<i>bk</i>2  16<i>ak</i>2  4<i>abc</i>  16<i>ck</i>2  4<i>bck</i>  4<i>ack</i>  <i>abc</i>  <i>abc</i>


=



3 2 2 2


4 16<i>k</i>  4<i>bk</i>  4<i>ak</i>  <i>abk</i>  4<i>ck</i> <i>bck</i>  <i>ack</i>  2<i>abc</i>
(*)
Giả sử a, b, c đều chia 2 dư 1  <sub> a+ b + c chia 2 dư 1 (1)</sub>


Mà: a + b + c <sub> 4 </sub> <sub>a + b + c </sub><sub> 2 (theo giả thiết) (2)</sub>
Do đó (1) và (2) mâu thuẫn  <sub> Điều giả sử là sai </sub>


 <sub>Trong ba số a, b, c ít nhất có một số chia hết cho 2</sub>
 <sub>2abc </sub><sub> 4 (**)</sub>


Từ (*) và (**) <sub> P </sub><sub> 4. </sub>


</div>
<span class='text_page_counter'>(10)</span><div class='page_container' data-page=10>

a) Ta có

 




2 2


6 4 2 2 2


n  2n + n n n  1 <sub></sub>n n + 1 n + 2 <sub></sub>


Lại có: n n + 1 2; n n

 1 n

 

1 3

 (vì n N*) mà (2 ; 3) = 1 nên n n

 1 n

 

1 6


Đặt A = n n

 1 n

 

1

ta có A <sub> 6 </sub> A 362


Vậy n6 2n + n4 2 chia hết cho 36 với mọi n N*


b) Ta có:



2
x  a + b + c  9ab


; y =



2
a + b + c  9bc


; z =



2
a + b + c  9ac
Do đó: x + y + z


2

<sub></sub>

2 2 2

<sub></sub>



3 a + b + c 9 ab + bc + ca 3 a + b + c 6 ab + bc + ca 9 ab + bc + ca



    


=



2 2 2 2 2 2


3 a + b + c  3 ab + bc + ca 3 a + b + c  ab  bc  ca


=



2 2 2


3


2a + 2b + 2c 2ab 2bc 2ca


2   



2 2 2


3


a b + b c + c a


2 


   


 



Vì a, b, c phân biệt nên



2 2 2


a  b + b  c + c a <sub> > 0 </sub> <sub>x + y + z</sub> <sub>0</sub><sub> (1)</sub>
Giả sử cả ba số x, y, z <sub> 0. Từ (1) suy ra vơ lí</sub>


Nếu có 2 số nhỏ hơn 0 hoặc 2 số bằng 0 thì thì từ (1) suy ra có ít nhất 1 số lớn hơn 0
Nếu có 1 số nhỏ hơn 0 và 1 số bằng 0 thì từ (1) suy ra đpcm.


<b>Bài 22:</b><i> ( HSG TỈNH BÌNH ĐỊNH NĂM HỌC 2018 – 2019).</i>


Gọi A Ai j<sub> là hai điểm xa nhau nhất trong các điểm thuộc tập hợp 8073 điểm đã cho .</sub>
Giả sử A là điểm cách xa đoạn thẳng k A Ai j<sub>nhất . Khi đó</sub>


Tam giác A Ai jA là tam giác lớn nhất và có diện tích hơng lớn hơn 1k


Vẽ các đường thẳng đi qua các điểm A , i Aj<sub>, </sub>A lần lượt song song với các cạnh của k <sub></sub>A Ai j Ak
Ta được 4 tam giác nhỏ bằng nhau và một tam giác lớn chứa cả 4 tam giác nhỏ


Tam giác lớn có diện tích khơng q 4 đơn vị. Do đó, tam giác lớn chứa tất cả 8073 điểm đã cho
Ta có 8073 chia cho 4 được 2018 và dư là 1 nên theo ngun lý Dirichlet suy ra có ít nhất 1 trong 4
tam giác có 1 tam giác chứa 2019 trong 8073 điểm đã cho.


<b>Bài 23:</b><i> ( HSG TỈNH BÌNH DƯƠNG NĂM HỌC 2012 – 2013).</i>


a) Ta có: n6<sub> - n</sub>4<sub> – n</sub>2<sub> + 1 = n</sub>4<sub> (n</sub>2<sub> - 1)- (n</sub>2<sub> – 1) = (n</sub>2<sub> – 1)(n</sub>4<sub> – 1) = (n</sub>2<sub> – 1)</sub>2<sub>(n</sub>2<sub> + 1)</sub>
Vì n lẻ , nên n = 2k + 1 với k là số tự nhiên.



Khi đó: (n2<sub> – 1) = 4k</sub>2<sub> + 4k = 4k(k + 1) chia hết cho 4</sub>
Mặt khác: k(k + 1) chia hết cho 2


 (n2<sub> – 1)</sub>2<sub> chia hết cho (4.2)</sub>2<sub> = 64 (1)</sub>


Ta có: (n2<sub> + 1) = 4k</sub>2<sub> + 4k + 2 = 2(2k</sub>2<sub> + 2k + 1) thì chia hết cho 2 (2)</sub>
Từ (1) và (2)  n6<sub> - n</sub>4<sub> – n</sub>2<sub> + 1 chia hết cho 64.2 = 128</sub>


b) Gọi q, r là thương và số dư của phép chia a cho b.
Ta có: a = p.q + r, 0 , 0 ≤ r ≤ b


Ta có thể viết: a= (b+1)q – q + r


Nếu q là thương của a chia cho b + 1 thì phải có điều kiện 0 ≤ r-q < b+1
 q ≤ r < q+b+1 là hiển nhiên.


Vậy chỉ cần điều kiện q ≤ r


</div>
<span class='text_page_counter'>(11)</span><div class='page_container' data-page=11>

Đặt A n320n 96 <sub>. Để ý rằng </sub>48 3.16 <sub> và </sub>

3;16

1<sub> nên để chứng minh A chia hết cho 48 ta </sub>


chứng minh A chia hết cho 3 và A chia hết cho 16.


Dễ thấy

 



3 3


n 20n n  n 21n  n 1 n n 1  21n


nên n320n<sub> chia hết cho 3. Do đó suy ra A </sub>



chia hết cho 3. Mặt khác do n là số chẵn, khi đó ta đặt n 2k <sub> với k là một số tự nhiên. Từ đó ta có </sub>


 

3

 



3 3 3


n 20n 2k 20.2k 8k 40k 8 k  k 48k 8 k 1 k k 1   48k


Suy ra n320n<sub> chia hết cho 16. Từ đó dẫn đến A chia hết cho 16.</sub>


Vậy A chia hết cho 48 với mọi n là số tự nhiên chẵn.


<b>Bài 25:</b><i> ( HSG TỈNH THANH HÓA NĂM HỌC 2013 – 2014).</i>


<b>a)</b> * Chứng minh xyz chia hết cho 5
+) Nếu xy chia hết cho 5 thì xyz chia hết cho 5


+) Nếu xy khơng chia hết cho 5 thì x2<sub> và y</sub>2<sub> chia 5 dư 1 hoặc dư 4</sub>


Khi đó z2<sub> = x</sub>2<sub> + y</sub>2<sub> chia 5 dư 0 hoặc 2 hoặc 3 nhưng vì z</sub>2<sub> không thể chia 5 dư 2 hoặc dư 3 => z</sub>2<sub> chia </sub>
hết cho 5 hay z chia hết cho 5 .


Vậy xyz chia hết cho 5


* Chứng minh xyz chia hết cho 3


- Nếu x hoặc y khơng chia hết cho 3 thì x2<sub> hoặc y</sub>2<sub> chỉ có thể chia 3 dư 1</sub>
khi đó z2 <sub> chia 3 dư 2 (Vơ lí)</sub>


Vậy xy chia hết cho 3 hay xyz chia hết cho 3


* Chứng minh xyz chia hết cho 4


+) Nếu x ,y chẵn thì xyz chia hết cho 4


+) Nếu trong hai số x hoặc y có một số lẻ, giả sử x chẵn, y lẻ suy ra z lẻ
Đặt x = 2k; y = 2n + 1, z = 2m + 1.Theo bài ra : (2m+1)2<sub> = 4k</sub>2 <sub>+ (2n+1)</sub>2
suy ra k2<sub> = m(m+1) - n(n+1) chia hết cho 2 => x chia hết cho 4</sub>


Vậy, xyz chia hết cho 4.


<i><b>Mà (3,4,5) =1 nên xyz chia hết cho 60.</b></i>


<i><b>b)</b></i> Gọi 3 số tự nhiên thoả mãn đề bài là x, y, z với x,y,z đều khác nhau và khác 0
Giả sử 1<sub>x< y <z khi đó ta có 0 < </sub>


1 1 1


<i>x</i><i>y</i><i>z</i><sub>< 3. ta cần tìm x, y, z để </sub>


1 1 1


<i>x</i><i>y</i><i>z</i><sub> có giá trị nguyên , </sub>
khi đó có 2 trường hợp sau:


TH1)


1 1 1
<i>x</i> <i>y</i><i>z</i> <sub>= 1 ,</sub>
Ta có 1=



1 1 1
<i>x</i> <i>y</i><i>z</i><sub>< </sub>


3


<i>x</i> <sub> suy ra 1</sub><sub>x < 3</sub>
- Xét x =1 (loại)


- Xét x =2 khi đó


1 1 1


2
<i>y</i><i>z</i>  <sub><</sub>


2


<i>y</i> <sub>=> 2 <y<4</sub>


=> y = 3 => z = 6 (thoả mãn ) Ta được 2 cặp số (2 ;3 ;6)
TH2)


1 1 1
<i>x</i><i>y</i><i>z</i><sub>=2</sub>
Ta có 2 =


1 1 1
<i>x</i><i>y</i><i>z</i><sub>< </sub>


3



</div>
<span class='text_page_counter'>(12)</span><div class='page_container' data-page=12>

b
a


D
B


c
=> x =1 =>


1 1
1
<i>y</i><i>z</i>  <sub>< </sub>


2


<i>y</i> <sub> suy ra 1</sub><sub></sub><sub>x <y <2 (loại) </sub>


<i><b>Vậy từ các TH trên ta được 3 số thoả mãn đề bài là 2 ; 3 và 6</b></i>.


<b>c)</b> Gọi độ dài ba cạnh của tam giác vuông ABC là: a, b, c. (a, b, c N*<sub>)</sub>
Ta có: a, b P và b – a = 50: là số chẵn nên a, b đều lẻ (b > a).


Giả sử cạnh thứ ba c là cạnh huyền.
Theo định lý Pi-ta-go, ta có:


c2<sub> = a</sub>2<sub> + b</sub>2<sub>  c</sub>2<sub> = a</sub>2<sub> + (a + 50)</sub>2<sub> = 2a</sub>2<sub> + 100a + 2500 = 2(a</sub>2<sub> + 50a + 1250): số chẵn.</sub>


Vì a lẻ nên (a2<sub> + 50a + 1250): lẻ do đó 2(a</sub>2<sub> + 50a + 1250) 4: điều này vơ lý vì c</sub>2<sub> là số chính </sub>
phương chẵn phải chia hết cho 4.



Do đó cạnh thứ ba c không thể là cạnh huyền.
Suy ra b là cạnh huyền (vì b > a).


Theo định lý Pi-ta-go ta có:


b2<sub> = a</sub>2<sub> + c</sub>2<sub>  c</sub>2<sub> = b</sub>2<sub> – a</sub>2<sub> = (b – a)(b + a) = 50(b + a) = 5</sub>2<sub>.2.( a + b) </sub>
Vì c2<sub> là số chính phương nên:</sub>


Suy ra: a + b = 2k2<sub> (k </sub><sub>N</sub>*<sub>), vì b > 50 nên a + b > 50, do đó k  6.</sub>
Khi đó: (a + b)min = 2.62<sub> = 72, ta có:</sub>


a b 72 a 11 P


b a 50 b 61 P


   


 




 


   


  <sub>: thỏa điều kiện</sub>


Từ đó: cmin = 5.2.k = 5.2.6 = 60 khi a = 11, b = 61.



<i><b>Vậy giá trị nhỏ nhất của cạnh thứ ba của tam giác vuông là 60.</b></i>


<b>Bài 26:</b><i> ( HSG TỈNH ĐÀ NẴNG NĂM HỌC 2010 – 2011).</i>


Đặt: S = 1<b>2</b>34<b>5</b>6789<b>10</b>1112<b> </b>


 100
S




3467891112 (1) là một số nguyên
 hai chữ số tận cùng của S là 00


Mặt khác, trong suốt quá trình nhân liên tiếp các thừa số ở vế phải của (1), nếu chỉ để ý đến chữ số
tận cùng, ta thấy 100


S


có chữ số tận cùng là 6 (vì 34=12; 26=12; 27=14; 48=32; 29=18;
811=88; 812=96)


Vậy ba chữ số tận cùng của S là 600


<b>Bài 27:</b><i> ( HSG TỈNH ĐAKLAK NĂM HỌC 2010 – 2011).</i>


27.1a)



3



3 3 3 <sub>3</sub> <sub>3</sub> 3 <sub>3</sub>


<i>Q a</i> <i>b</i> <i>c</i>  <i>abc</i> <i>a b</i>  <i>ab a b</i> <i>c</i>  <i>abc</i>


 



3


3 3


<i>a b c</i> <i>ab a b c</i> <i>a b c a b c</i>


          <sub> </sub>


 



2


3 3 3


<i>a b c</i>  <i>a b c</i> <i>ab</i> <i>ac</i> <i>bc</i>


       


  <sub> </sub>




2 2 2



<i>a b c a</i> <i>b</i> <i>c</i> <i>ab ac bc</i>


       




 

2

2

2 3 3 3


2<i>Q</i> <i>a b c</i>  <i>a b</i> <i>b c</i> <i>c a</i>  0 <i>a</i> <i>b</i> <i>c</i> 3<i>abc</i>


             


 


b) Vẽ tam giác vuông ABC đỉnh A, và đường cao AD có kích thước như hình vẽ có
AB = 2010, AC = 2011 và <i>P ab bc</i>  2<i>SABC</i>


Khi đó, <i>P</i>2010.2011 4042110 <sub> </sub>


<i><b>Hãy ln chiến thắng chính mình. 12</b></i>


B
C


A
c


</div>
<span class='text_page_counter'>(13)</span><div class='page_container' data-page=13>





27.2. Ta có:

 

 

 



2 2


12 1 2 1 1 23 12 2 1 23


<i>n</i>


<i>x</i>  <i>n</i> <i>n</i> <i>n n</i>    <i>n</i>  <i>n</i> <i>n</i> <i>n</i>  




2

2 2  2  2  2


12 <i>n</i> <i>n</i> 1 23 12<i>n</i> 12<i>n</i> 11 2<i>n</i> 1 2<i>n</i> 1 2<i>n</i> 3


            



Vậy có điều phải chứng minh.


<b>Bài 28:</b><i> ( HSG TỈNH ĐAKLAK NĂM HỌC 2012 – 2013).</i>


Do <i>p</i> là số nguyên tố lớn hơn 3 nên <i>p</i> có dạng

<i>p</i>

3

<i>k</i>

1


*) Nếu <i>p</i>3<i>k</i>1 thì 2<i>p</i> 1 6<i>k</i> 3 3(2<i>k</i>1)


2<i>p</i> 1


  <sub> là hợp số (Vô lý)</sub>



*) Nếu <i>p</i>3<i>k</i> 1,<i>k</i>2 thì 4<i>p</i> 1 12<i>k</i> 33(4<i>k</i> 1)
Do 4<i>k</i> 17<sub> nên </sub>4<i>p</i>1<sub> là một hợp số.</sub>


<b>Bài 29:</b><i> ( HSG TỈNH ĐỒNG NAI NĂM HỌC 2018 – 2019).</i>


<b>a) Cách 1:</b>


1

1 1



(

) (1)



<i>bc a b c</i>



<i>a</i>

 

<i>b c</i>



TH1: Nếu a là số nguyên chẵn, suy ra

<i>a b c</i>

(

) 2

, theo (1)Suy ra:

b.c 2


Vậy abc chia hết cho 4


TH2: Nếu a là số nguyên lẻ. Với b và c là hai số cũng lẻ thì:

<i>b c</i>

2

<i>a b c</i>

(

) 2


<i>a b c</i>

. .

khơng chia hết cho 2 (vì a, b, c đều lẻ). Suy ra mâu thuẫn.


Vậy trong hai số, b, c tồn tại ít nhất 1 số chẵn.


+ Với b chẵn, mà a lẻ nên c chẵn (vì b.c chẵn nên a(b+c) chẵn suy ra c chẵn, vì a lẻ)
Suy ra abc chia hết cho 4


+ Với c chẵn, tương tự abc chia hết cho 4


<b>Cách 2:</b>



2


1

1 1



(

)

abc=a (b+c) (2)



<i>bc a b c</i>



<i>a</i>

 

<i>b c</i>



Ta thấy a, b, c khơng thể đều là số lẻ vì nếu vây thì abc là số lẻ, còn b+c là số chẵn.
Vậy trong 3 số tồn tại ít nhất 1 số chẵn.


Nếu a chẵn thì a2 <sub>chia hết cho 4, từ (2) suy ra abc chia hết cho 2.</sub>


Nếu b chẵn, do a lẻ nên b + c chẵn (vì abc chẵn) suy ra c chẵn. Vậy abc chia hết cho 2.
Tương tự cho trường hợp c chẵn.


<b>b) Cách 1: Dùng hàm Ơle:</b>


<b>Phân tích số m ra thừa số nguyên tố: </b> 1 . 2 . 3 ...


<i>x</i> <i>y</i> <i>z</i>
<i>m</i><i>p p p</i>


<b>Số các số nguyên dương không vượt quá m và nguyên tố cùng nhau với m là</b>


1 2 3


1

1

1




( )

<i>m</i>

<i>m</i>

. 1

. 1

. 1

....



<i>p</i>

<i>p</i>

<i>p</i>



<sub></sub>

 

<sub> </sub>

<sub></sub>

<sub></sub>

<sub></sub>



 

 



<b>Ta có: </b>


3

1

1



999 3 .37

(999) 999. 1

. 1

648



3

37



 



<sub></sub>

<sub> </sub>

<sub></sub>



 



Có 648 số nguyên tố cùng nhau với 999 và khơng vượt q 999.
Vây có 649 số nguyên tố cùng nhau với 999 và không vượt quá 1000.


</div>
<span class='text_page_counter'>(14)</span><div class='page_container' data-page=14>

Gọi A là số các số nguyên dương không vượt quá 1000. Suy ra A = 1000


B là số các số nguyên dương không vượt quá 1000 mà không nguyên tố cùng nhau với 999.
C là số các số nguyên dương không vượt quá 1000 nguyên tố cùng nhau với 999



Ta có:

999 3 .37

3


B = (Số các số nguyên dương không vượt quá 1000 và chia hết cho 3) – (Số các số nguyên dương
không vượt quá 1000 và chia hết cho 37 mà không chia hết cho 3)


+ Số các số nguyên dương không vượt quá 1000 và chia hết cho 3 là:


999 3



1 333


3





 



+ Số các số nguyên dương không vượt quá 1000 và chia hết cho 37 là:


999 37



1 27


37





 



+ Số các số nguyên dương không vượt quá 1000 và chia hết cho cả 37 và 3 (chia hết cho 111) là:



999 111



1 9


111





 



+ Số các số nguyên dương không vượt quá 1000 và chia hết cho 37 mà không chia hết cho 3 là:


27 9 18



Suy ra B = 333+ 18 = 351. Vậy C= A – B = 1000 – 351 = 649


<b>Bài 30:</b><i> ( HSG TỈNH HÀ GIANG NĂM HỌC 2011 – 2012).</i>


P = 1.2.3...2009.2010.


1 1 1 1 1


1 ...


2 3 4 2009 2010


 


     


 



 


= 1.2.3...2009.2010 + 1.1.3...2009.2010 + 1.2.1.4...2009.2010 +...+1.2.3...2009


= 1.2.3...2009(2010+1) + 1.1.3... 2010 (2009+2) + ... + 1.2.3....1004.1007...2010(1006+1005)
= 2011(1.2.3...2009+ 1.1.3... 2010+ ... +1.2.3....1004.1007...2010) <sub>2011 .</sub>


<b>Bài 31:</b><i> ( HSG TỈNH HÀ TỈNH NĂM HỌC 2008 – 2009).</i>


Từ


1


<i>a</i>+


1


<i>b</i>+


1


<i>c</i>=1 <sub> => ab + ac + bc = abc => (a + b + c)(ab + ac + bc) = abc </sub>


=> (a + b)(b + c)(c + a) = 0=> a + b = 0 hoặc b + c= 0 hoặc c + a = 0.
Nếu: a + b = 0 => c = 1 => a2009<sub> + b</sub>2009<sub> = 0 => a</sub>2009<sub> + b</sub>2009<sub> + c</sub>2009 <sub>= 1 </sub>
Tương tự với: b + c= 0 hoặc c + a = 0.


<b>Bài 32:</b><i> ( HSG TỈNH HÀ TỈNH NĂM HỌC 2010 – 2011).</i>



Từ giả thiết


2


2 2 2


1 1 1 1 1 1 1 1 1


( ) 2( ) 0 a b c 0


ab c a b c  ab bc ca      


3 3 3 3 3 3


a b c a b 3ab(a b) c a b c 3abc


            <sub> </sub><b><sub>(*)</sub></b>


Từ <b>(*) </b>dễ thấy khi a, b, c Z<sub> thì </sub>(a3b3c ) 33  <sub>, đpcm.</sub>


<b>Bài 33:</b><i> ( HSG TỈNH HÀ TỈNH NĂM HỌC 2012 – 2013).</i>


<b>a)</b> Điều kiện có nghĩa là<b> </b><i>a, b, c</i> ¿0 <b><sub>.</sub></b>


Ta có P = <i>abc</i>−<i>a</i>−<i>b</i>−<i>c</i>+


1


<i>a</i>+



1


<i>b</i>+


1


<i>c</i>−


1


<i>abc</i> <sub>, nên P nguyên </sub> ⇔ <sub>S = </sub>


1


<i>a</i>+


1


<i>b</i>+


1


<i>c</i>−


1


<i>abc</i> <sub> ngun.</sub>


Khơng mất tính tổng qt, giả sử 1 ¿<i>a</i><<i>b</i><<i>c</i>⇒



1


<i>a</i>+


1


<i>b</i>+


1


<i>c</i>−


1


</div>
<span class='text_page_counter'>(15)</span><div class='page_container' data-page=15>

Hơn nữa ta có
1
<i>a</i>+
1
<i>b</i>+
1
<i>c</i>>
1
<i>a</i>>
1


<i>abc</i> ⇒ <sub> S > 0. Do đó S = 1 hoặc S = 2.</sub>


+) S = 1. Ta có 1 =


1


<i>a</i>+
1
<i>b</i>+
1
<i>c</i>−
1
<i>abc</i> <sub>< </sub>
1
<i>a</i>+
1
<i>b</i>+
1


<i>c</i> <sub> < </sub>


3


<i>a</i> ⇒


3


<i>a</i> <sub>>1 </sub> ⇒ <sub> a =1 hoặc a = 2.</sub>


Với a = 1 ⇒ <sub> 1 = 1 +</sub>


1
<i>b</i> <sub>+</sub>
1
<i>c</i> <sub></sub>
-1


<i>bc</i> ⇒
1
<i>b</i> <sub>+</sub>
1
<i>c</i> <sub></sub>
-1


<i>bc</i> <sub> = 0 không xảy ra</sub>


Với a = 2 ⇒ <sub> 1 = </sub>


1
2 <sub>+</sub>
1
<i>b</i> <sub>+</sub>
1
<i>c</i> <sub></sub>
-1
2<i>bc</i>⇔


1


<i>b</i> <sub>+</sub>


1


<i>c</i> <sub></sub>


-1
2<i>bc</i> <sub> = </sub>



1


2 <sub>. Suy ra </sub>
2


<i>b</i> <sub>> </sub>


1
2 ⇒


b < 4.


Từ đó 2<<i>b</i><4 ⇒ <sub> b = 3. Thay vào được c = 5. Vậy a = 2, b = 3 , c = 5.</sub>


+) S = 2. Ta có 2 =


1
<i>a</i>+
1
<i>b</i>+
1
<i>c</i>−
1
<i>abc</i> <sub>< </sub>
1
<i>a</i>+
1
<i>b</i>+
1



<i>c</i> <sub> < </sub>


3


<i>a</i> ⇒ <i>a</i><


3


2 ⇒ <sub> a =1. </sub>


Thay vào được


1


<i>b</i> <sub>+</sub>


1


<i>c</i> <sub></sub>


-1


<i>bc</i> <sub> = 1 </sub> ⇒


2


<i>b</i> <sub>>1 </sub> ⇒ <i><sub>b</sub></i><sub> =1 loại vì khơng thỏa mãn </sub><i><sub>b </sub></i><sub>></sub><i><sub> a</sub></i><sub>.</sub>


Kết hợp các trường hợp và do vai trị bình đẳng nên các số (<i>a, b, c</i>) cần tìm là:


(2,3,5), (2,5,3), (3,5,2), (3,2,5), (5,3,2), (5,2,3).


<b>b)</b> Đặt


<i>b</i>+<i>c</i>=<i>x</i>


<i>c</i>+<i>a</i>=<i>y</i>
<i>a</i>+<i>b</i>=<i>z</i>


¿
{¿ {¿ ¿ ¿


¿ thì <i>x, y, z</i> dương và


<i>a</i>=<i>y</i>+<i>z</i>−<i>x</i>
2 <i>,b</i>=


<i>x</i>+<i>z</i>−<i>y</i>
2 <i>, c</i>=


<i>x</i>+<i>y</i>−<i>z</i>
2 .


Ta có:

<i>a</i>



1

<i>a</i>

+



<i>b</i>




1

<i>b</i>

+



<i>c</i>



1

<i>c</i>

=



<i>a</i>


<i>b</i>

+

<i>c</i>

+



<i>b</i>


<i>c</i>

+

<i>a</i>

+



<i>c</i>


<i>a</i>

+

<i>b</i>

=



<i>y</i>

+

<i>z</i>

<i>x</i>



2

<i>x</i>

+



<i>x</i>

+

<i>z</i>

<i>y</i>



2

<i>y</i>

+



<i>x</i>

+

<i>y</i>

<i>z</i>



2

<i>z</i>


=

1


2

(


<i>y</i>


<i>z</i>

+



<i>x</i>


<i>y</i>

)

+



1


2

(



<i>z</i>


<i>x</i>

+



<i>x</i>


<i>z</i>

)

+



1


2

(



<i>z</i>


<i>y</i>

+



<i>y</i>


<i>z</i>

)



3



2

1

+

1

+

1


3



2

=



3




2

<sub>. </sub>


Dấu “=” xảy ra ⇔<i>x</i>=<i>y</i>=<i>z</i>


Với x = y = z thì a = b = c hay tam giác ABC đều.


<b>Bài 34:</b><i> ( HSG TỈNH HÀ TỈNH NĂM HỌC 2016 – 2017).</i>


Do
<i>ab</i>


<i>a b</i> <sub> là số hữu tỉ và </sub><i>a b</i> <sub> là số nguyên dương nên từ </sub>


<i>ab</i>


<i>a b</i> <i>a b</i>


<i>a b</i>


   


 <sub>là số chính phương.</sub>
Do <i>a b</i> 18 <i>a b</i> 1;4;9;16


Thử lại các trường hợp ta có <i>a</i>2;<i>b</i>7 suy ra số cần tìm là 27.


<b>Bài 35:</b><i> ( HSG TỈNH HẢI DƯƠNG NĂM HỌC 2009 – 2010).</i>


+Xét n > 9  <i>A</i>292132<i>n</i>2 (1 29  42<i>n</i>9)



Thấy 1 2 42<i>n</i>9<sub> là số lẻ nên A chia hết cho 2</sub>9<sub> nhưng không chia hết cho 2</sub>10<sub> nên A khơng là số </sub>
chính phương.


+Xét n = 9  <i>A</i>29213292 (1 29  41) 9.2 10 962 là số chính phương.
+Xét n < 9  <i>A</i>292132<i>n</i> 2 (2<i>n</i> 9<i>n</i>213<i>n</i>1)


Do 29<i>n</i>213<i>n</i>1<sub> là số lẻ và A là số chính phương nên 2</sub><i>n</i><sub> là số chính phương nên n là số chẵn,</sub>
*


</div>
<span class='text_page_counter'>(16)</span><div class='page_container' data-page=16>

Khi đó A chính phương, 2<i>n</i> chính phương suy ra


9 13


2 <i>n</i> 2 <i>n</i> 1


<i>B</i>     <sub> là số chính phương.</sub>


Nhận xét số chính phương lẻ chỉ có thể tận cùng là 1; 5; 9.
Với n = 2  <i>B</i>27211 1 2177<sub> (loại)</sub>


Với n = 4 59221545<i>B</i><sub>, thấy B chia hết cho 5 nhưng không chia hết cho 25 nên B khơng là số chính phương.</sub>
Với n = 6 37221137<i>B</i><sub> (loại)</sub>


Với n = 8 <i>B</i> 2 25 1 35<sub> (loại). Vậy n = 9.</sub>


<b>Bài 36:</b><i> ( HSG TỈNH HẢI DƯƠNG NĂM HỌC 2012 – 2013).</i>


 














2 2 2 2 2 2


2


2 2


4a

3ab 11b 5

5a

5ab 10b

4a

3ab 11b

5


a

2ab

b 5

a

b

5



 



a

b 5

<sub> ( Vì 5 là số nguyên tố) </sub>


 



4 4 2 2


a

b

a

b

a

b a

b 5








<b>Bài 37:</b><i> ( HSG TỈNH HẢI DƯƠNG NĂM HỌC 2013 – 2014).</i>


a) <i>a</i>3 <i>a b ab</i>2  2 6<i>b</i>3  0 (<i>a</i> 2 )(<i>b a</i>2<i>ab</i>3 ) 0 (*)<i>b</i>2 
Vì a > b > 0  <i>a</i>2<i>ab</i>3<i>b</i>2 0 nên từ (*) ta có a = 2 b
Vậy biểu thức


4 4 4 4


4 4 4 4


4 16 4


4 64


<i>a</i> <i>b</i> <i>b</i> <i>b</i>


<i>B</i>


<i>b</i> <i>a</i> <i>b</i> <i>b</i>


 


 


 


4
4



12 4


63 21


<i>b</i>
<i>b</i>




 



b) 2<i>a</i>2<i>a</i>3<i>b</i>2<i>b</i>


2
(<i>a b</i>)(2<i>a</i> 2<i>b</i> 1) <i>b</i>


     <sub> (*)</sub>


Gọi d là ước chung của (a - b, 2a + 2b + 1) (<i>d</i> *<sub>). Thì</sub>


 

2
2 2


( )


2 2 1


(2 2 1)



<i>a b d</i>


<i>a b</i> <i>a</i> <i>b</i> <i>d</i>


<i>a</i> <i>b</i> <i>d</i>


<i>b d</i> <i>b d</i>





   




 




 







 


Mà (<i>a b d</i> )  <i>a d</i>  (2<i>a</i>2 )<i>b d</i> mà (2<i>a</i>2<i>b</i>1)<i>d</i> 1<i>d</i> <i>d</i> 1



Do đó (a - b, 2a + 2b + 1) = 1. Từ (*) ta được <i>a b</i> <sub> và 2</sub><i>a</i>2<i>b</i>1<sub> là số chính phương </sub>
=> 2<i>a</i>2<i>b</i>1<sub> là số chính phương.</sub>


<b>Bài 38:</b><i> ( HSG TỈNH HẢI DƯƠNG NĂM HỌC 2014 – 2015).</i>


a) Tìm số nguyên tố <i>p</i> sao cho các số 2<i>p</i>21; 2<i>p</i>2 3; 3<i>p</i>24 đều là số nguyên tố.
+) Nếu p=7k+i; k,i nguyên, i thuộc tập

  1; 2; 3

. Khi đó <i>p</i>2chia cho 7 có thể dư: 1;4;2
Xét <i>p</i> 2 2<i>p</i>21; 2<i>p</i>23 & 3<i>p</i>2 4 7


</div>
<span class='text_page_counter'>(17)</span><div class='page_container' data-page=17>

+) Xét p = 7k, vì p nguyên tố nên p = 7 là nguyên tố, có:


2 2 2


2<i>p</i>  1 97; 2 <i>p</i>  3 101; 3<i>p</i>  4 151<sub>đều là các số nguyên tố</sub>


Vậy p =7


b) Giả thiết



2 2 2 2 2


3 <i>x</i> 3 18<i>y</i> 2<i>z</i> 3<i>y z</i> 54


      <sub>(1)</sub>


+) Lập luận để <i>z</i>23 <i>z</i>3 <i>z</i>29 <i>z</i>2 9<sub>(*)</sub>
(1) 3(<i>x</i> 3)22<i>z</i>23 (<i>y z</i>2 2 6) 54(2)


(2) 54 3( <i>x</i> 3)22<i>z</i>23 (<i>y z</i>2 2 6) 3( <i>x</i> 3)22.9 3 .3 <i>y</i>2



2 2


(<i>x</i> 3) 3<i>y</i> 12


2 2 2


4 1; 4


<i>y</i> <i>y</i> <i>y</i>


     <sub> vì y nguyên dương</sub>


Nếu <i>y</i>2  1 <i>y</i>1 thì (1) có dạng:



2 <sub>2</sub> <sub>2</sub> <sub>2</sub> 72 <sub>2</sub>


3 3 5 72 5 72 9 3


5


<i>x</i>  <i>z</i>   <i>z</i>   <i>z</i>   <i>z</i>   <i>z</i>


(vì có(*))


Khi đó



2 2


3 <i>x</i> 3 27 <i>x</i> 3 9<sub>, x nguyên dương nên tìm được x=6</sub>



Nếu <i>y</i>2  4 <i>y</i>2(vì y ngun dương) thì (1) có dạng:


2 2 2 2 2


3 <i>x</i> 3 14<i>z</i> 126 14 <i>z</i> 126 <i>z</i>  9 <i>z</i>  9 <i>z</i>3<sub>(vì z nguyên dương)</sub>
Suy ra (<i>x</i> 3)2  0 <i>x</i>3(vì x nguyên dương)


Đáp số


3 6


2; 1


3 3


<i>x</i> <i>x</i>


<i>y</i> <i>y</i>


<i>z</i> <i>z</i>


 


 


 


 



 


 <sub></sub>  <sub></sub>


 


<b>Bài 39:</b><i> ( HSG TỈNH HẢI DƯƠNG NĂM HỌC 2016 – 2017)</i>


<i>Tìm các cặp số nguyên</i>(x; y)<i>thoả mãn: </i>2x22y2 3x 6y 5xy 7   <i>. </i><b>(1)</b>


Ta có (1) 2x22y23x 6y 5xy  7






 



2 2


2x 4xy 2y xy 3x 6y 7
2x x 2y y 2y x 3 x 2y 7


x 2y 2x y 3 7


      


      


    



Vì x, y  suy ra

x 2y ; 2x y 3

 

 

Z


Ta có -7=(-1).7=1.(-7) nên ta có các trường hợp sau:


<b>+ TH1:</b>


x 2y 1 x 3


2x y 3 7 y 2


  


 




 


   


  <sub> (Thoả mãn)</sub>


<b>+ TH2:</b>


x 2y 7 x 5


2x y 3 1 y 6


  



 




 


   


  <sub> (Thoả mãn)</sub>


<b>+ TH3:</b>


x 2y 1 x 7


2x y 3 7 y 4


  


 




 


   


  <sub> (Thoả mãn)</sub>


<b>+ TH4:</b>



x 2y 7 x 1


2x y 3 1 y 4


  


 




 


   


  <sub> (Thoả mãn)</sub>


</div>
<span class='text_page_counter'>(18)</span><div class='page_container' data-page=18>

3 ; 2 ,

 

5 ; 6 ,

 

7 ; 4 , 1 ;4 .

 



<i>Tìm tất cả các số tự nhiên </i>n<i> sao cho </i>n22n n22n 18 9  <i> là số chính phương.</i>
Ta có n22n n2 2n 18 9  là số chính phương


<i>Mà </i>n2 2n 9 N, n N    suy ra n22n 18 là số tự nhiên
Đặt n2 2n 18 k  (k N <sub>)</sub>


 <sub> n</sub>2<sub> + 2n + 18 = k</sub>2<sub></sub> <sub> n</sub>2<sub> + 2n + 1 +17 = k</sub>2 <sub></sub> <sub>( n + 1 )</sub>2<sub> + 17 = k</sub>2
 <sub> k</sub>2<sub> - ( n + 1 )</sub>2<sub> = 17</sub><sub></sub> <sub>( k + n + 1 )(k - n - 1) = 17</sub>


Vì k, n đều là số tự nhiên nên k+n+ 1 > k - n-1, đồng thời k > n nên:
( k + n + 1 )(k - n - 1) = 17.1





1 17 16 9


1 1 2 7


   <sub></sub>   <sub></sub> 


     


<i>k n</i> <i>k n</i> <i>k</i>


<i>k n</i> <i>k n</i> <i>n</i>


Từ đó ta có n2 2n n2 2n 18 9 81 9    2 (Thoả mãn). Vậy n=7


<b>Bài 40:</b><i> ( HSG TỈNH HẢI PHÒNG NĂM HỌC 2016 – 2017)</i>


Ta có (a + b2<sub>)  (a</sub>2<sub>b – 1) suy ra: a + b</sub>2<sub> = k(a</sub>2<sub>b – 1), với k  </sub>*
 a + k = b(ka2<sub> – b) hay mb = a + k (1) với </sub>m ka – b<sub></sub> 2 <sub> </sub>*
 m + b = ka2<sub> (2)</sub>


Từ (1) và (2) suy ra: mb m b 1 a k ka      21
 (m – 1)(b – 1) = (a + 1)(k + 1 – ka) (3)


Do

 



*



m, b  m –1 b –1 0


Vì thế từ (3) suy ra: (a + 1)(k + 1 – ka)  0.


Lại do a > 0 nên suy ra: k + 1 – ka  0  1  k(a – 1)
Vì a – 1  0, k > 0 nên 1 k a –1 0

 vµ k a –1

 


a 1
k(a 1) 0


a 2
k(a 1) 1


k 1




 


 <sub></sub>


 <sub></sub>  <sub></sub> 


  <sub></sub>


 <sub></sub>







Với a = 1. Thay vào (3) ta được: (m – 1)(b – 1) = 2.




2
2


m 1 2


b 1 1 <sub>b 2</sub> <sub>k.a</sub> <sub>5</sub> <sub>a 1</sub>


b 3 k.a 5 a 1


m 1 1
b 1 2


  




  <sub>  </sub> <sub> </sub> <sub></sub>




 <sub> </sub>


<sub></sub> <sub></sub> <sub></sub> <sub> </sub> <sub> </sub> <sub></sub>



 


 
 




Vậy, trường hợp này ta được hai cặp a = 1; b = 2 và a = 1; b = 3.
Với a = 2 và k = 1. Thay vào (3) ta có: (m – 1)(b – 1) = 0 


b 1
m 1




 <sub></sub>


 <sub>.</sub>


Khi b = 1, ta được: a = 2, b = 1.


Khi m = 1: từ (1) suy ra a + k = b  b = 3.
Khi đó: a = 2, b = 3.


Vậy có 4 cặp số (a; b) thỏa mãn là: (1; 2), (1; 3), (2; 3), (2; 1).


<b>Bài 41:</b><i> ( HSG TỈNH HẬU GIANG NĂM HỌC 2012 – 2013).</i>


</div>
<span class='text_page_counter'>(19)</span><div class='page_container' data-page=19>

y, z là các số ngun dương có vai trị như nhau nên ta có các hệ:



{

<i>y</i>

1

=

1



<i>z</i>

1

=

6

{



<i>y</i>

=

2



<i>z</i>

=

7

<sub> và </sub>

{



<i>y</i>

1

=

2



<i>z</i>

1

=

3

{



<i>y</i>

=

3



<i>z</i>

=

4


Kết luận: Ba số nguyên tố cần tìm là 2, 5, 7


<b>Bài 42:</b><i> ( HSG TỈNH HẬU GIANG NĂM HỌC 2014 – 2015).</i>


a) So sánh hai số sau:


Theo tính chất của phân thức ta có:


2 <sub>2</sub> <sub>2</sub>


2
2015 2014 2015 2014 2015 2014 2015 2014


.



2015 2014 2015 2014 2015 2014 (2015 2014)


   


 


 


 


   


 


=


2 2


2 2


2015 2014


2015 2.2015.2014 2014


  <sub>< </sub>


2 2



2 2


2015 2014
2015 2014





<b>Vậy A < B.</b>


b) Chứng minh tổng M = 1 + 2 + 22<sub> + … + 2</sub>2015 <sub> chia hết cho 15 .</sub>
M = 1 + 2 + 22<sub> + … + 2</sub>2015


= (1 + 2 + 22<sub> + 2</sub>3<sub> ) + (2</sub>4<sub> + 2</sub>5<sub> + 2</sub>6<sub> + 2</sub>7<sub> ) + …+ ( 2</sub>2012<sub> + 2</sub>2013 <sub>+2</sub>2014<sub> + 2</sub>2015<sub>)</sub>
= (1 + 2 + 22<sub> + 2</sub>3<sub> )+ 2</sub>4<sub> (1 + 2 + 2</sub>2<sub> + 2</sub>3<sub> )+ …+2</sub>2012<sub>(1 + 2 + 2</sub>2<sub> + 2</sub>3<sub> )</sub>
= 15 ( 1 + 24<sub> + …+ 2</sub>2012 <sub>) chia hết cho 15.</sub>


c) Chứng minh rằng tổng S = 13<sub> + 2</sub>3<sub> + 3</sub>3<sub> + . . . + 2015</sub>3<sub> là một số chính phương</sub>
Gọi Ak = 1 + 2 + 3 + . . . + k =


( 1)
2
<i>k k</i>


Ak-1 = 1 + 2 + 3 + . . . + (k – 1) =


( 1)
2
<i>k k</i>
 <i>Ak</i>2 <i>Ak</i>21<i>k</i>3



Khi đó: 13 <i>A</i>12
23 <i>A</i>22 <i>A</i>12
. . .


<i>k</i>3 <i>Ak</i>2 <i>Ak</i>21


Cộng vế theo vế các đẳng thức trên ta có:
13<sub> + 2</sub>3<sub> + 3</sub>3<sub> + . . . + k</sub>3<sub> = </sub>


2
2 ( 1)


2
<i>k</i>


<i>k k</i>
<i>A</i> <sub></sub>  <sub></sub>


 


13<sub> + 2</sub>3<sub> + 3</sub>3<sub> + . . . + 2015</sub>3


2
2015(2015 1)


2


 



 


  <sub>= (2015.1008)</sub>2


Là số chính phương.


<b>Bài 43:</b><i> ( HSG TỈNH HỊA BÌNH NĂM HỌC 2009 – 2010).</i>


Thư trùc tiÕp có số
cần tìm là 7744.


Gọi số cần tìm là A = aabb = 1100.a +11.b =11(100.a + b)
Do sè cần tìm là số chính phơng và 11 là số nguyªn tè nªn


</div>
<span class='text_page_counter'>(20)</span><div class='page_container' data-page=20>

<b>Bài 44:</b><i> ( HSG TỈNH HỊA BÌNH NĂM HỌC 2013 – 2014).</i>
a) Tính giá trị của biểu thức


( 5)( 1)
( 5)


<i>x</i> <i>y</i>


<i>A</i>


<i>x x</i>


 





 <sub> biết </sub>2<i>x</i>29<i>y</i>2 6<i>xy</i> 6<i>x</i> 9 0.


Giả thiết 


2 2 3


( 3 ) ( 3) 0


1


<i>x</i>


<i>x</i> <i>y</i> <i>x</i>


<i>y</i>





     





Thay vào


8
3
<i>A</i>



b) Chứng minh rằng với mọi số nguyên <i>n</i> thì <i>n</i>311<i>n</i><sub> chia hết cho 6.</sub>
Phân tích:


3 <sub>11</sub> <sub>(</sub> 3 <sub>) 12</sub> <sub>(</sub> <sub>1) (</sub> <sub>1) 12</sub>


<i>n</i>  <i>n</i> <i>n</i>  <i>n</i>  <i>n</i> <i>n</i> <i>n n</i>  <i>n</i>


Lập luận được


( 1) ( 1) 6
12 6


<i>n</i> <i>n n</i>
<i>n</i>


  


   <i>n</i> <sub> dẫn đến điều phải c/m.</sub>
<i>c)</i> Tìm các cặp số nguyên ( ;<i>x y</i>) sao cho <i>x x</i>( 1)<i>y</i>21
Giả thiết  (2<i>x</i>2<i>y</i>1)(2<i>x</i> 2<i>y</i>1) 5


Vì ,<i>x y</i>  nên (2<i>x</i>2<i>y</i>1);(2<i>x</i> 2<i>y</i>1)là ước của 5
Xét 4 trường hợp:


2 2 1 5 2 2 1 5 2 2 1 1 2 2 1 1


; ; ;


2 2 1 1 2 2 1 1 2 2 1 5 2 2 1 5



<i>x</i> <i>y</i> <i>x</i> <i>y</i> <i>x</i> <i>y</i> <i>x</i> <i>y</i>


<i>x</i> <i>y</i> <i>x</i> <i>y</i> <i>x</i> <i>y</i> <i>x</i> <i>y</i>


           


   


   


           


   


Giải hệ tìm được các cặp ( ; ) (1; 1);( 2;1);(1;1);( 2; 1)<i>x y</i>     
d)


Giả sử 3<i>n</i>  4 <i>a</i>2<sub>, với a nguyên dương (</sub><i>a</i>2<sub>), ta có: </sub>3<i>n</i> (<i>a</i> 2)(<i>a</i>2)<sub> (*)</sub>


Vì (<i>a</i>2) ( <i>a</i> 2) 4 nên (<i>a</i> 2) và (<i>a</i>2) không cùng chia hết cho 3 nên với <i>n</i>2<sub>thì khơng có </sub>
a thỏa mãn (*)


Với <i>n</i>1<sub> thì </sub>31 4 7<sub> khơng là số chính phương. Vậy khơng có số ngun dương n nào để 3</sub><i>n</i>4
là số chính phương.


<b>Bài 45:</b><i> ( HSG TỈNH NGHỆ AN NĂM HỌC 2008 – 2009).</i>


a) Có <i>a b c</i>   <i>a</i> <i>b</i> <i>c</i>



2 ( ) 2


( ) ( )( ) 0


<i>a b c</i> <i>b</i> <i>a</i> <i>c</i> <i>a b c</i> <i>b a b c</i> <i>b a c</i> <i>ac</i>


<i>a b</i>
<i>b a b c</i> <i>ac</i> <i>a b b c</i>


<i>b c</i>


               





       <sub>  </sub>





Nếu a = b và a , c dương .Ta có


1 1 1 2 1


1 1 2<i>c a ac</i> (<i>a</i> 2)(<i>c</i> 1) 2


<i>a b c</i>    <i>a c</i>        
Vì a,b,c ngun dương nên ta có các trường hợp sau :



2 2 4 2 1


1) 2) 3


1 1 2 1 2


<i>a</i> <i>a</i> <i>b</i> <i>a</i>


<i>a c</i> <i>b</i>


<i>c</i> <i>c</i> <i>c</i>


     


  


    


  


    


  


Nếu b = c và b,c dương .Ta có


1 1 1 1 2


1 1 2<i>a b ab</i> (<i>b</i> 2)(<i>a</i> 1) 2



<i>a b c</i>    <i>a b</i>        
Vì a,b,c nguyên dương nên ta có các trường hợp sau :


2 2 2 2 4


1) 3 2)


1 1 1 1 2


<i>b</i> <i>b</i> <i>b</i> <i>c</i>


<i>a b</i> <i>c</i>


<i>a</i> <i>a</i> <i>a</i>


     


  


    


  


    


  


</div>
<span class='text_page_counter'>(21)</span><div class='page_container' data-page=21>

b) Vì <i>f x</i>( )<i>ax</i>2<i>bx c</i> thoả mãn với mọi x sao cho   1 <i>x</i> 1<sub> và </sub> <i>f x</i>( ) <i>p</i><sub>.Nên :</sub>
- Với x = 1  <i>a b c</i>  <i>p</i> (1)



- Với x = - 1  <i>a b c</i>  <i>p</i> (2)
- Với x = 0  <i>c</i> <i>p</i> (3)


Từ (1) và (2)  <i>a b c</i>   <i>a b c</i>  2<i>p</i> mà <i>a b c</i>   <i>a b c</i>  2<i>b</i>
Nên suy ra : <i>b</i> <i>p</i>


Ta có  <i>b c</i>  <i>b c</i> <i>c</i> <i>b</i>  <i>p p</i>2<i>p</i>


Kết hợp với (1): <i>a</i>   <i>a b c</i>   <i>b c</i> 3<i>p</i> <i>a</i> 3<i>p</i>
5


<i>a</i> <i>b</i> <i>c</i> <i>p</i>


   


.Vậy số q nhỏ nhất là 5


<b>Bài 46:</b><i> ( HSG TỈNH NGHỆ AN NĂM HỌC 2010 – 2011).</i>


a) Với

a

Z

<sub> thì </sub>

a

3

a

(a

1)a(a

1)

<sub> là tích 3 số tự nhiên liên tiếp nên chia hết cho 2 và 3.</sub>


Mà (2.3)=1
3


a

a 6



 



3 3 3



1 1 2 2 n n


S

P

(a

a )

(a

a )

...

(a

a ) 6





Vậy

S 6

P 6



b)

n

6

n

4

2n

3

2n

2

n (n

2

1) .(n

2 2

2n

2)



với

n

N

<sub>, n > 1 thì </sub>

n

2

2n

 

2

(n 1)

2

1

<sub> > </sub>

(n 1)

2


n

2

2n

 

2

n

2

2(n 1)

<

n

2


Vậy

(n 1)

2<

n

2

2n

2

<sub><</sub>

n

2

n

2

2n

2

<sub> khơng là số chính phương </sub>

<sub> đpcm</sub>


c)+Nếu

n 3

n

2

n 3

nên

n

2

n

 

2 3

<sub> (1)</sub>
+Nếu

n 3

 

n

2

2 3

n

2

n

 

2 3

<sub> (2)</sub>


Từ (1) và (2)

  

n

Z

thì

n

2

n

 

2 3


d)Đặt

m

2

n

2

17

(m

N)



m

2

n

2

17

(m

n)(m

n) 17 1.17

=17.1
Do m + n > m - n


m

n

17

m

9



m

n

1

n

8








<sub></sub>

<sub></sub>







Vậy với n = 8 ta có

n

2

17

64

17

81 9

2


<b>Bài 47:</b><i> ( HSG TỈNH NGHỆ AN NĂM HỌC 2011 2012).</i>


a) Để n + 21 và n 18 là hai số chính phơng
2


21


<i>n</i> <i>p</i>


<sub> và </sub><i>n</i>18<i>q p q N</i>2( ,  )


2 2 <sub>(</sub> <sub>21) (</sub> <sub>18) 39</sub> <sub>(</sub> <sub>)(</sub> <sub>) 39</sub>


</div>
<span class='text_page_counter'>(22)</span><div class='page_container' data-page=22>

1
39
<i>p q</i>
<i>p q</i>
 



 


 <sub> Hc </sub>


3
13
<i>p q</i>
<i>p q</i>
 


 

20
19
<i>p</i>
<i>q</i>


 


 <sub> Hc </sub>


8
5
<i>p</i>
<i>q</i>






Víi
20
19
<i>p</i>
<i>q</i>





  <sub> n = 379 (TM) ; Víi </sub>
8
5
<i>p</i>
<i>q</i>





  <sub>n = 43(TM)</sub>


VËy víi n = 379 hoặc n = 43 thì n +21 và n 18 là hai số chính phơng
b) Ta có:


 




2 2 2 2 2 2 2 2


2 <sub>2</sub>


4 4 4 4 0


2 2 (2 1) 1 2 2 2 1 2 2 2 1 1


0
0


2 2 2 1 1


2 2 2 1 1 1


1


2 2 2 1 1


2 2 2 1 1 1


1


<i>x</i> <i>xy y</i> <i>x y</i> <i>x</i> <i>xy</i> <i>y</i> <i>x y</i>


<i>x</i> <i>y</i> <i>xy</i> <i>x</i> <i>y</i> <i>xy</i> <i>x</i> <i>y</i> <i>xy</i>


<i>x</i>
<i>y</i>



<i>x</i> <i>y</i> <i>xy</i>


<i>x</i> <i>y</i> <i>xy</i> <i>x</i>


<i>y</i>


<i>x</i> <i>y</i> <i>xy</i>


<i>x</i> <i>y</i> <i>xy</i> <i>x</i>


<i>y</i>
       
            
 



     <sub></sub>

 <sub></sub>
     

 <sub></sub>
 <sub> </sub>
<sub></sub> <sub></sub> <sub></sub> <sub></sub> <sub></sub> <sub></sub> <sub></sub>

  
  




<sub></sub> <sub></sub>


Vậy phơng trình có 3 nghiệm lµ (x,y) = (0;0); (1;-1);(-1;1).


<b>Bài 48:</b><i> ( HSG TỈNH NGHỆ AN NĂM HỌC 2015 – 2016).</i>


a) Nhận xét:


n2<sub> + (n+5)</sub>2<sub> = 2n</sub>2<sub> + 10n + 25 = X + 25</sub>
(n+1)2<sub> + (n+4)</sub>2<sub> = 2n</sub>2<sub> + 10n + 17 = X + 17</sub>
(n+2)2<sub> + (n+3)</sub>2<sub> = 2n</sub>2<sub> + 10n + 13 = X + 13</sub>


Lần thứ nhất, chia 6 vật có khối lượng 19992<sub>, ..., 2004</sub>2<sub> thành ba phần: A+25, A+17, A+13</sub>
Lần thứ hai, chia 6 vật có khối lượng 20052<sub>, ..., 2010</sub>2<sub> thành ba phần: B+25, B+17, B+13</sub>
Lần thứ ba, chia 6 vật có khối lượng 20112<sub>, ..., 2016</sub>2<sub> thành ba phần: C+25, C+17, C+13</sub>


Lúc này ta chia thành các nhóm như sau: Nhóm thứ nhất A+25, B+17, C+13; nhóm thứ hai B+25,
C+17, A+13; nhóm thứ ba C+25, A+17, B+13. Khối lượng của mỗi nhóm đều bằng A + B + C +
55 gam.


b) Viết phương trình đã cho về dạng: 9.(3<i>x</i> – 2<sub> +19) = </sub><i><sub>y</sub></i>2<sub> (x</sub><sub></sub><sub>2). Để </sub><i><sub>y</sub></i><sub> là số nguyên thì điều kiện</sub>
cần và đủ là 3<i>x</i> – 2<sub> + 19 = </sub><i><sub>z</sub></i>2<sub> là số chính phương (z là số nguyên dương)</sub>


Nếu <i>x</i> – 2 = 2<i>k</i> + 1 là số lẻ thì 32<i>k + </i>1<sub> + 19 = (3</sub>2<i>k</i> + 1<sub> + 1) + 18 = 4.B + 18 chia hết cho 2 nhưng không</sub>
chia hết cho 4 nên khơng thể là số chính phương.


Do đó <i>x </i>– 2 = 2<i>k</i> là số chẵn



Ta có 3<i>x</i> – 2<sub> + 19 = </sub><i><sub>z</sub></i>2

3

 

3

19


<i>k</i> <i>k</i>


<i>z</i> <i>z</i>


   


. Vì 19 là số nguyên tố và <i>z</i> 3<i>k</i>  <i>z</i> 3<i>k</i><sub> nên</sub>
3 1
3 19
<i>k</i>
<i>k</i>
<i>z</i>
<i>z</i>
  


 


10 10
2
3<i>k</i> 9


<i>z</i> <i>z</i>
<i>k</i>
 
 


 <sub></sub>  <sub></sub>

 <sub></sub>

Vậy <i>x </i>= 6 và <i>y</i> = 30.


<b>Bài 49:</b><i> ( HSG TỈNH NGHỆ AN NĂM HỌC 2016 – 2017).</i>


Ta có P(x) =


2 <sub>2</sub> <sub>2</sub>


b

b

b



x

c

c



2

4

4





 

 





<sub> Suy ra </sub>


2

b



c

1

<sub>b</sub>

<sub>4</sub>




</div>
<span class='text_page_counter'>(23)</span><div class='page_container' data-page=23>

Vậy, P(x) =

x

2

4x 3

.


b)Khơng mất tính tổng qt giả sử

m n p q.

 


Nếu

m 3

thì


1

1

1 1

1

1 1 1

1

1



1.


m

n

p q

mnpq

 

3 5 7 11 3.5.7.11


Vậy

m 2

và (1) trở thành


1

1 1

1

1



n

p q

2npq

2

<sub> (2).</sub>


Nếu

n 5

ta có


1

1 1

1

1 1

1

1

1



.


n

p q

2npq

 

5 7 11 2.5.7.11 2



Vậy

n 3

và (2) trở thành

 



1 1

1

1



p 6 q 6

37



p q 6pq

 

6




suy ra

p 7

q 43.



Vậy

m;n;p;q

2;3;7;43

và các hoán vị của nó.


<b>Bài 50:</b><i>( HSG TỈNH NGHỆ AN NĂM HỌC 2018 – 2019).</i>


a) Ta có:

2

<i>y</i>

2

 

<i>x</i>

2

<i>y</i>

 

5

<i>xy</i>

<i>x y</i>

(

1) 2

<i>y</i>

2

2

<i>y</i>

5


5



2



1



<i>x</i>

<i>y</i>



<i>y</i>





<sub>(</sub><i><sub>y =1</sub></i><sub> khơng thỏa mãn PT)</sub>
Vì <i>x, y</i> là các số nguyên nên <i>y -1</i> là ước của 5.


1:

1 1

2

9.



<i>TH y</i>

 

<i>y</i>

 

<i>x</i>



2 :

1

1

0

5.



<i>TH</i>

<i>y</i>

 

<i>y</i>

 

<i>x</i>






3:

1 5

6

13.



<i>TH</i>

<i>y</i>

 

<i>y</i>

 

<i>x</i>



4 :

1

5

4

9.



<i>TH</i>

<i>y</i>

 

<i>y</i>

 

<i>x</i>





Vậy PT có các nghiệm nguyên (<i>x;y</i>) là: (9;2), (-5;0), (13;6), (-9;-4).


b) Ta có

2

2

4

16



<i>n</i> <i><sub>n</sub></i>


<i>A</i>



2


2

<i>n</i>

1

4

<i>n</i>

1

18





Đặt

2

2

2

2


<i>n</i> <i><sub>k</sub></i>




*



<i>k</i>

 



suy ra

2

2

1 2

2

1 4

1 3



<i>n</i> <i><sub>k</sub></i> <i><sub>k</sub></i>




Do đó với mọi <i>n</i> nguyên dương ta có:

2

2

1 3; 4

1 3; 18 3



<i>n</i> <i><sub>n</sub></i>




2


2

<i>n</i>

4

<i>n</i>

16 3


<i>A</i>





c) Ta có:


2

5



5 2

2



<i>xy</i>

<i>y</i>

<i>x</i>

<i>y</i>




<i>y</i>





(vì y =0 khơng thỏa mãn PT)


vì x, y là các số nguyên nên y là ước của 5.


1:

1

7.



<i>TH</i>

<i>y</i>

 

<i>x</i>



2 :

1

7.



<i>TH</i>

<i>y</i>

 

<i>x</i>





3:

5

11.



<i>TH</i>

<i>y</i>

 

<i>x</i>



4 :

5

11.



<i>TH</i>

<i>y</i>

 

<i>x</i>





</div>
<span class='text_page_counter'>(24)</span><div class='page_container' data-page=24>

d)Ta có

<i>A</i>

4

<i>n</i>

17

4

1

18



<i>n</i>





Với mọi <i>n</i> nguyên dương ta có:

4

<i>n</i>

1 3; 18 3


4

<i>n</i>

17 3



<i>A</i>





<b>Bài 51:</b><i>( HSG TỈNH HUẾ NĂM HỌC 2006 – 2007).</i>


<i>a</i><sub> là số chính phơng, nên </sub><i>a</i>1, 4,9<sub>.</sub>


Ta cú 92 81; 102 100 nên khơng có số 9<i>x</i> nào là số chính phơng. Do đó <i>a</i> chỉ có thể là 1
hoc 4.


<i>ad</i><sub> là số chính phơng nên </sub><i>ad</i> <sub> chỉ có thể là 16, hoặc 49. Nên </sub><i>d</i><sub> chỉ có thể là 6 hoặc 9.</sub>
<i>cd</i><sub> là số chính phơng nên </sub><i>cd</i><sub> chỉ có thể là 16, hoặc 36, hoặc 49. Nên Nên </sub><i>c</i><sub> chỉ có thể là 1,</sub>
hoặc 3, hc 4.


Nếu <i>a</i>1 thì <i>d</i> 6và <i>c</i>1 hoặc <i>c</i>3, khi đó <i>abcd</i> 1 16<i>b</i> <i>hay b</i>1 36 và

 

 



2 2


1 6<i>bc</i>  <i>x</i>4 <i>hay x</i>6
.
Ta cã:


2 2 2 2 2


26 676; 34 1156; 36 1296; 44 1936; 46 2126<sub>. Chỉ chọn đợc 1936.</sub>



Nếu <i>a</i>4 thì <i>d</i> 9 và <i>c</i>4, khi đó

 

 



2 2


4 49 3 7


<i>abcd</i>  <i>b</i>  <i>x</i> <i>hay x</i>
.
Ta cã:


2 2 2


63 3969; 67 4489; 73 5329<sub>. Khơng chọn đợc số nào.</sub>


VËy chØ cã c¸c chữ số <i>a</i>1,<i>b</i>9, <i>c</i>3, <i>d</i> 6 thỏa mÃn điều kiện bài toán.


b) + Gi x l s hc sinh gii cả 3 mơn Văn, Tốn, Ngoại ngữ
(x > 0), dựa vào biểu đồ ta có:


Sè häc sinh chØ giái mét môn Toán là:



70 49 32 <i>x</i>


Số học sinh chỉ giỏi một môn Văn là:



65 49 34 <i>x</i>


Số học sinh chỉ giỏi một môn Ngoại ngữ là:




62 34  32 <i>x</i>


+ Có 6 học sinh khơng đạt yêu cầu nên:




111 6 70 49    32 <i>x</i> 65 49  34 <i>x</i> 62 34  32 <i>x</i> 
49

32 <i>x</i>

 

 34 <i>x</i>


82 <i>x</i> 105 <i>x</i> 23


    


VËy cã 23 häc sinh giỏi cả 3 môn


<b>Bi 52:</b><i>( HSG TNH HNG YấN NM HỌC 2013 – 2014).</i>


<b> </b>


Vì <i>x, y</i> nguyên nên ta có:

5

<i>x</i>

2

+

2

<i>y</i>

2

2

<i>xy</i>

+

4

<i>x</i>

+

2

<i>y</i>

(*)
⇔ (<i>x</i>2−2<i>xy</i>+<i>y</i>2)+(4<i>x</i>2−4<i>x</i>+1)+(<i>y</i>2−2<i>y</i>+1)≤2

⇔(

<i>x</i>

<i>y</i>

)

2

+(

2

<i>x</i>

1

)

2

+(

<i>y</i>

1

)

2

2



Ta có

(

2

<i>x</i>

1

)

2

2

, 2<i>x</i>−1 <sub> lẻ </sub>

⇒ (

2

<i>x</i>

1

)

2

=

1

⇔[



<i>x</i>

=

0



<i>x</i>

=

1




Với <i>x</i>=0 <sub> thì (*) </sub> ⇔<i>y</i>(<i>y</i>−1)≤0

0

<i>y</i>

1

⇒ [



<i>y</i>

=

0



</div>
<span class='text_page_counter'>(25)</span><div class='page_container' data-page=25>

Với <i>x</i>1<sub> thì (*) </sub>

2

<i>y</i>

2

4

<i>y</i>

+

1

0

⇔2(<i>y</i>−1)2≤1

<i>y</i>

=

1

<sub> ( thỏa mãn)</sub>


Vậy (<i>x, y</i>)=(0<i>;</i>0)<i>;</i>(0<i>;</i>1)<i>;</i>(1<i>;</i>1) .


<b>Bài 53:</b><i>( HSG TỈNH HƯNG YÊN NĂM HỌC 2014 – 2015).</i>


Ta có








2 2


2 2 2 2


2 2


2 2 2 2


5 6 2 2 2 40 0


2 2 2 1 4 4 41



1 2 41


1 2 4 5


<i>x</i> <i>xy</i> <i>y</i> <i>x</i> <i>y</i>


<i>x</i> <i>y</i> <i>xy</i> <i>x</i> <i>y</i> <i>x</i> <i>xy y</i>
<i>x y</i> <i>x y</i>


<i>x y</i> <i>x y</i>


     


         


     


      


TH1:


1 4 2


2 5 1


<i>x y</i> <i>x</i>


<i>x y</i> <i>y</i>


   



 




 


  


  <sub> TH2: </sub>


1 5 0


2 4 4


<i>x y</i> <i>x</i>
<i>x y</i> <i>y</i>


   


 




 


  


  <sub> (loại)</sub>



Vậy các nghiệm nguyên dương của phương trình là (2; 1).


<b>Bài 54:</b><i>( HSG TỈNH KOMTUM NĂM HỌC 2016 – 2017).</i>


Với <i>xyz</i>0. Ta có


1 1 1 1 1


2 2


<i>xy yz zx</i>


<i>x</i> <i>y</i> <i>z</i> <i>xyz</i>


 


    


(1)
Vì <i>x y z</i>  2


Nên (1) trở thành


1


<i>xy yz zx</i>


<i>xyz</i> <i>x y z</i>


 




  <sub> </sub>


(<i>xy yz zx x y z</i>)( ) <i>xyz</i> 0


       <sub> </sub>


2


(<i>xy yz zx x y</i>)( ) <i>z x y</i>( ) 0


      


2


(<i>x y xy xz z</i>)( <i>yz</i>) 0


     


(<i>x y y z z x</i>)( )( ) 0


    


0
0
0


<i>x y</i>
<i>y z</i>


<i>z x</i>


 




  


  


2
2
2


<i>z</i>
<i>x</i>
<i>y</i>






 



 



 <sub> (2)</sub>


Từ (2) suy ra có ít nhất một trong ba số

<i>x y z</i>

, ,

phải bằng

2

.


<b>Bài 55:</b><i>( HSG TỈNH LẠNG SƠN NĂM HỌC 2015 – 2016).</i>


Ta có

a

n

0, n 1

 

<sub> nên đặt </sub>


n 1
n


1



b

b

2



a

<sub> và</sub>



n 1


n n n 1 n


n 1

a



a

b

b

2n

b

n n 1



2n.a

1













n


1

1

1



a



n(n 1)

n

n 1



 



</div>
<span class='text_page_counter'>(26)</span><div class='page_container' data-page=26>

Vậy : 1 2 2016


2015



S a

a

... a



2016





<b>Bài 56:</b><i>( HSG TỈNH LẠNG SƠN NĂM HỌC 2018 – 2019).</i>



a) Từ giả thiết ta có

<i>f</i>

(2017), (2018)

<i>f</i>

là các số nguyên và <i>x</i> = 2017,
<i>x</i> = 2018 không là nghiệm của PT

<i>f x</i>

( ) 0



Giả sử PT

<i>f x</i>

( ) 0

có nghiệm nguyên là

<i>x a Z</i>

 

, theo định lý Bơ-zu :

<i>f x</i>

( ) (

<i>x a g x</i>

). ( )


với

<i>g x</i>

( )

là đa thức hệ số nguyên không nhận <i>x</i> = 2017, <i>x</i> = 2018 làm nghiệm


Do vậy:

<i>f</i>

(2017) (2017

<i>a g</i>

). (2017), (2018) (2018

<i>f</i>

<i>a g</i>

). (2018)


Nhân vế với vế và áp dụng giả thiết

<i>f</i>

(2017). (2018) 2019

<i>f</i>

:


2019 (2017

<i>a g</i>

). (2017).(2018

<i>a g</i>

). (2018)



Điều này là vơ lý vì vế trái là số lẻ, cịn vế phải là số chẵn (

(2017

<i>a</i>

); (2018

<i>a</i>

)

là 2 số nguyên
liên tiếp, tích là số chẵn)


Vậy

<i>f x</i>

( ) 0

khơng có nghiệm ngun (đpcm)
GV có thể mở rộng cho HS:


- Số 2017 và 2018 có thể thay bởi bất cứ số nguyên nào miễn sao có 1 số chẵn và 1 số lẻ. Số 2019 có
thể thay bằng 1 số nguyên lẻ bất kỳ.


- Liệu có tìm được đa thức nào hệ số ngun thỏa mãn giả thiết

(2017). (2018) 2019



<i>f</i>

<i>f</i>

<sub>?</sub>


- Đa số chứng minh phương trình khơng có nghiệm đều sử dụng phương pháp phản chứng (dựa vào
chia hết, số tận cùng ...)


b) Gọi số trận hòa là x (

x

<i>N</i>

*

)

tổng số điểm của các trận hòa là 2x, (1 trận hịa có 2 đội,
mỗi đội được 1 điểm)


Theo giả thiết số trận thắng là 4x

tổng số điểm của các trận thắng là 12x
Tổng số điểm các đội là 336

2x + 12x = 336

x = 24


Vậy ta có tất cả 24 + 4.24 = 120 trận đấu diễn ra


Từ giả thiết có n đội, mỗi đội đấu với n – 1 đội còn lại nên số trận đấu diễn ra là n(n – 1) , nhưng đây
là tính cả trận lượt đi và lượt về, giả thiết mỗi đội đấu với nhau đúng 1 lần nên tổng số trận giảm đi
một nửa, do đó có tất cả


n(n 1)


2





trận đấu
Vậy


n(n 1)



120

(

1) 240

16, (

15



2

<i>n n</i>

<i>n</i>

<i>n</i>









loại)
KL : Có tất cả 16 đội bóng tham gia.



<b>Bài 57:</b><i>( HSG TỈNH LONG AN NĂM HỌC 2011 – 2012).</i>


a)


+) Kí hiệu hình thang ABCD cần tìm như hình vẽ.
+) Tính được C(


6
;5)
<i>a</i> <sub>; D(</sub>


</div>
<span class='text_page_counter'>(27)</span><div class='page_container' data-page=27>

6
5
4
3
2
1
-1
-2
-3
-4


-10 -8 -6 -4 -2 2 4 6 8 10


D
C
B
A
O


y=1
y=5
BC =
6


<i>a</i><sub>; AD = </sub>
2
<i>a</i>
+)


6 2


.4 : 2 8
<i>ABCD</i>
<i>S</i>
<i>a</i> <i>a</i>
 
<sub></sub>  <sub></sub> 
 


 <sub>a = 2 ( Thỏa ĐK a > 0) </sub>


+) Vậy phương trình đường thẳng là y = 2x – 1.




b ) Ta có


1 1 1



2
<i>x</i> <i>y</i>  <i>z</i>  


2


1 1 1


4


<i>x</i> <i>y</i> <i>z</i>


 
  
 
 
Do đó
2
2
11121
<i>xyzxyz</i>





2 2 2 2


1 1 1 2 2 2 2 1


0
<i>x</i> <i>y</i> <i>z</i> <i>xy</i> <i>yz</i> <i>zx</i> <i>xy</i> <i>z</i>



        


2 2 2 2


1 2 1 1 2 1


0


<i>x</i> <i>xz</i> <i>z</i> <i>y</i> <i>yz</i> <i>z</i>


 
 
 <sub></sub>   <sub></sub><sub></sub>   <sub></sub>
   
2
2


1 1 1 1


0


<i>x</i> <i>z</i> <i>y</i> <i>z</i>


 
 
 <sub></sub>  <sub></sub> <sub></sub>  <sub></sub> 
   
2
2



1 1 <sub>1</sub> <sub>1</sub>


0


1 1


1 1
0


<i>x</i> <i>z</i> <i><sub>x</sub></i> <i><sub>z</sub></i>


<i>x</i> <i>y</i> <i>z</i>


<i>y</i> <i>z</i>
<i>y</i> <i>z</i>
 <sub></sub>  <sub></sub>  
  <sub></sub> 
 
 
 <sub></sub>  <sub></sub>   

 
  <sub></sub>
 
 
 <sub></sub>
 

Thay vào



1 1 1


2


<i>x</i> <i>y</i> <i>z</i>  <sub> ta được x = y = </sub>
1
2 ; z =


1
2


Khi đó P =


2012
2012


1 1 1


2. 1 1


2 2 2




 


   



 


 


c) Ta có: xy + 2x = 27 – 3y


xy 2x 3y

27



Û

+

+

=



 <i>x y</i>

2

3

<i>y</i>2

33


(x

3)(y

2) 33



Û

+

+ =

<sub> </sub>


Û



x 3 1


y 2 33



ì + =


ïï



íï + =



ïỵ

<sub> hoặc</sub>


x 3 33


y 2 1




ì + =


ïï



íï + =



ïỵ

<sub>hoặc</sub>


x 3 3


y 2 11



ì + =


ïï



íï + =



ïỵ

<sub>hoặc</sub>


x 3 11


y 2 3



ì + =


ïï



íï + =


ïỵ



do x > 0, y > 0.


Û



x

2


y

31


ì


=-ïï


íï =


ïỵ

<sub>(loại)hoặc</sub>

x

30


y

1


ì =


ïï


íï


=-ïỵ

<sub>(loại)hoặc</sub>

x

0


y 9


ì =


ïï


íï =


ïỵ

<sub>(loại)hoặc</sub>

x 8


y 1


ì =


ïï


íï =


ïỵ

<sub>(tđk)</sub>


Vậy cặp số ngun dương cần tìm là (x; y) = (8;1)


<b>Bài 58:</b><i>( HSG TỈNH NAM ĐỊNH NĂM HỌC 2015 – 2016).</i>


</div>
<span class='text_page_counter'>(28)</span><div class='page_container' data-page=28>

Tương tự <i>yz x</i>  1

<i>y</i>1

 

<i>z</i> 1

và <i>zx y</i>  1

<i>z</i> 1

 

<i>x</i> 1




Suy ra

 

 

 

 

 

 

 



1 1 1 3


1 1 1 1 1 1 1 1 1


<i>x y z</i>
<i>S</i>


<i>x</i> <i>y</i> <i>y</i> <i>z</i> <i>z</i> <i>x</i> <i>x</i> <i>y</i> <i>z</i>


  


   


        


 



1 1


1


<i>xyz</i> <i>xy yz zx</i> <i>x y z</i> <i>xy yz zx</i>




 



        


Ta có



2 <sub>2</sub> <sub>2</sub> <sub>2</sub>


2 7


<i>x y z</i>  <i>x</i> <i>y</i> <i>z</i>  <i>xy yz zx</i>   <i>xy yz zx</i>  


Suy ra


1
7


<i>S</i> 


<i>b)</i> Ta có



2 2 2


2 2


1 1 1 4


<i>x</i>  <i>y</i> <i>xy x y</i>    <i>x y</i>  <i>x</i>  <i>y</i> 
Ta có bảng giá trị tương ứng (học sinh có thể xét từng trường hợp)


<i>x y</i> <i><sub>x</sub></i><sub></sub> <sub>1</sub> <i><sub>y</sub></i><sub></sub> <sub>1</sub>



Nghiệm

<i>x y</i>;



2 0 0

<sub></sub>

<sub>1;1</sub>

<sub></sub>



-2 0 0 Loại


0 2 0 Loại


0 -2 0

<sub></sub>

<sub></sub><sub>1;1</sub>

<sub></sub>



0 0 2 Loại


0 0 -2

<sub></sub>

<sub>1; 1</sub><sub></sub>

<sub></sub>



Vậy các số

<i>x y</i>;

cần tìm là

1;1

<i><b>, </b></i>

1;1

<i><b>, </b></i>

1; 1



<i><b>c)</b></i> Với mỗi số nguyên dương <i>k</i> ta có

 



2 <sub>1</sub> 2 <sub>1</sub> <sub>1</sub> <sub>1</sub> <sub>1</sub>


<i>k</i> <i>k</i>   <i>k</i>    <i>k</i> <i>k</i>


<i><b>.</b></i>


Sử dụng đẳng thức trên liên tiếp với <i>k</i>3,4,...,<i>n</i> ta được


 



3 1 2.4 1 2 1 3.5 1 2 1 3 1 4.6



1 2 1 3 1 1 <i>n</i> 1 <i>n</i> 1


         


      





 

2



1 2 1 3 1 1 <i>n</i> 1 <i>n</i> 1 2 3 4... <i>n</i> 1 <i>n</i>


        


Ta có điều phải chứng minh.


d) Giả sử tất cả các câu lạc bộ đều có khơng q 8 học sinh.
Gọi N là số câu lạc bộ có hơn 1 học sinh.


Nếu N 4 <sub>, từ 5 trong số các câu lạc bộ này, chọn mỗi câu lạc bộ 2 học sinh, khi đó 10 học </sub>
sinh này khơng thỏa mãn điều kiện bài tốn.


Nếu N<4, khi đó số học sinh tham gia các câu lạc bộ này không quá 3.8 24 <sub>, nghĩa là cịn </sub>
ít nhất 35 24 11  <sub> học sinh, mỗi học sinh tham gia 1 câu lạc bộ mà câu lạc bộ này chỉ có 1 học </sub>
sinh. Chọn 10 học sinh trong số này, khơng thỏa mãn điều kiện bài tốn.


Vậy N=4.


Số học sinh tham gia 4 câu lạc bộ này không quá 4.8 32 <sub>, nghĩa là cịn ít nhất 3 học sinh, mỗi học </sub>


sinh tham gia 1 câu lạc bộ mà câu lạc bộ này chỉ có 1 học sinh.


</div>
<span class='text_page_counter'>(29)</span><div class='page_container' data-page=29>

Vậy điều giả sử sai, nghĩa là tồn tại một câu lạc bộ có ít nhất 9 học sinh tham gia.


<b>Bài 59:</b><i>( HSG TỈNH NINH BÌNH NĂM HỌC 2011 – 2012).</i>
Ta có a3 <sub>+ b</sub>3 <sub>+ c</sub>3 <sub>- 3abc = (a+b+c)(a</sub>2<sub>+b</sub>2<sub>+c</sub>2<sub>-ab-bc-ca)</sub>


=> 1-3abc = 1-ab-bc-ca => ab + bc + ca = 3abc
mà 12 <sub>= (a+b+c)</sub>2 <sub>= a</sub>2 <sub>+ b</sub>2 <sub>+ c</sub>2 <sub>+ 2(ab+bc+ca)</sub>
=> ab+bc+ca=0 => abc=0


=> a=0 hoặc b=0 hoặc c=0


Nếu a = 0 =>


2 2
3 3


1


1


1


<i>b c</i>



<i>b</i>

<i>c</i>



<i>b</i>

<i>c</i>



 











<sub></sub>

<sub></sub>





=>b2 <sub>+ c</sub>2 <sub>+ 2bc = 1 => 2bc=0</sub>


=>(a,b,c) =(0,0,1) hoặc (a,b,c) =(0,1,0)


Nếu b = 0 làm tương tự =>(a,b,c) =(0,0,1) hoặc (a,b,c) =(1,0,0)
Nếu c = 0 làm tương tự =>(a,b,c) =(0,1,0) hoặc (a,b,c) =(1,0,0)
Vậy mọi trường hợp ta có P = 1


<b>Bài 60:</b><i>( HSG TỈNH PHÚ THỌ NĂM HỌC 2008 – 2009).</i>
Phơng trình đã cho tơng đơng với


1 1 1


1
xy yz zx  <sub>.</sub>


Không mất tính tổng quát, giả sử x y z  (*)


- NÕu z 3 th× 2



1 1 1 3 1


1
xy yz zx z  3 <sub> (lo¹i).</sub>


- Nếu z 2 thì phơng trình đã cho trở thành


2xy x y 2   <sub>.</sub>


Hay

2x 1 2y 1

 

5.


Do (*) nên chỉ có trờng hợp 2x - 1 = 5 và 2y - 1 = 1, suy ra x = 3 và y = 1
- Nếu

z 1

thì phơng trình đã cho trở thành


xy x y 1  

x 1 y 1

 

2.


Do (*) nên chỉ có trờng hợp x - 1 = 2 vµ y - 1 = 1, suy ra x = 3 vµ y = 2.
NghiƯm lµ: (3 ; 2 ; 1), (3 ; 1 ; 2), (2 ; 3 ; 1), (2 ; 1 ; 3), (1 ; 3 ; 2), (1 ; 2 ; 3).


<b>Bài 61:</b><i>( HSG TỈNH PHÚ THỌ NĂM HỌC 2009 – 2010).</i>


a) Theo giả thiết n là số tự nhiên nên: 2n<sub> 1, 2</sub>n<sub> , 2</sub>n<sub> + 1 lµ 3 sè tù nhiên liên tiếp. </sub>


Vì tích của 3 số tự nhiên liên tiếp luôn chia hết cho 3 nên
(2n<sub> - 1).2</sub>n<sub>.(2</sub>n<sub> + 1) chia hết cho 3</sub>


Mặt khác (2n<sub>, 3) = 1 nên </sub>

2 1 2

 

1



<i>n</i> <i>n</i>



 


</div>
<span class='text_page_counter'>(30)</span><div class='page_container' data-page=30>

b) Ta thấy B là số chính phơng 4B là số chính phơng


Đặt 4B = k2<sub> (k</sub><sub></sub><sub>N) th× 4B = 4n</sub>2–<sub> 4n + 52 = k</sub>2  <sub>(2n-1-k)(2n-1+k) =-51</sub>


Vì 2n-1+k 2n-1-k nên ta có các hÖ


2 1 1


(1)


2 1 51


<i>n</i> <i>k</i>


<i>n</i> <i>k</i>


  




  


2 1 3


(2)



2 1 17


<i>n</i> <i>k</i>


<i>n</i> <i>k</i>


  




  


2 1 51


(3)


2 1 1


<i>n</i> <i>k</i>


<i>n</i> <i>k</i>


  





  


2 1 17


(4)


2 1 3


<i>n</i> <i>k</i>


<i>n</i> <i>k</i>


  




  


Giải hệ (1), (2), (3), (4) ta tìm đợc n = -12, n =-3, n =13, n =4
Vậy các số nguyên cần tìm là n 

12; 3;4;13



<b>Bài 62:</b><i>( HSG TỈNH PHÚ THỌ NĂM HỌC 2012 – 2013).</i>
a) 8<i>x</i>2 3x<i>y</i> 5<i>y</i>25


⇔<i>y</i>(3<i>x</i>+5)=8<i>x</i>2−25⇔<i>y</i>=8<i>x</i>


2



−25


3<i>x</i>+5 ⇔9<i>y</i>=24<i>x</i>−40−
25


3<i>x</i>+5∈<i>Z</i>
Khi 3x+5 là ước 25 từ đó tìm được (<i>x; y</i>)∈

{

(−10<i>;</i>−31)<i>;</i>(−2<i>;</i>−7)<i>;</i>(0<i>;</i>−5)

}



b) Với n chẵn n = 2k thì




2 2 2


2 .4 3 (2 1).4 (16 9 ) 7 2 1 7


7 1


14 1 14 6


2


<i>k</i> <i>k</i> <i>k</i> <i>k</i> <i>k</i>


<i>A</i> <i>k</i> <i>k</i> <i>k</i>


<i>t</i>


<i>k</i> <i>n</i> <i>t</i> <i>m</i> <i>m N</i>



       




       


 


Với n lẻ n = 2k+1


<i>A</i>=(2<i>k</i>+1). 42<i>k</i>+1+32<i>k</i>+1=2<i>k</i>. 42<i>k</i>+1+(42<i>k</i>+1+32<i>k</i>+1)7⇒2<i>k</i>7⇒<i>k</i>=7<i>t</i>⇒<i>n</i>=14<i>m</i>+1(<i>m</i>∈<i>N</i>)


Vậy <i>n</i>=14<i>m</i>+6 <sub> hoặc </sub> <i>n</i>=14<i>m</i>+1 <sub> ( với mọi n</sub>

¿

<i>N</i>

)

<sub> thì A chia hết cho 7</sub>


c)


2 2 <sub>x</sub> 2


<i>x</i> <i>yz</i> <i>y</i> <i>z</i> <i>z</i> <i>xy</i>


<i>a</i> <i>b</i> <i>c</i>


  


 


2 2 2


2 2



4 2 2 2 2 2 3 3 2 3 3 3


2 2


4 2 2 2 2 2 3 3 2 3 3 3


2 2


4 2 2 2 2 2 3 3 2


(1)


2 ( 3 )


: (2)


2 ( 3 )


:
2


<i>a</i> <i>b</i> <i>c</i>


<i>x</i> <i>yz</i> <i>y</i> <i>xz</i> <i>z</i> <i>xy</i>


<i>a</i> <i>bc</i> <i>a</i> <i>bc</i>


<i>x</i> <i>x yz y z</i> <i>y z</i> <i>xy</i> <i>xz</i> <i>x yz</i> <i>x x</i> <i>y</i> <i>z</i> <i>xyz</i>



<i>b</i> <i>ac</i> <i>b</i> <i>ac</i>


<i>Tuongtu</i>


<i>y</i> <i>y xz x z</i> <i>x z</i> <i>x y</i> <i>yz</i> <i>xy z</i> <i>y x</i> <i>y</i> <i>z</i> <i>xyz</i>


<i>c</i> <i>ab</i> <i>c</i> <i>ab</i>


<i>Tuongtu</i>


<i>z</i> <i>xyz</i> <i>x y</i> <i>x y</i> <i>x z</i> <i>y z</i> <i>xyz</i> <i>z</i>


  


  




  


       




 


       





 


     (<i>x</i>3<i>y</i>3<i>z</i>3 3<i>xyz</i>)(3)
Từ (1) (2) (3) ta có ĐPCM


<b>Bài 63:</b><i>( HSG TỈNH PHÚ THỌ NĂM HỌC 2013 – 2014)</i>


a)

<i>x</i>2+5y2−4xy+4x−8y−12=0⇔<i>x</i>2¿−4<i>x</i>(<i>y</i>−1)−(5y2−8y−12)=0(∗¿

)để PT(*) có



</div>
<span class='text_page_counter'>(31)</span><div class='page_container' data-page=31>

<b>Cách khác</b>

<i>x</i>

2

+

5y

2

4xy

+

4x

8y

12

=

0

⇔(

<i>x</i>

2

<i>y</i>

+

2

)

2

+

<i>y</i>

2

=

16

=

4

2

+

0

2

xét từng trường hợp sẽ ra nghiệm



b) Ta có

<i>P</i>

(

<i>x</i>

)=

<i>x</i>

3

3x

2

+

14x

2

=(

x-2

)(

<i>x</i>

2

-x

+

12

)+

22



để P(x) chia hết 11 thì

(

x-2

)(

<i>x</i>

2

-x

+

12

)

11

<sub>mà </sub>

(

<i>x</i>

2

-x

+

12

)=

<i>x</i>

(

x-1

)+

1

+

11

<sub> ta có</sub>



<i>x</i>(<i>x</i>−1)+1

<sub> khơng chia hết cho 11, suy ra </sub>

(

<i>x</i>

2

-x

+

12

)

<sub> không chia hết cho 11 nên </sub>


x-2 chia hết cho 11 mà x < 100 ;

<i>x</i>∈<i>N</i>

<sub> suy ra </sub>

<i>x</i>

{

2

<i>;</i>

13

<i>;</i>

22

<i>;</i>

35

<i>;</i>

47

<i>;</i>

57

<i>;</i>

68

<i>;</i>

79

<i>;</i>

90

}



<b>Cách khác: </b>

<i>P</i>

(

<i>x</i>

)=

<i>x</i>

3

3x

2

+

14x

2

=[(

x-1

)

3

1

+

11

<i>x</i>

]

11

⇔[(

x-1

)

3

1

]

11



Suy ra (x-1)

3

<sub> chia co 11 dư 1 suy ra x-1 chia cho 11 dư 1 suy ra x chia cho 11 dư 2 mà </sub>


x < 100 suy ra kết quả



c) Ta có


 



 




2
3


2


3 2


1 2


3 2 2


4 5 2 <sub>1</sub> <sub>2</sub> 2


<i>a</i> <i>a</i>


<i>a</i> <i>a</i> <i>a</i>


<i>P</i>


<i>a</i> <i>a</i> <i>a</i> <i><sub>a</sub></i> <i><sub>a</sub></i> <i>a</i>


 


  


  


      <sub>; </sub>





3 <sub>110 3 55</sub>3 2 <sub>3024</sub> 3<sub>55</sub> <sub>3024</sub> 3<sub>55</sub> <sub>3024 .</sub>


<i>a</i>      


3 <sub>110 3</sub> 3 <sub>3</sub> <sub>110 0</sub>


<i>a</i> <i>a</i> <i>a</i> <i>a</i>


       <sub>.</sub>


<i>a</i> 5

<i>a</i>2 5<i>a</i> 22

0 <i>a</i> 5


      


. Suy ra
7
3


<i>P</i>


.
d) Ta có <i>x</i>33<i>x</i>1(1), <i>y</i>3 3<i>y</i>1 (2), <i>z</i>33<i>z</i>1 (3).


Từ (1), (2) và (3) suy ra









3 3 <sub>2</sub> <sub>2</sub>


3 3 2 2


2 2


3 3


3 <sub>3 (4)</sub>


3 3 (5)


3 (6).
3


<i>x</i> <i>y</i> <i>x y</i> <i><sub>x</sub></i> <i><sub>xy y</sub></i>


<i>y</i> <i>z</i> <i>y z</i> <i>y</i> <i>yz z</i>


<i>z</i> <i>zx x</i>


<i>z</i> <i>x</i> <i>z x</i>


       


 <sub></sub>





     


 


 


  


   <sub></sub>


 <sub> </sub>


Từ (4) và (5) suy ra


<i>x</i>2 <i>z</i>2<i>xy yz</i>  0

<i>x y x y z</i>

 

 

 0 <i>x y z</i>  0,
(vì <i>x, y, z</i> đơi một phân biệt).


Cộng (4), (5) và (6) theo vế với vế ta có


2 2 2

2 2 2 2


3 1


9 6


2 <i>x</i> <i>y</i> <i>z</i> 2 <i>x y z</i>    <i>x</i> <i>y</i> <i>z</i>  <sub>.</sub>


<b>Bài 64:</b><i>( HSG TỈNH PHÚ THỌ NĂM HỌC 2014 – 2015).</i>



a)


<i>x</i>

2

+

<i>y</i>

2

<i>xy</i>

=

<i>x</i>

+

<i>y</i>

+

2

2

<i>x</i>

2

+

2

<i>y</i>

2

2

<i>xy</i>

2

<i>x</i>

2

<i>y</i>

+

2

=

6


⇔(

<i>x</i>

<i>y</i>

)

2

+(

<i>x</i>

1

)

2

+(

<i>y</i>

1

)

2

=

6



PT có 6 nghiệm ( ; )<i>x y</i> 

2;0 ; 0;2 ; 3; 2 ; 2;3 ; 1;0 ; 0; 1

 

 

 

 

 


b)Đặt a + b - c = z; b + c – a = x; a + c – b = y thì x + y + z = a + b + c
Ta có

(

<i>x</i>

+

<i>y</i>

+

<i>z)</i>

3

<i>x</i>

3

<i>y</i>

3

<i>z</i>

3

=

3

(

<i>x</i>

+

<i>y</i>

)(

<i>y</i>

+

<i>z</i>

)(

<i>x</i>

+

<i>z</i>

)



</div>
<span class='text_page_counter'>(32)</span><div class='page_container' data-page=32>

a) Ta có: 4

S =

1.2 .3 .4+2.3.4 .(5−1)+3 .4 .5 .(6−2)+...+<i>n</i>(<i>n</i>+1)(<i>n</i>+2)

[

(<i>n</i>+3)−(<i>n</i>−1)

]


=

1.2.3.4

+

2.3.4.5

1.2.3.4

+

3.4.5.6

2.3.4.5

+

...

+

<i>n</i>

(

<i>n</i>

+

1

)(

<i>n</i>

+

2

)(

<i>n</i>

+

3

)−(

<i>n</i>

1

)

<i>n</i>

(

<i>n</i>

+

1

)(

<i>n</i>

+

2

)



=

<i>n</i>

(

<i>n</i>

+

1

)(

<i>n</i>

+

2

)(

<i>n</i>

+

3

)



Do đó

4

<sub>S</sub>

+ 1

=

<i>n</i>

(

<i>n</i>

2

+

<i>n</i>

)(

+

+

<i>n</i>

)(

1

3

)+

1

=

(

<i>n</i>2+3<i>n</i>

) (

<i>n</i>2+3<i>n</i>+2

)

+1


=

(

<i>n</i>

2

+

3

<i>n</i>

)

2

+

2

(

<i>n</i>

2

+

3

<i>n</i>

)

+

1



=

(

<i>n</i>

2

+

3

<i>n</i>

+

1

)

2

<sub>. Vậy </sub>

4

<sub>S</sub>

+ 1

là số chính phương.



b)Ta có:

<i>x</i>

2

+

2

<i>y</i>

2

+

2

<i>xy</i>

=

<i>y</i>

+

2

(

<i>x</i>

+

<i>y</i>

)

2

=−

<i>y</i>

2

+

<i>y</i>

+

2

(

<i>x</i>

+

<i>y</i>

)

2

=

(

1

+

<i>y</i>

) (

2

<i>y</i>

)


Do

(

<i>x</i>

+

<i>y)</i>

2

0,

<i>x , y</i>

nên

(

1

+

<i>y</i>

) (

2

<i>y</i>

)≥

0

⇔−

1

<i>y</i>

2

. Suy ra

<i>y</i>

{

1

<i>;</i>

0

<i>;</i>

1

<i>;</i>

2

}


Với

<i>y</i>

=−

1

, PT trở thành <i>x</i>2−2<i>x</i>+1=0⇔<i>x</i>=1∈<i>Z</i>


Với

<i>y</i>

=

0

, PT trở thành <i>x</i>2−2=0⇒<i>x</i>∉<i>Z</i>
Với

<i>y</i>

=

1

, PT trở thành <i>x</i>2+2<i>x</i>−1=0⇒<i>x</i>∉<i>Z</i>


Với

<i>y</i>

=

2

, PT trở thành <i>x</i>2+4<i>x</i>+4=0⇔<i>x</i>=−2∈<i>Z</i> <sub>.</sub>
Vậy có 2 cặp

(

<i>x; y</i>

)

thỏa mãn đề bài

(

1

<i>;</i>

1

)

<i>;</i>

(−

2

<i>;</i>

2

)

.
c)Ta có


<i>x</i>


<i>x</i>2+<i>x</i>+1=


1


4⇔4<i>x</i>=<i>x</i>


2<sub>+</sub><i><sub>x</sub></i><sub>+</sub><sub>1</sub><sub>⇔</sub><i><sub>x</sub></i>2<sub>=</sub><sub>3</sub><i><sub>x</sub></i><sub>−</sub><sub>1</sub>


Khi đó

<i>x</i>

3

=

<i>x</i>

2

.

<i>x</i>

=(

3

<i>x</i>

1

)

<i>x</i>

=

3

<i>x</i>

2

<i>x</i>

=

3

(

3

<i>x</i>

1

)−

<i>x</i>

=

8

<i>x</i>

3


<i>x</i>

4

=

<i>x</i>

3

.

<i>x</i>

=(

8

<i>x</i>

3

)

<i>x</i>

=

8

<i>x</i>

2

3

<i>x</i>

=

8

(

3

<i>x</i>

1

)−

3

<i>x</i>

=

21

<i>x</i>

8


<i>x</i>

5

=

<i>x</i>

4

.

<i>x</i>

=(

21

<i>x</i>

8

)

<i>x</i>

=

21

<i>x</i>

2

8

<i>x</i>

=

21

(

3

<i>x</i>

1

)−

8

<i>x</i>

=

55

<i>x</i>

21


Suy ra P =


<i>x</i>5−4<i>x</i>3−17<i>x</i>+9


<i>x</i>4+3<i>x</i>2+2<i>x</i>+11 =


(55<i>x</i>−21)−4(8<i>x</i>−3)−17<i>x</i>+9
(21<i>x</i>−8)+3(3<i>x</i>−1)+2<i>x</i>+11


=


6<i>x</i>


32<i>x</i>=


3



16 <sub> ( do </sub> <i>x</i>≠0 <sub> ). Vậy P = </sub>


3
16 <sub>.</sub>


<b>Bài 66:</b><i>( HSG TỈNH PHÚ THỌ NĂM HỌC 2017 – 2018).</i>


a) Ta có






2 2 <i>a</i> <i>b</i> <i>a b</i> 1<sub>.</sub>


<i>a b c</i> <i>b c a</i>


<i>bc ab</i> <i>ab ca</i> <i>c b a</i> <i>c</i>


      


  


Suy ra <i>ab bc ca</i>   0 <i>bc</i><i>a b c</i>

 <i>abc a b c</i> 2

2018.(1)


2



0 .(2)



<i>ab bc ca</i>    <i>ab</i><i>c a b</i>  <i>abc c a b</i> 
Từ (1) và (2) ta được



2


2018.
<i>c a b</i> 


b)Đặt <i>b qa c q a q</i> ;  2

1

thì ta được


2


a 1 <i>q q</i> 91 13.7.
Trường hợp 1: Nếu <i>q</i> là số tự nhiên thì ta được


2


1 1


1; 9; 81.


9


1 91


<i>a</i> <i>a</i>


<i>a</i> <i>b</i> <i>c</i>


<i>q</i>


<i>q q</i>


 


 


    


 




   <sub></sub>




2


7 7


7; 21; 63.
3


1 13


<i>a</i> <i>a</i>


<i>a</i> <i>b</i> <i>c</i>


<i>q</i>


<i>q q</i>


 


 


    


 




   <sub></sub>




2


13 13


13; 26; 52.


2


1 7


<i>a</i> <i>a</i>


<i>a</i> <i>b</i> <i>c</i>



<i>q</i>
<i>q q</i>


 


 


    


 




   <sub></sub>


</div>
<span class='text_page_counter'>(33)</span><div class='page_container' data-page=33>

Trường hợp 2: Nếu <i>q</i> là số hữu tỷ thì giả sử


3; 2 .
<i>x</i>


<i>q</i> <i>x</i> <i>y</i>


<i>y</i>


  


Khi đó



2 2 2 2


a 1 <i>q q</i> 91 <i>a x</i> <i>xy y</i> 91<i>y</i>

<i>x</i>2<i>xy y</i> 2 19




Ta có


2


2 2 2


2 2 91 6; 5.


<i>ax</i> <i>a</i>


<i>c</i> <i>a ty</i> <i>x</i> <i>xy y</i> <i>x</i> <i>y</i>


<i>y</i> <i>y</i>


          


và <i>a</i>25;<i>b</i>30;<i>c</i>36.


Vậy có 8 bộ số

<i>a b c</i>; ;

thỏa mãn

1;9;81 , 81;9;1 , 7;21;63 , 63; 21;7 ;...

 

 

 



<b>Bài 67:</b><i>( HSG TỈNH QUẢNG BÌNH NĂM HỌC 2013 – 2014).</i>


Xét

<i>n Aa a</i>

(

9)

suy ra

<i>n</i>

 

1

<i>A a</i>

(

1)

Do đó

<i>S n</i>

(

1)

<i>S n</i>

( ) 1



Suy ra



2


(

1). ( ) 87

( )

( ) 87




<i>S n</i>

<i>S n</i>

<i>S n</i>

<i>S n</i>

<sub> không có nghiệm ngun dương.</sub>


Xét



Aa99...9


ơ 9
<i>k s</i>


<i>n</i>



với

<i>a</i>

9,

<i>k</i>

 

* thì



A(a+1)00...0


ơ 0


1



<i>k s</i>


<i>n</i>

 



Suy ra

<i>S n</i>

( )

<i>S n</i>

(

1)

<i>S A</i>

( )

<i>a</i>

9

<i>k</i>

 

<i>S A</i>

( )

<i>a</i>

1

9

<i>k</i>

1

,

<i>k</i>

 

*


Ta tìm n nguyên dương nhỏ nhất thỏa mãn

<i>S n</i>

(

1). ( ) 87

<i>S n</i>

<i>S n</i>

(

1)

<i>S n</i>

( ) 9

<i>k</i>

1

,

<i>k</i>

 

*

87 87.1 29.3

.


Nếu

<i>S n</i>

( ) 87

. (

<i>S n</i>

1) 1

 

<i>S n</i>

( )

<i>S n</i>

(

1) 86 9

<i>k</i>

1

,

<i>k</i>

 

*


Nếu

<i>S n</i>

( ) 29

. (

<i>S n</i>

1) 3

 

<i>S n</i>

( )

<i>S n</i>

(

1) 26 9.3 1

suy ra k=3.
Khi đó


Aa 999


<i>n</i>



suy ra số n nhỏ nhất thỏa

<i>S n</i>

( ) 29

<i>n</i>

2999

.


<b>Bài 68:</b><i>( HSG TỈNH QUẢNG BÌNH NĂM HỌC 2015 – 2016).</i>


Từ điều kiện

 



2 2 2 4 2


1

3

5

8

15



<i>n</i>

  

<i>n</i>

<i>m</i>

<i>m</i>

<i>m</i>

<i>m</i>

<i>m</i>

<i>m</i>

<i>m</i>


Xét phương trình bậc hai :

<i>n</i>

2

 

<i>n</i>

(

<i>m</i>

4

<i>m</i>

2

8

<i>m</i>

16)

0 (1)

(ẩn số n)


Để phương trình (1) có nghiệm ngun dương thì

 

4

<i>m</i>

4

4

<i>m</i>

2

32

<i>m</i>

63

phải là một số chính
phương.


Ta có

 

  



2 2 2



2 2


2

<i>m</i>

2

4

<i>m</i>

4

3

2

<i>m</i>

2 ,

<i>m</i>

*



Mặt khác



2
2


2

<i>m</i>

1

32

<i>m</i>

2



 



Do đó

 



2 2


2 2


2

<i>m</i>

1

32

<i>m</i>

2

2

<i>m</i>

1 ,

<i>m</i>

2



Khi đó:

  



2 2


2 2


2

<i>m</i>

1

2

<i>m</i>

2 ,

<i>m</i>

2



Suy ra (1) chỉ có nghiệm nguyên dương

<i>n</i>

khi

<i>m</i>

1

hoặc

<i>m</i>

2




</div>
<span class='text_page_counter'>(34)</span><div class='page_container' data-page=34>

Nếu

<i>m</i>

2

<sub> thì </sub>





 

<sub>  </sub>








2

4



20

0



5 (lo¹i)



<i>n</i>


<i>n</i>

<i>n</i>



<i>n</i>



Thử lại m = 2 và n = 4 thỏa mãn điều kiện bài toán.


<b>Kết luận :</b> m = 2 ; n = 4.


<b>Bài 69:</b><i>( HSG TỈNH QUẢNG BÌNH NĂM HỌC 2018 – 2019).</i>


a) Xác định các hệ số a và b để đa thức

 




4 3 2


P x x  2x 3x ax b


là bình phương của một đa
thức.


<b><sub> Lời giải 1. </sub></b><sub>Do đa thức </sub>P x

 

<sub> có bậc 4 với hệ số cao nhât là 1. Giả sử </sub>

 



2
2


P x  x mx n


.


Hay ta được

 



2


2 4 3 2 2 2


P x  x mx n x 2mx  m 2n x 2mnx n


.


Từ đó ta được



4 3 2 4 3 2 2 2



x  2x 3x ax b x  2mx  m 2n x 2mnx n


Đồng nhất hệ số hai vế ta được
2


2
2


2m 2 m 1 m 1


2n 2


m 2n 3 n 1


a 2n


a 2mn a 2


b 1
b n


b n


     


  




  



  


 


  <sub></sub> 


 


  


  <sub></sub>   <sub></sub>




Vậy đa thức đã cho là

 



4 3 2


P x x  2x 3x  2x 1


.


<b><sub> Lời giải 2. </sub></b><sub>Biến đổi đa thức đã cho như sau</sub>


 



 

 






4 3 2 4 3 2


4 3 2 3 2 2


2
2


P x x 2x 3x ax b x 2x 3x 2x 1 a 2 x b 1


x x x x x x x x 1 a 2 x b 1


x x 1 a 2 x b 1


             


            


      


Để P x

 

viết được thành bình phương của một đa thức khác thì ta cần có

a 2 x

b 1

0 với
mọi x. Do Vậy ta có a 2 b 1 0    <sub> hay </sub>a2;b 1 <sub> thỏa mãn yêu cầu bài toán.</sub>


b)<b><sub> Lời giải. </sub></b><sub>Ta xét các trường hợp sau.</sub>


+ Trường hợp 1. Với n là số tự nhiên lẻ, khi đó ta đặt n 2k 1 k N 

. Từ đó ta được


2k


n 2k 1



C 2019 2020 C 2019  2020 2019 2020 1 2020


       


Dễ thấy 2019 chia 4 có số dư là 3 và


2k


2020 1


chia 4 có số dư là 1, từ đó ta được C chia 4 có số dư
là 3 nên C khơng thể là số chính phương.


</div>
<span class='text_page_counter'>(35)</span><div class='page_container' data-page=35>

2


n 2k k


C2019 2020 2019 2020 2019 2020


Do C là số chính phương tồn tại số tự nhiên t thỏa mãn


2


k 2


2019 2020 t


. Dễ thấy 2019k<sub> và t cùng </sub>


là số lẻ. Ta có

 




2


k 2 k k


2019 2020 t  t 2019 t 2019 2020


. Do 2019k<sub> và t cùng là số lẻ nên</sub>


<sub>t 2019</sub>k







k


t 2019


cùng là các ước chẵn của 2020. Từ đó ta xét các khả năng sau.


<sub> Với </sub>


k


k
k


t 506


t 2019 2



2019 504


t 2019 1010


    


 




 




  <sub></sub>


 


 <sub> (loại do t và </sub>2019k<sub> là các số lẻ).</sub>


<sub> Với </sub>


k


k
k


t 106



t 2019 10


2019 96


t 2019 202


    


 




 




  <sub></sub>


 


 <sub> (loại do t và </sub>2019k<sub> là các số lẻ).</sub>


Do đó với n là số chẵn thì C khơng thể là số chính phương.


Vậy khơng tồn tại số tự nhiên n để C 2019 n 2020<sub> là một số chính phương.</sub>


<b>Bài 70:</b><i>( HSG TỈNH QUẢNG NAM NĂM HỌC 2012 – 2013).</i>


Ta chứng minh s

5

<sub> – s chia hết cho 5, tương tự các số t, x, t, z rồi cộng lại suy ra điều </sub>


cần chứng minh. ( Xem lại

<b>Bài 5: </b><i>( HSG TỈNH PHÚ YÊN NĂM HỌC 2018 – 2019))</i>


<b>Bài 71:</b><i>( HSG TỈNH QUẢNG NAM NĂM HỌC 2013 – 2014).</i>


<b>a)</b> Với a, b, c, d, e, f là các số thực khác 0, ta có:


2


2 2 2


2 2 2


2 2 2


2 2 2


a

b c

a

b

c



1

1



d

e

f

d

e

f



a

b

c

2ab 2bc 2ac



1



d

e

f

de

ef

df



a

b

c

2abc f

d

e



1




d

e

f

def

c

a

b





 

<sub></sub>

<sub></sub>









<sub></sub>

<sub></sub>







d

e

f



0


a

b c


Vậy


2 2 2


2 2 2


a

b

c




B



d

e

f





= 1


<b>b)</b> Đặt n2<sub> – 14n – 256 = k</sub>2<sub> (k  </sub>

<sub>)</sub>
 (n – 7)2<sub> – k</sub>2<sub> = 305</sub>


 (n – 7 – k)(n – 7 + k) = 305


</div>
<span class='text_page_counter'>(36)</span><div class='page_container' data-page=36>

n 7 k 1


n 7 k 305



n 7 k

305



n 7 k

1



n 7 k 5


n 7 k 61



n 7 k

61



n 7 k

5










<sub></sub>

<sub></sub>

<sub></sub>

<sub></sub>


 





 





<sub></sub>


<sub></sub>

<sub></sub>

<sub></sub>

<sub></sub>









<sub></sub>

<sub></sub>

<sub></sub>



<sub> </sub>

n 160


k 152


n

146


k 152


n 40


k 28


n

26


k 28









<sub></sub>

<sub></sub>


 



 


 




<sub></sub>


<sub></sub>

<sub></sub>









<sub></sub>





Vì n và k là các số tự nhiên nên ta chọn n = 160 hoặc n = 40.


<b>c)</b>




 

 




  

 

  

 



8n 4n 8n 4n


n n


8 4


n 1 n 2 n 1 n 2


8 8 8 4 4 4


A a

3a

4

a

1

3 a

1



a

1

3 a

1



a

1

a

a

... 1

3 a

1

a

a

... 1





<sub></sub>

<sub></sub>

<sub></sub>

<sub></sub>







 


 




4 4 4


4 4


a

1 a

1 .B 3 a

1 .C



a

1

a

1 .B 3C







<sub></sub>

<sub></sub>



<sub>a</sub>

2

<sub>1 a</sub>

 

2

<sub>1 .D</sub>





Vì a là số tự nhiên lớn hơn 5 và không chia hết cho 5 nên:






2
2
2
2


a 5k 1

a

1 5




a 5k 2

a

1 5



a 5k 3

a

1 5



a 5k 4

a

1 5



 


 


 
















(với k là số nguyên dương)


Vậy



8n 4n



a

3a

4

<sub></sub>



5 với mọi số tự nhiên n.


<b>d)</b> Ta có :


2 2 2 2 2 2


a

b

c

1

a

1 b

c

1



1 a 1

1 a 0



 

 



      


Tương tự :

1 b 0; 1 c 0

 

 


 (1 + a)(1 + b) (1 + c) ≥ 0


 1 + a + b + c + ab + ac + bc + abc ≥ 0 (1)


Mặt khác: (1 + a + b + c)2<sub> = (1 + a)</sub>2<sub> + (b + c)</sub>2<sub> + 2(1 + a)(b + c)</sub>
= 1 + a2<sub> + b</sub>2<sub> + c</sub>2<sub> + 2a + 2b + 2c + 2ab + 2ab + 2bc</sub>


= (a2<sub> + b</sub>2<sub> + c</sub>2<sub>) + (a</sub>2<sub> + b</sub>2<sub> + c</sub>2<sub>) + 2a + 2b + 2c + 2ab + 2ab + 2bc</sub>
= 2(a2<sub> + b</sub>2<sub> + c</sub>2<sub> + a + b + c + ab + ac + bc)</sub>


 a2<sub> + b</sub>2<sub> + c</sub>2<sub> + a + b + c + ab + ac + bc = </sub>

1



2

<sub>(1 + a + b + c)</sub>2<sub> ≥ 0 (2)</sub>

Cộng (1) và (2) vế theo vế ta được :


</div>
<span class='text_page_counter'>(37)</span><div class='page_container' data-page=37>

 abc + 2(1 + a + b + c + ab + ac + bc) ≥ 0


<b>Bài 72:</b><i>( HSG TỈNH QUẢNG NAM NĂM HỌC 2017– 2018).</i>


<i>a)</i>

Cho ba số thực , ,

<i>a b c</i>

thỏa 1

<i>a b c</i>

, ,

2.

Chứng minh :



7



<i>a b c</i>

<i>a c b</i>



<i>b c a</i>

<i>c b a</i>

<sub>. </sub>



Vì a,b,c có vai trị như nhau và

1

<i>a b c</i>

, ,

2

nên giả sử 2 ≥ a ≥b ≥ c ≥ 1
Khi đó: (b - a)(b - c) ≤ 0


 b2<sub> + ac ≤ ab + bc (*)  </sub> 1


<i>a b</i> <i>a</i>


<i>b c</i>  <i>c</i> <sub>( chia 2 vế (*) cho bc) </sub>


và 1


<i>b c</i> <i>c</i>


<i>a b</i>  <i>a</i><sub> ( chia 2 vế (*) cho ab)</sub>


 2 2( )



<i>a b b</i> <i>c</i> <i>a</i> <i>c</i> <i>a</i> <i>c</i>


<i>b</i> <i>c</i><i>a b</i> <i>c</i> <i>a</i>   <i>c</i><i>a</i> <sub> </sub>


Để chứng minh (1) ta tiếp tục chứng minh 2 2( )
<i>a</i> <i>c</i>
<i>c</i> <i>a</i>


 


 7 


5
2


<i>a</i> <i>c</i>


<i>c</i><i>a</i>  <sub> (2)</sub>
Ta có: 2 ≥ a ≥ c ≥ 1  1 2


<i>a</i>
<i>x</i>


<i>c</i>
  


(2)  x+


1



<i>x</i><sub>  </sub>


5


2<sub>  2x</sub>2<sub>5x+2  0  (x2)(2x1)  0 ( đúng vì </sub>1<sub> </sub><i>x</i> 2
(2) được chứng minh  (1) được chứng minh.


Dấu “=”xảy ra khi a=2, b=c=1 hoặc a=b=2, c=1 và các hoán vị của nó.


<i>b) Chứng minh rằng với mọi số tự nhiên n </i><i> 1 thì (n+2)(n+1)(n+8) khơng thể là lập phương</i>
<i>của một số tự nhiên.</i>


Ta có: (n+2)2<sub>< (n+2)(n+1)(n+8) < (n+4)</sub>3<sub> (*)</sub>


 n3<sub>+ 6n</sub>2<sub>+12n+8 < (n</sub>2<sub>+3n+2) (n+8)=n</sub>3<sub>+ 11n</sub>2<sub>+26n+16 < n</sub>3<sub>+ 12n</sub>2<sub>+48n+64 ( đúng)</sub>


Giả sử có nN, n  1 sao cho (n+2)(n+1)(n+8) là lập phương của một số tự nhiên, khi đó, từ
(*) suy ra: (n+2)(n+1)(n+8) =( n+3)3<sub> </sub>


 n3<sub>+ 11n</sub>2<sub>+26n+16 = n</sub>3<sub>+ 9n</sub>2<sub>+27n+27</sub>
 2n2<sub>  n 11 =0  </sub>


1 89


4


<i>n</i>  <i>N</i>
(!)



Vậy n  1, n  N thì (n+2)(n+1)(n+8) khơng là lập phương của một số tự nhiên.


<i>c) Cho số nguyên tố </i>

<i>p</i>

<i>p</i>3

<i> và hai số nguyên dương a, b thỏa mãn phương trình</i>


2 2 2


<i>p</i> <i>a</i> <i>b</i> <i><sub>. Chứng minh a chia hết cho 12 và </sub></i>2(<i>p a</i> 1)<i><sub> là số chính phương.</sub></i>
Ta có: <i>p</i>2<i>a</i>2 <i>b</i>2  <i>p</i>2 (<i>b a b a</i> )(  ).


Các ước của <i>p</i>2<sub> là 1, </sub><i><sub>p</sub></i><sub> và p</sub>2<sub> ; không xảy ra trường hợp </sub><i><sub>b</sub></i><sub> + </sub><i><sub>a</sub></i><sub> = </sub><i><sub>b</sub></i><sub> ‒ </sub><i><sub>a</sub></i><sub> = </sub><i><sub>p</sub></i>
Do đó chỉ xảy ra trường hợp <i>b</i> + <i>a</i> = <i>p</i>2<sub> và </sub><i><sub>b</sub></i><sub> ‒ </sub><i><sub>a</sub></i><sub> = 1. </sub>


Khi đó


2 2


1 1


à


2 2


<i>p</i> <i>p</i>


<i>b</i>  <i>v a</i> 


</div>
<span class='text_page_counter'>(38)</span><div class='page_container' data-page=38>

Từ <i>p</i> lẻ suy ra <i>p</i> + 1, <i>p</i> ‒1 là hai số chẵn liên tiếp  (<i>p </i>‒1)(<i>p</i> + 1) chia hết cho 8.
Suy ra 2<i>a</i> chia hết cho 8 (1)


Từ <i>p</i> nguyên tố lớn hơn 3 nên <i>p</i> không chia hết cho 3. Do đó <i>p</i> có dạng 3<i>k</i>+1 hoặc 3<i>k</i>+2.


Suy ra một trong hai số <i>p</i> + 1; <i>p</i> ‒1 chia hết cho 3 . Suy ra 2<i>a</i> chia hết cho 3 (2)
Từ (1) và (2) suy ra 2<i>a</i> chia hết cho 24 hay a chia hết cho 12 (đpcm).


Xét




2


2
2


p -1


2 p + a + 1 =2 p+ +1 =2p+p +1= p+1


2


 


 


  <sub> là số chính phương.</sub>


<b>Bài 73:</b><i>( HSG TỈNH QUẢNG NGÃI NĂM HỌC 2008– 2009).</i>


a) Gọi a, b, c là số đo 3 cạnh của tam giác vuông cần tìm. Giả sử 1 a b c

  

<sub>.</sub>


Ta có hệ phương trình :



2 2 2



a

b

c



ab 2(a b c)







 



<sub> </sub>



(1)


(2)


Từ (1)

<sub> c</sub>

2

<sub> = (a + b)</sub>

2

<sub>− 2ab</sub>



c

2

= (a + b)

2

− 4(a + b + c) (theo (2))



<sub>(a + b)</sub>

2

<sub>− 4(a + b) = c</sub>

2

<sub> + 4c</sub>



<sub>(a + b)</sub>

2

<sub>− 4(a + b) + 4 = c</sub>

2

<sub> + 4c + 4.</sub>



<sub>(a + b − 2)</sub>

2

<sub> = (c + 2)</sub>

2

<sub> </sub>

<sub></sub>

<sub>a + b − 2</sub>

<sub> = c + 2 (do a + b </sub>

<sub></sub>

<sub>2)</sub>



<sub>c = a + b − 4.</sub>



Thay vào (2) ta được: ab = 2(a + b + a + b − 4)



<sub>ab −4a−4b + 8 = 0 </sub>

<sub>b(a −4) −4(a−4) = 8 </sub>

<sub>(a −4)(b−4) = 8</sub>



Phân tích 8 = 1.8 = 2.4 nên ta có:



a 4 1



ho

ho



b 4 8



 







<sub></sub>

<sub></sub>

<sub></sub>





a - 4 = 2

a = 5

a = 6



Ỉc

Ỉc



b - 4 = 4

b = 12

b =8



Từ đó ta có 2 tam giác vng có các cạnh (5 ; 12 ; 13) và (6 ; 8 ; 10) thỏa mãn u cầu


của bài tốn.



b) Ta có:



1 1



<i>a</i> <i>b</i>


<i>a</i> <i>b</i>


 




=



2


2


<i>ab b ab a</i> <i>ab a b</i> <i>a b</i>


<i>ab</i> <i>ab</i> <i>ab</i> <i>ab</i>


    


   


: là số nguyên


Suy ra :



<i>a b</i>
<i>ab</i>





là số nguyên và a, b là số nguyên dương


Nên



<i>a b</i>
<i>ab</i>




¿

1

a + b

¿

ab



Do d là ước của a nên a

d

<sub></sub>

a

<sub></sub>

d > 0


Và d là ước của b nên b

d

<sub></sub>

b

<sub></sub>

d > 0


Suy ra : ab

d

2

nên a + b

d

2


Vậy :

<i>d</i>  <i>a</i><i>b</i>


c) - Nếu y chẵn thì với mọi x

Z có 2008x

2009

+ 2009y

2010

là số chẵn; mà 2011 là số lẻ,


(vô lý)



</div>
<span class='text_page_counter'>(39)</span><div class='page_container' data-page=39>

cho 4 dư 3, (vô lý)


Vậy khơng có các số nguyên x, y nào thỏa mãn hệ thức :



2008x

2009

<sub> + 2009y</sub>

2010

<sub> = 2011. </sub>



<b>Bài 74:</b><i>( HSG TỈNH QUẢNG NGÃI NĂM HỌC 2009– 2010).</i>


a) Ta có: 6<i>x</i>5<i>y</i>18 2 <i>xy</i> 2xy - 6x - 5y = 18  2xy - 6x + 15 - 5y = 33 <sub></sub> <sub> 2x(y – </sub>


3) – 5(y – 3) = 33



 <sub> (y – 3)(2x – 5) = 33 = 1.33 = 3.11 </sub>


Ta xét các trường hợp sau :
*


3 1 19


2 5 33 4


<i>y</i> <i>x</i>


<i>x</i> <i>y</i>


  


 




 


  


 


*


3 33 3



2 5 1 36


<i>y</i> <i>x</i>


<i>x</i> <i>y</i>


  


 




 


  


 


*


3 11 4


2 5 3 14


<i>y</i> <i>x</i>


<i>x</i> <i>y</i>


  



 




 


  


 


*


3 3 8


2 5 11 6


<i>y</i> <i>x</i>


<i>x</i> <i>y</i>


  


 




 


  



 


Các cặp số nguyên dương đều thỏa mãn đẳng thức trên.
Vậy các cặp số cần tìm là : (3; 36); (4; 14); (8; 6); (19; 4)


b) Vì a chẵn nên a = 2k

k N


Do đó


3 2 3 2


8 4 2


24 8 12 3 2 6


<i>k</i> <i>k</i> <i>k</i> <i>k</i> <i>k</i> <i>k</i>


<i>A</i>     


 



3 2 <sub>1 2</sub> <sub>1</sub>


2 3


6 6


<i>k k</i> <i>k</i>


<i>k</i>  <i>k</i> <i>k</i>  



 


Ta có : k k+1 2

  k k+1 2k+1 2

 



Ta chứng minh : <i>k k</i>

1 2

 

<i>k</i> 1 3

Thật vậy :
- Nếu <i>k</i> = <i>3n (vớin N</i> <i><sub>) </sub></i><sub> thì </sub><i>k k</i>

1 2

 

<i>k</i> 1 3



- Nếu <i>k</i> = <i>3n</i> + <i>1(vớin N</i> <i><sub>) </sub></i><sub> thì </sub>2<i>k</i> 1 3


- Nếu <i>k</i> = <i>3n</i> + <i>2(vớin N</i> <i><sub>) </sub></i><sub> thì </sub><i>k</i> 1 3


Với mọi <i>k N</i>  <i>k k</i>

1 2

 

<i>k</i>1

luôn chia hết cho 2 và cho 3
Mà (2, 3) = 1  <i>k k</i>

1 2

 

<i>k</i> 1 6

Vậy A có giá trị nguyên.


<b>Bài 75:</b><i>( HSG TỈNH QUẢNG NGÃI NĂM HỌC 2010– 2011).</i>


Chia hình trịn ra 8 phần bằng nhau. Vì 17 = 8.2 + 1, nên tồn tại một phần chứa ít nhất 3 điểm
(khơng thẳng hàng). Do đó có ít nhất 3 điểm lập thành một tam giác có diện tích nhỏ hơn


1
8<sub>.</sub>


<b>Bài 76:</b><i>( HSG TỈNH QUẢNG NGÃI NĂM HỌC 2013– 2014).</i>


a) 2a

2

<sub>+a = 3b</sub>

2

<sub>+b </sub>

<sub>⇔</sub>

<sub>2a</sub>

2

<sub>+a −2b</sub>

2

<sub>−b = b</sub>

2

<sub> (</sub>

<sub>⇔</sub>

<sub>a−b)(2a+2b+1) = b</sub>

2

Gọi (a−b,2a+2b+1) = d



</div>
<span class='text_page_counter'>(40)</span><div class='page_container' data-page=40>

Mà a – b

d

a

d



a

d; b

d mà 2a+2b+1

d nên 1

d

d=1



Vậy phân số đã cho tối giản.



b)

Gọi cạnh hình vng ABCD nhỏ nhất chứa bên trong 5 đường trịn có bán kính


bằng 1cm và đơi một khơng có q 1 điểm trong chung là x (cm).



Từ đây suy ra các tâm của 5 đường tròn này nằm trong hình vng MNPQ có


cạnh bằng x – 2 cm. (vì tâm của các đường trịn các đường trịn cách cạnh hình vng


ít nhất 1cm).



Chia hình vng MNPQ thành 4 hình vng nhỏ có độ dài mỗi cạnh là


x 2


(cm)
2




-. (hình vẽ)



Theo ngun lí Dirichlet có ít nhất hai tâm đường trịn cùng thuộc một hình


vng. Giả sử hai tâm đó là O

1

.O

2

.



Vì hai đường trịn này có khơng q 1 điểm


chung nên O

1

O

2

khơng nhỏ hơn hai lần bán kính và


khơng lớn hơn độ dài đường chéo của hình vng


cạnh



x 2
(cm)
2





-.



Hay 2 ≤ O

1

.O

2



(x 2) 2
2


=>


(x 2) 2


2 x 2 2 2 x 2 2 2


2


-³ Þ - ³ Þ ³ +


Vậy cạnh hình vng nhỏ nhất chứa 5 đường


trịn có bán kính bằng 1 và 5 đường trịn này đơi một


khơng có quá 1 điểm chung là

2 2 2+


<b>Bài 77:</b><i>( HSG TỈNH QUẢNG NGÃI NĂM HỌC 2015– 2016).</i>


a) Ta có 5(a + b+ c ) = a.b.c (1)



Từ (1) suy ra a,b,c một trong ba số phải có 1 số chia hết cho 5.



Giả sử c chia hết cho 5 mà c là số nguyên tố nên c =5 .Với c =5 ta có :



5(a + b + 5 ) = a.b.5 nên (1-b)(1- a) = 6



TH1: 1 – b = 2 và 1 – a = 3 nên b = -1 và a = -2 (trường hợp này không thỏa mãn )


TH2: 1 – b = 3 và 1 – a = 2 nên b = -2 và a = -1 (trường hợp này không thỏa mãn )


TH3: 1 – b = -3 và 1 – a = -2 nên b = 4 và a = 3 (trường hợp này không thỏa mãn )


TH4: 1 – b = -1 và 1 – a = -6 nên b = 2và a = 7 (trường hợp này thỏa mãn )



TH5: 1 – b = -6 và 1 – a = -1 nên b = 7 và a = 2 (trường hợp này thỏa mãn )



TH6: 1 – b = -2 và 1 – a = -3 nên b = 3 và a = 4 (trường hợp này không thỏa mãn )


Vậy c = 5; b = 2, a = 7 và c = 5; a = 2, b = 7.



b) Từ đề bài ta có (x - 2y)(x – y + 2) = -3 = -1.3 = -3.1



TH1: x - 2y = 3và x – y + 2 = -1 nên y = -6 và x = -9(trường hợp này không thỏa mãn)


TH2: x - 2y = -3và x – y + 2 = 1 nên y = 2 và x = 1 (trường hợp này thỏa mãn )



Vậy y = 2 và x = 1



c) Từ giả thuyết đề bài ta có a,b,c >0 .



Q


P
N
M


D <sub>C</sub>



B
A


1cm
1cm


X cm


O1


O4
O2


</div>
<span class='text_page_counter'>(41)</span><div class='page_container' data-page=41>

Từ giả thuyết



2
2


2
1


<i>b</i>
<i>a</i>


<i>b</i>


<sub> ta suy ra </sub>

<i>a b</i>



Từ giả thuyết



2
2


2
1


<i>c</i>
<i>b</i>


<i>c</i>


<sub> ta suy ra </sub>

<i>b c</i>


Từ giả thuyết



2
2


2
1


<i>a</i>
<i>c</i>


<i>a</i>



<sub> ta suy ra </sub>

<i>c a</i>


Lúc đó a = b = c .Thay a = b = c vào phương trình ta có : a = b = c =1


Vậy a = b = c =1



<b>Bài 78:</b><i>( HSG TỈNH QUẢNG NGÃI NĂM HỌC 2016– 2017).</i>


a) Chứng minh rằng với k là số ngun thì 2016k + 3 khơng phải là lập phương của một


số nguyên.



Giả sử 2016k + 3 = a

3

<sub> với k và a là số nguyên. </sub>


Suy ra: 2016k = a

3

<sub> - 3</sub>



Ta chứng minh a

3

<sub> – 3 không chia hết cho 7.</sub>



Thật vậy: Ta biểu diễn a = 7m + r, với r

0;1; 1; 2; 2;3; 3  

.



Trong tất cả các trường hợp trên ta đều có a

3

<sub> – 3 không chia hết cho 7</sub>


Mà 2016k luôn chia hết cho 7, nên a

3

<sub> – 3 </sub>

<sub></sub>

<sub> 2016k. ĐPCM</sub>



b) Tìm nghiệm nguyên của phương trình:

<i>x</i>2 25<i>y y</i>( 6)

Từ

<i>x</i>2 25<i>y y</i>( 6)


Ta có : (y+3+x)(y+3-x) = - 16



Để ý trong phương trình chỉ chứa ẩn số x với số mũ bằng 2 , do đó ta có thể hạn chế giải


với x là số tự nhiên.



Khi đó: y + 3 + x y + 3 - x .




Ta có ( y + 3 + x)+(y + 3 - x) = 2(y + 3) là số chẵn



Suy ra 2 số ( y + 3 + x ) và (y + 3 - x) cùng tính chẵn lẻ . Ta lại có tích của chúng là số


chẵn , vậy 2 số ( y + 3 + x ) và (y + 3 - x) là 2 số chẵn.



Ta chỉ có cách phân tích -16 ra tích của 2 số chẵn sau đây:



</div>
<span class='text_page_counter'>(42)</span><div class='page_container' data-page=42>

Khi y + 3 + x = 8 , y + 3 - x = -2 ta cú x = 5 , y = 0.
Khi y + 3 + x = 4 , y + 3 - x = -4 ta cú x = 4 , y = -3.
Khi y + 3 + x = 2 , y + 3 - x = -8 ta cú x = 5 , y = -6.
Vì thế phơng trình đã cho có các nghiệm :
( x;y) 

5;0 ; 5; 6 ; 4; 3 .

 

 

 

 



<b>Bài 79:</b><i>( HSG TỈNH QUẢNG NINH NĂM HỌC 2004– 2005).</i>


Gi¶ sư tån tại số tự nhiên n thoả mÃn n2<sub> + 2006 là số chính phơng </sub>


thì n2<sub> + 2006 = m</sub>2<sub> với m là số tự nhiên => (m-n)(m+n) = 2006 (*).</sub>


Khi ú:


- nếu m và n khác tính chẵn lẽ thì (m - n)( m + n) lẻ . Mâu thuẫn với (*)


- nếu m và n cùng tính chẵn lÏ th× (m - n)(m + n) chia hÕt cho 4, nhng 2006 không chia hết
cho 4. Cũng mâu thn víi (*)


Tóm lại giả sử trên khơng đúng.


VËy không tồn tại số tự nhiên n thoả mÃn n2<sub> + 2006 là số chính phơng. </sub>



<b>Bi 80:</b><i>( HSG TNH QUẢNG NINH NĂM HỌC 2013– 2014).</i>


a) x2<sub>(y – 5) + x + y – 3 = 0</sub>
 <sub>y(x</sub>2<sub> + 1) = 5x</sub>2<sub> – x + 3</sub>


2


2 2


5x – x 3 2
5


x 1 1


<i>x</i>
<i>y</i>


<i>x</i>


 


  


  <sub> (1)</sub>


y nguyên  2
2
5



1
<i>x</i>
<i>x</i>





 <sub>nguyên </sub> 2
2


1
<i>x</i>
<i>x</i>




 <sub> nguyên</sub>


 <sub>(x + 2) </sub><sub> (x</sub>2<sub> + 1) (vì x + 2 và x</sub>2<sub> + 1 nguyên do x</sub><sub></sub><sub>Z)</sub>
 <sub>(x+2)(x – 2) </sub><sub> (x</sub>2<sub> + 1)</sub>


 <sub>(x</sub>2<sub>+1) – 5 </sub><sub></sub><sub> (x</sub>2<sub> + 1)</sub>
 <sub>5</sub><sub> (x</sub>2<sub> + 1)</sub>



2
2


1 1
1 5


<i>x</i>


<i>x</i>
  


 


 <sub> </sub> <sub>x = 0; x = 2; x = -2</sub>


Thay vào (1) nhận được y tương ứng là 3 ;
21


5 ( Loại); 5
Vậy tìm được hai cặp số (x; y) thỏa mãn phương trình là
(0; 3) ; (-2; 5)


A =


1 1 1


1 <i>x xy</i>1 <i>y yz</i>1 <i>z zx</i>


Có A =



1


1   1  1 


<b>(</b> <b>)</b> <b>(</b> <b>)</b>



<i>z</i> <i>xz</i>


<i>z</i> <i>x xy</i> <i>xz</i> <i>y yz</i> <i>z zx</i>

<sub> </sub>



=

2


1
1


 


     


<i>z</i> <i>xz</i>


<i>z zx xyz</i> <i>xz xyz xyz</i> <i>z zx</i>


=



1


1 1 1


     


<i>z</i> <i>xz</i>


</div>
<span class='text_page_counter'>(43)</span><div class='page_container' data-page=43>

=




1
1


 


 


<i>z zx</i>


<i>z zx</i>

<sub> = 1. </sub>

<sub>Vậy, A = 1</sub>


.


<b>Bài 81:</b><i>( HSG CẤP THÀNH PHỐ QUY NHƠN NĂM HỌC 2012– 2013).</i>


a) A= 1 +3 + 5 + 7 + … + 2013


B = 2 + 4 + 6 + ….+2012


A-B =1 +1 +1 +1 + ….+1



Vì tổng A có (2013-1):2+1=1007 số hạng => A-B = 1007


Vậy A lớn hơn B 1007 đơn vị.



b) Vì f(n+1) = n.(-1)

n+1

<sub> -2f(n), suy ra:</sub>



f(2)= f(1+1)=1.(-1)

2

<sub> -2f(1) =1 – 2f(1)</sub>


f(3)=f(2+1)=2.(-1)

3

<sub> – 2f(2) = -2 -2f(2)</sub>


f(4)=f(3+1)=3(-1)

4

<sub> - 2f(3) = 3 -2f(3)</sub>


f(5)=f(4+1)=4(-1)

5

<sub> – 2f(4) = -4 – 2f(4)</sub>


……….




f(2013)= ………….. =-2012 – 2f(2012)


f(1) = f(2014)= ………….. = 2013 – 2f(2013)


Suy ra:



S=f(1)+f(2)+f(3)+…+f(2013)



=(1 – 2 + 3 – 4 + …… - 2012 + 2013) – (f(1)+f(2)+f(3)+…+f(2013) )


=> 3(f(1)+f(2)+f(3)+…+f(2013)) =1007 => f(1)+f(2)+f(3)+…+f(2013) =



1007


3



c) Ta có a ≥ 0 ; b ≥ 0 ; c ≥ 0:



 

 



  





a b c

ab

bc

ca

2a 2b 2c 2 ab 2 bc 2 ca



a 2 ab b

b 2 bc c

c 2 ca a

0



 

 



<sub></sub>

<sub></sub>







 

<sub></sub>










2 2 2

a

b 0



a

b

b

c

c

a

0

b

c 0



c

a 0



a b c



  

<sub> ∆ ABC đều.</sub>



d) Ta có: a

n

= 2

2n+1

+2

n+1

+1 ; b

n

= 2

2n+1

-2

n+1

+1



=> a

n

– b

n

= 2 .2

2n+1

= 2

2n+2

5 => a

n

; b

n

không đồng thời chia hết cho 5 (1)


a

n

.b

n

= (2

2n+1

+1)

2

–(2

n+1

)

2

= 2

4n+2

+2.2

2n+1

+1 – 2

2n+2

= 2

4n+2

+1=4

2n+1

+1= 4.16

n

+1


+Nếu n= 0 thì 4.16

n

<sub>+1 = 4.1+1 = 5 </sub>



5



+Nếu n > 0 , ta có 16

n

<sub> có chữ số tận cùng là 6=>4.16</sub>

n

<sub> có chữ số tận cùng là 4 </sub>


=>4.16

n

<sub>+1 </sub>




5



</div>
<span class='text_page_counter'>(44)</span><div class='page_container' data-page=44>

Từ (1) và (2) suy ra với mỗi số tự nhiên n, có một và chỉ một trong hai số a

n

hoặc b

n

chia hết cho 5.



<b>Bài 82:</b><i>( HSG TỈNH TÂY NINH NĂM HỌC 2012– 2013).</i>


a) A = n3 <sub>- 3n</sub>2 <sub>- n + 3 = (n -3)(n + 1)(n - 1)</sub>
Vì n lẻ nên n = 2k + 1


Do đó A= 2k(2k - 2)(2k + 2)
= 8 k (k - 1)(k + 1)
Vì (k -1)k(k + 1) chia hết cho 6
Nên A chia hết cho 48


b)B = (x + y)(x + 2y)(x + 3y)(x + 4y) + y

4


<sub>= (x</sub>

2

<sub> + 5xy + 4y</sub>

2

<sub>)( x</sub>

2

<sub> + 5xy + 6y</sub>

2

<sub>) + y</sub>

4

Đặt x

2

<sub> + 5xy + 5y</sub>

2

<sub> = t ( t </sub>

<sub>¿</sub>

<sub> Z) thì</sub>



A = (t - y

2

<sub>)( t + y</sub>

2

<sub>) + y</sub>

4

<sub> = t</sub>

2

<sub> –y</sub>

4

<sub> + y</sub>

4

<sub> = t</sub>

2

<sub> = (x</sub>

2

<sub> + 5xy + 5y</sub>

2

<sub>)</sub>

2


V ì x, y

¿

<sub> Z nên x</sub>

2 ¿

<sub> Z, 5xy </sub>

¿

<sub> Z, 5y</sub>

2 ¿

<sub> Z </sub>

x

2

+ 5xy + 5y

2 ¿

<sub> Z</sub>



Vậy A là số chính phương.



<b>Bài 83:</b><i>( HSG TỈNH THÁI BÌNH NĂM HỌC 2012– 2013).</i>


Tìm các số nguyên x, y thoả mãn:




2 2 3 3


2y 2x 1  2x 2y 1 1 x y (1) 
Ta có (1)  4xy(x y) 2(x y) 1 x y      3 3


Đặt


a x y
b xy


 





 <sub>vì x, y nguyên nên a, b nguyên.</sub>
Khi đó ta có pt : 4ab 2a 1 b    3 <sub>với a, b nguyên </sub>




3


b 1


2a


2b 1





 


 <sub> (vì b nguyên nên 2b - 1 0)</sub>




2 7


16a 4b 2b 1


2b 1


    




Vì a, b nguyên, nên 2b – 1 phải là ước của 7


b 1 a 0


2b 1 1 <sub>1</sub>


b 0 a (L)


2b 1 1 <sub>2</sub>


2b 1 7 9



b 4 a (L)


2


2b 1 7


b 3 a 2


  




 




   


 <sub></sub> <sub></sub>





  <sub></sub>


  


  






  




   




Với a = 0, b = 1 ta có hệ


x y 0


x y 1


xy 1


 


  





Với a = 2, b = -3 ta có hệ


2



y x 2
x y 2


(VN)


xy 3 x 2x 3 0


 
  




 


   


</div>
<span class='text_page_counter'>(45)</span><div class='page_container' data-page=45>

KL : Các số x, y nguyên thoả mãn điều kiện bài toán là : x = y = 1, x = y = -1


<b>Bài 84:</b><i>( HSG TỈNH THÁI NGUYÊN NĂM HỌC 2011– 2012).</i>


C1: Gọi 5 số nguyên liên tiếp là n-2, n-1, n, n+1, n+2 với n nguyờn, dễ thấy tổng các


bình phương của 5 số đó là 5(n

2

<sub> + 2) chia hết cho 5 nhưng không chia hết cho 25 nên </sub>


khơng thể là số chính phương.



C2: Xét tính chẵn lẻ của 5 số nguyên liên tiếp đó.



<b>Bài 85:</b><i>( HSG TỈNH THÁI NGUYÊN NĂM HỌC 2012– 2013).</i>



Điều kiện a, b, c

<sub>0. Từ </sub>



1

1

1

1



(bc ac ba)(a b c) abc 0



a

b

c

a b c

 



<sub>(a + b)(b + c)(c + a) = 0 </sub>



a b 0


b c 0


c a 0


 




<sub> </sub>





  



<sub> </sub>



Nếu a + b = 0 mà a + b + c = 2013 thì c = 2013


Nếu b + c = 0 mà a + b + c = 2013 thì a = 2013



Nếu c + a = 0 mà a + b + c = 2013 thì b = 2013. Vậy 1 trong 3 số a, b, c bằng 2013.



<b>Bài 86:</b><i>( HSG TỈNH THANH HÓA NĂM HỌC 2010– 2011).</i>



1 2 ... 44 ( 2 1) ( 3 2) ... ( 45 44) 45 1 130 1 129.


<i>d d</i>  <i>d</i> <i>a a</i>  <i>a a</i>   <i>a</i>  <i>a</i> <i>a</i>  <i>a</i>    <sub> (1)</sub>


Nếu mỗi hiệu <i>dj</i> (<i>j</i> 1, 2,...., 44)<sub> xuất hiện khơng q 10 lần thì </sub>


1 2 ... 44 9(1 2 3 4) 8.5 130


<i>d</i> <i>d</i>  <i>d</i>       <sub> mâu thuẫn với (1).</sub>


Vậy phải có ít nhất một hiêụ <i>dj</i> (<i>j</i>1,...,44)<sub> xuất hiện khơng ít hơn 10 lần</sub>


<b>Bài 87:</b><i>( HSG TỈNH THANH HĨA NĂM HỌC 2013– 2014).</i>


a) Giả sử (a + b2<sub>)  (a</sub>2<sub>b – 1), tức là: a + b</sub>2<sub> = k(a</sub>2<sub>b – 1), với k  </sub>*<sub> </sub>
 a + k = b(ka2<sub> – b)  a + k = mb</sub> <sub>(1)</sub>


Ở đó m   mà: m = ka2<sub> – b  m + b = ka</sub>2 <sub>(2)</sub>
Từ (1) và (2) suy ra: (m – 1)(b – 1) = mb – b – m + 1 


 (m – 1)(b – 1) = (a + 1)(k + 1 – ka) (3)
Do m > 0 (điều này suy ra từ (1) do a, k, b > 0) nên m  1 (vì m  ).
Do b > 0 nên b – 1  0 (do b  )  (m – 1)(b – 1)  0.


Vì thế từ (3) suy ra: (a + 1)(k + 1 – ka)  0.


Lại do a > 0 nên suy ra: k + 1 – ka  0  k + 1  ka  1  k(a – 1) (4)


Vì a – 1  0 (do a  , a > 0) và k  , k > 0 nên từ (4) có:



a 1
k(a 1) 0


a 2
k(a 1) 1


k 1



 


 <sub></sub>


  


 <sub></sub> <sub></sub> 




 <sub></sub>


</div>
<span class='text_page_counter'>(46)</span><div class='page_container' data-page=46>

- Với a = 1. Thay vào (3) ta được: (m – 1)(b – 1) = 2 


m 1 2


b 1 1 b 2


b 3


m 1 1


b 1 2
  





   





 


<sub></sub> <sub></sub> <sub></sub> <sub></sub> <sub></sub>


 
 
 

Vậy, trường hợp này ta có: a = 1, b = 2 hoặc a = 1, b = 3.


- Với a = 2 (vì k = 1). Thay vào (3) ta có: (m – 1)(b – 1) = 0 
b 1
m 1









 <sub>.</sub>


Khi b = 1, ta được: a = 2, b = 1.


Khi m = 1: Từ (1) suy ra a + k = b  b = 3. Lúc này được: a = 2, b = 3.
Tóm lại, có 4 cặp số (a; b) thỏa mãn bài toán là: (1; 2), (1; 3), (2; 3), (2; 1).


b) Ta có

<i>x</i>

+

2

3

=

<i>y</i>

+

<i>z</i>

<i>x</i>

+

2

3

=

<i>y</i>

+

<i>z</i>

+

2

<i>yz</i>



(

<i>x</i>

<i>y</i>

<i>z</i>

)+

2

3

=

2

<sub>√</sub>

<i>yz</i>

(

<i>x</i>

<i>y</i>

<i>z</i>

)

2

+

4

<sub>√</sub>

3

(

<i>x</i>

<i>y</i>

<i>z</i>

)+

12

=

4

<i>yz</i>

<sub> (1)</sub>


<b>TH1</b>. Nếu

<i>x</i>

<i>y</i>

<i>z</i>

0

Ta có

3=


4<i>yz</i>−

(

<i>x</i>−<i>y</i>−<i>z</i>

)

2−12


4

(

<i>x</i>−<i>y</i>−<i>z</i>

)

<sub> (2) vô lý </sub>


( do

<i>x, y ,z</i>

<i>N</i>

nên vế phải của (2) là số hữu tỷ ).


<b>TH2.</b>

<i>x</i>

<i>y</i>

<i>z</i>

=

0

khi đó


(1) ⇔


<i>x</i>−<i>y</i>−<i>z</i>=0


<i>yz</i>=3
¿



¿{¿ ¿ ¿ (3)


Giải (3) ra ta được


<i>x</i>=4


<i>y</i>=1


<i>z</i>=3


¿


{<sub>¿</sub>{<sub>¿ ¿¿</sub>


¿ hoặc


<i>x</i>=4


<i>y</i>=3


<i>z</i>=1


¿


{<sub>¿</sub>{<sub>¿ ¿¿</sub>


¿ thử lại thỏa mãn


<b>Bài 88:</b><i>( HSG TỈNH THANH HĨA NĂM HỌC 2014– 2015).</i>



Ta có:

5(

<i>x</i>

2

<i>xy y</i>

2

) 7(

<i>x</i>

2 )

<i>y</i>

(1)


7(

<i>x</i>

2 ) 5

<i>y</i>

(

<i>x</i>

2 ) 5

<i>y</i>

<sub>. Đặt </sub>

<i>x</i>

2

<i>y</i>

5

<i>t</i>

<sub> (2) (</sub><i>t Z</i> )<sub> thì</sub>
(1) trở thành

<i>x</i>

2

<i>xy y</i>

2

7

<i>t</i>

(3).


Từ (2)

<i>x</i>

5

<i>t</i>

2

<i>y</i>

<sub> thay vào (3) ta được </sub>3<i>y</i>215<i>ty</i>25<i>t</i>2 7<i>t</i> 0<sub> (*), coi đây là PT bậc </sub>
hai đối với y có:

 

84

<i>t</i>

75

<i>t</i>

2


Để (*) có nghiệm

   

0

84

<i>t</i>

75

<i>t</i>

2

0



28
0


25


<i>t</i>
  

<i>t Z</i>

<i>t</i>

0

hoặc

<i>t</i>

1

. Thay vào (*) :


</div>
<span class='text_page_counter'>(47)</span><div class='page_container' data-page=47>

+ Với

<i>t</i>

1



2 2


3 3


3 1


2 1



<i>y</i> <i>x</i>


<i>y</i> <i>x</i>


  


 


  


Vậy phương trình có 3 nghiệm ngun (x, y) là (0; 0), (-1; 3) và ( 1; 2)


Nếu <i>p q</i> thì


2


2(

1)

4



2

2



1

1



<i>m</i>



<i>p</i>

<i>m</i>



<i>m</i>

<i>m</i>








<sub>. </sub>


Do <i>m</i> <sub> và </sub><i><sub>p</sub></i><sub> là số nguyên tố nên </sub>4 (<i>m</i>1) <i>m</i>0;<i>m</i>1;<i>m</i>3

<i>p</i>

2;

<i>p</i>

5.



Nếu <i>p q</i> thì <i>pq</i> và <i>p</i> + <i>q</i> là nguyên tố cùng nhau vì <i>pq</i> chỉ chia hết cho các ước nguyên tố là <i>p</i> và
<i>q</i> còn <i>p</i> + <i>q</i> thì khơng chia hết cho <i>p</i> và không chia hết cho <i>q</i>.


Gọi <i>r</i> là một ước chung của

<i>m</i>

2

1

<i>m</i>

 

1



2

(

<i>m</i>

1)(

<i>m</i>

1)

<i>r</i>

(

<i>m</i>

1)

<i>r</i>



2 2


(

<i>m</i>

1) (

<i>m</i>

1)

<i>r</i>

2

<i>r</i>





<sub></sub>

<sub> </sub>

<i><sub>r</sub></i>

<sub>1</sub>



hoặc

<i>r</i>

2

.

)

<i>r</i>

1



<sub> suy ra </sub>

<i>p q m</i>

 

1,

<i>pq m</i>

2

 

1

<i>p q</i>

,

<sub> là hai nghiệm của phương trình</sub>


2

<sub>(</sub>

<sub>1)</sub>

2

<sub>1 0</sub>




<i>x</i>

<i>m</i>

<i>x m</i>

 

<sub> vô nghiệm do</sub>


2 2 2


3

<i>m</i>

2

<i>m</i>

3

(

<i>m</i>

1)

(2

<i>m</i>

2) 0



 





)

<i>r</i>

2



<sub> suy ra </sub>

2

<i>pq m</i>

2

1 và 2(

<i>p q</i>

)

  

<i>m</i>

1

<i>p q</i>

,

<sub> là hai nghiệm của phương trình</sub>


2 2


2

<i>x</i>

(

<i>m</i>

1)

<i>x m</i>

 

1 0

<sub> vơ nghiệm do</sub>


2 2 2


7

<i>m</i>

2

<i>m</i>

7

(

<i>m</i>

1)

(6

<i>m</i>

6) 0



 



<sub>.</sub>


Vậy bộ các số nguyên tố (<i>p</i>; <i>q</i>) cần tìm là ( ; ) (2;2); ( ; ) (5;5).<i>p q</i>  <i>p q</i> 


<b>Bài 89:</b><i>( HSG TỈNH THANH HÓA NĂM HỌC 2016– 2017).</i>


a)Ta có :


2016

<sub>1 (</sub>

4 504

<sub>)</sub>

<sub>1</sub>

504

<sub>(</sub>

4

<sub>1).</sub>

<sub>(</sub>

<sub>1)(</sub>

<sub>1)(</sub>

2

<sub>1).</sub>

<sub>(1) (</sub>

<sub>)</sub>




<i>p</i>

<i>p</i>

<i>p</i>

<i>A</i>

<i>p</i>

<i>p</i>

<i>p</i>

<i>A</i>

<i>A N</i>



Vì P là số nguyên tố lớn hơn 5 nên p là số lẻ, suy ra p – 1, p +1 là hai số chẵn liên tiếp


(

<i>p</i>

1)(

<i>p</i>

1) 4

(2)



 



Vì p – 1, p, p+1 là ba số tự nhiên liên tiếp nên

(

<i>p</i>

1) (

<i>p p</i>

 

1) 3

.

Nhưng p không chia hết
cho 3 nên

(

<i>p</i>

1)(

<i>p</i>

 

1) 3

(3)



Vì p khơng chia hết cho 5 nên p có một trong các dạng

5

<i>k</i>

1; 5

<i>k</i>

2


- Nếu

<i>p</i>

5

<i>k</i>

1

thì

<i>p</i>

2

25

<i>k</i>

2

10

<i>k</i>

 

1 5

<i>n</i>

1



- Nếu

<i>p</i>

5

<i>k</i>

2

thì

<i>p</i>

2

25

<i>k</i>

2

20

<i>k</i>

 

4 5

<i>l</i>

1



Cả hai trường hợp trên đều cho ta

<i>p</i>

4

1 5 5

<i>q</i>

(4)

(

( , ,

<i>n l q N</i>

)



Vì 3, 4, 5 là các số nguyên tố cùng nhau từng đôi một nên từ (1), (2), (3), (4) suy ra

<i>p</i>

2016

1


chia hết cho 4.3.5 tức là chia hết cho 60


</div>
<span class='text_page_counter'>(48)</span><div class='page_container' data-page=48>

Khi đó , gọi t là thương của phép chia

<i>x</i>

3

<i>y</i>

3

<i>z x y z</i>

3

:

2 2 2. Suy ra :


3 3 3 3


3 3 3 2 2 2 2 2 2 2 2 2


2 2 2

(1)




<i>x</i>

<i>y</i>

<i>x</i>

<i>y</i>



<i>x</i>

<i>y</i>

<i>z</i>

<i>tx y z</i>

<i>z tx y</i>

<i>tx y</i>

<i>tx y</i>

<i>x y</i>



<i>z</i>

<i>x y</i>







- Nếu <i>tx y</i>2 2 <i>x y</i> 0 (*) thì 2 2


1 1


2 1


<i>t</i> <i>t</i>


<i>xy</i> <i>x y</i>


    


Thay t = 1 vào (*), ta được <i>x y</i>2 2  <i>x y</i>  0 <i>xy x y</i>   0 (<i>x</i> 1)(<i>y</i> 1) 1
1


<i>x</i>
 


2



0 ( 1) 0


<i>y</i> <i>y</i> <i>y y</i>


      <sub> ( vô lý)</sub>


Vậy <i>tx y</i>2 2 <i>x y</i> 0 (2)


- Từ (1), (2) suy ra : <i>z</i>2(<i>tx y</i>2 2 <i>x y</i> )2 (3)


- Mặt khác vì <i>x</i>3<i>y</i>3<i>z</i>3 <i>tx y z</i>2 2 2 nên <i>x</i>3<i>y z</i>3 2 <i>x</i>3<i>y</i>3<i>z</i>2 (4)
- Từ (3) và (4) suy ra :


3 3 2 2 2


3 3 2 4 4 2 2 2 2


3 3 2 2 2 4 4


3 3 2 2


3 3


3 3


( )


2 ( ) 2


2 ( )



2 ( )


1 1 1 1


2 (5)


<i>x</i> <i>y</i> <i>tx y</i> <i>x y</i>


<i>x</i> <i>y</i> <i>t x y</i> <i>tx y x y</i> <i>x</i> <i>xy y</i>


<i>x</i> <i>y</i> <i>tx y x y</i> <i>t x y</i>
<i>x</i> <i>y</i> <i>tx y x y</i>
<i>txy</i>


<i>tx y</i>
<i>txy</i>


<i>x</i> <i>y</i> <i>tx</i> <i>ty</i>


   


       


    


  


 



 


  <sub></sub>  <sub></sub> 


 


- Nếu <i>x</i>2<sub> thì </sub> 3 3 3 3


1 1 1 1 1 1 1 1


3 6 2 2


2 2 .2 .2 . .


<i>y</i> <i>txy</i>


<i>t</i> <i>t</i> <i>x</i> <i>y</i> <i>t x</i> <i>t y</i>


 


 


    <sub></sub>  <sub></sub>   <sub></sub>  <sub></sub> 


   


Điều này mâu thuẫn với (5).
Vậy x = 1. Khi đó (5) trở thành :


3



2 1 1


2 (6)


<i>ty</i>


<i>y t ty</i>
   


- Nếu <i>y</i>4 thì 3 3


2 1 1 2 1 1


4 2 2


4 .4


<i>ty</i>


<i>t t</i> <i>y t ty</i>


        


. Điều này mâu thuẫn với (6).
Vậy <i>y</i>

2;3

(Vì y > x = 1)


+ Nếu y = 2 thì


3 3 2



9


1; 2; 3


1; 2


<i>x</i> <i>y</i> <i>z</i>


<i>x</i> <i>y</i> <i>z</i>


<i>x</i> <i>y z</i>


<i>x</i> <i>y</i>


  


   


 


 <sub></sub> <sub></sub>






.



+ Nếu y = 3 thì


3 3 <sub>28</sub> 2


1; 3


<i>x</i> <i>y</i> <i>z</i>


<i>x y z</i>


<i>x</i> <i>y</i>


  


 


 <sub></sub> <sub></sub>






.( Loại)


</div>
<span class='text_page_counter'>(49)</span><div class='page_container' data-page=49>

Vậy thương của phép chia

<i>x</i>

3

<i>y</i>

3

<i>z x y z</i>

3

:

2 2 2là t = 1.


<b>Bài 90:</b><i>( HSG TỈNH THANH HĨA NĂM HỌC 2017– 2018).</i>



<b>1. Tìm nghiệm ngun của phương trình </b>



2 <sub>5</sub> <sub>62 (</sub> <sub>2)</sub> 2 2 <sub>6</sub> <sub>8</sub> <sub>(1).</sub>
<i>y</i>  <i>y</i>  <i>y</i> <i>x</i>  <i>y</i>  <i>y</i> <i>x</i>
Ta có (1)

<i>y</i> 2

 

<i>y</i> 3

56 ( <i>y</i> 2)<i>x</i>2

<i>y</i> 2

 

<i>y</i> 4

<i>x</i>


<i><sub>y</sub></i> <sub>2</sub>

<i><sub>x</sub></i>2

<i><sub>y</sub></i> <sub>4</sub>

<i><sub>x</sub></i>

<i><sub>y</sub></i> <sub>3</sub>

<sub>56</sub>


 


  <sub></sub>     <sub></sub> 


<i>x</i> 1

 

<i>y</i> 2

 

<i>x y</i> 3

56.


     


Nhận thấy

<i>y</i> 2

 

 <i>x</i>1

  <i>x y</i> 3, nên ta phải phân tích số 56 thành tích của ba số nguyên mà tổng
hai số đầu bằng số cịn lại.


Như vậy ta có


 


 



) 56 1.7.8 ; 2;9 .


) 56 7.1.8 ; 8;3 .


<i>x y</i>


<i>x y</i>


   


   


 


 

 



) 56 8 .1. 7 ; 7;3 .


) 56 1. 8 . 7 ; 2; 6 .
<i>x y</i>


<i>x y</i>


      


      


 


 

 



) 56 8 .7. 1 ; 7;9 .


) 56 7. 8 . 1 ; 8; 6 .
<i>x y</i>


<i>x y</i>



      


      


Vậy phương trình có 6 nghiệm ngun như trên.


<b>Chú ý 3:Học sinh có thể biến đổi phương trình đến dạng</b>


<i><sub>y</sub></i> <sub>2</sub>

<i><sub>x</sub></i>2

<i><sub>y</sub></i> <sub>4</sub>

<i><sub>x</sub></i>

<i><sub>y</sub></i> <sub>3</sub>

<sub>56</sub>


 


 <sub></sub>     <sub></sub> 


(<i><b>được 0,5đ</b></i>), sau đó xét các trường hợp xảy ra.


<i><b>Khi đó với mỗi nghiệm đúng tìm được thì cho 0,25 đ </b>(tối đa 6 nghiệm = 1,5 đ)</i>


<b>2. Cho </b><i>a b</i>, <b>là các số nguyên dương thỏa mãn </b> <i>p a</i> 2 <i>b</i>2<b>là số nguyên tố và </b> <i>p</i> 5<b> chia hết cho 8. </b>
<b>Giả sử </b><i>x y</i>, <b> là các số nguyên thỏa mãn </b><i>ax</i>2 <i>by</i>2<b>chia hết cho </b><i>p<b>. Chứng minh rằng cả hai số </b>x y</i>,
<b>chia hết cho </b><i>p<b>.</b></i>


Do <i>p</i> 5 8 nên <i>p</i>8<i>k</i>5 (<i>k</i> )




4 2 4 2


2 <i>k</i> 2 <i>k</i> 2 2



<i>ax</i>   <i>by</i>  <i>ax</i>  <i>by</i> <i>p</i>


nên <i>a</i>4<i>k</i>2<i>x</i>8<i>k</i>4 <i>b</i>4<i>k</i>2<i>y</i>8<i>k</i>4<i>p</i>


Nhận thấy



4<i>k</i> 2 8<i>k</i> 4 4<i>k</i> 2 8<i>k</i> 4 4<i>k</i> 2 4<i>k</i> 2 8<i>k</i> 4 4<i>k</i> 2 8<i>k</i> 4 8<i>k</i> 4
<i>a</i>  <i>x</i>  <i>b</i>  <i>y</i>  <i>a</i>  <i>b</i>  <i>x</i>  <i>b</i>  <i>x</i>  <i>y</i> 


      


Do

 

 



2 1 2 1


4<i>k</i> 2 4<i>k</i> 2 2 <i>k</i> 2 <i>k</i> 2 2
<i>a</i>  <i>b</i>  <i>a</i>  <i>b</i>  <i>a</i> <i>b</i> <i>p</i>


     


và <i>b</i> <i>p</i> nên <i>x</i>8<i>k</i>4<i>y</i>8<i>k</i>4<i>p</i> (*)


Nếu trong hai số<i>x y</i>, có một số chia hết cho <i>p</i> thì từ (*) suy ra số thứ hai cũng chia hết cho <i>p</i>.
Nếu cả hai số<i>x y</i>, đều không chia hết cho <i>p</i> thì theo định lí Fecma ta có :


8<i>k</i> 4 <i>p</i> 1 <sub>1(mod ),</sub> 8<i>k</i> 4 <i>p</i> 1 <sub>1(mod )</sub>


<i>x</i>  <i>x</i>  <i>p</i> <i>y</i>  <i>y</i>  <i>p</i>


   



8<i>k</i> 4 8<i>k</i> 4 <sub>2(mod )</sub>


<i>x</i>  <i>y</i>  <i>p</i>


   <sub>. Mâu thuẫn với (*).Vậy cả hai số</sub><i><sub>x</sub></i><sub> và </sub><i>y</i><sub> chia hết cho </sub><i>p</i><sub>.</sub>


<b>Bài 91:</b><i>( HSG TỈNH THANH HĨA NĂM HỌC 2018– 2019).</i>


<b>1</b><i><b>. </b></i><b>Tìm nghiệm nguyên của phương trình </b>

<i>x y x y</i>

2 2

  

<i>x</i>

2

<i>y x</i>

1

<b>.</b>


</div>
<span class='text_page_counter'>(50)</span><div class='page_container' data-page=50>

2
2
1
<i>a</i>
<i>b</i>
<i>a</i>

 



2 2 2 2 2 2


2 1 4 1 1 5 1 5 1


<i>a</i> <i>a</i> <i>a</i> <i>a</i> <i>a</i> <i>a</i> <i>a</i>


               





2 <sub>1</sub> <sub>1;5</sub> 2 <sub>0;4</sub> <sub>0; 2; 2</sub>


<i>a</i> <i>a</i> <i>a</i>


       


Nếu

 



0


0 2 , 0;2 , 2;0


2
<i>xy</i>


<i>a</i> <i>b</i> <i>x y</i>


<i>x y</i>


    <sub></sub>  
 

Nếu
2
2
2
2 0


0 <sub>2</sub>
2
<i>x</i>
<i>y</i>
<i>xy</i>
<i>a</i> <i>b</i>


<i>x y</i> <i><sub>x</sub></i>


<i>y</i>
  
 
 
 
 
      
  <sub></sub>
  




 


 <sub> (loại vì khơng thỏa mãn</sub><i>x y Z</i>,  <sub>)</sub>
Nếu


4


2 ,



5
<i>a</i>  <i>b</i>


loại vì khơng thỏa mãn <i>b Z</i> .


Vậy nghiệm nguyên

<i>x y</i>,

của phương trình đã cho là:

0; 2 , 2;0 .

 



<b>Cách 2 :</b>Đưa phương trình về dạng:

  



2


( 2) 0


<i>x y xy</i>  <i>xy</i> <i>x y</i>  


Đặt <i>t</i><i>xy</i>, <i>t Z</i> <sub>ta được phương trình ẩn</sub><i>t</i><sub>: </sub>

<i>x y t</i>

2 <i>t</i> (<i>x y</i>  2) 0 (1)


Nếu


2 2


0 2


0 <sub>2</sub>


<i>xy</i> <i>x</i>


<i>x y</i> <i>xy</i>



<i>x y</i> <i><sub>y</sub></i>




 


 


     <sub></sub>  <sub></sub>


  <sub></sub>


 <sub></sub> <sub> Hoặc </sub>


2
2
<i>x</i>
<i>y</i>
 





 <sub> (loại)</sub>


*) Nếu <i>x y</i> 0, ta có phương trình bậc 2 ẩn<i>t</i>:


<i><sub>x y t</sub></i>

2 <i><sub>t</sub></i>

<i><sub>x y</sub></i> <sub>2</sub>

<sub>0</sub> <sub>(2)</sub>



     


 

2 5


1 4 2 0 1


4


<i>x y x y</i> <i>x y</i>


           


<i>x y</i> 1

2

0;1

<i>x y</i> 1

 

1;0;1

<i>x y</i>

1; 2



           
*) Nếu
1 5
2
1
1 5
2
<i>xy</i>
<i>x y</i>
<i>xy</i>
 <sub></sub>



 <sub>  </sub>


 <sub></sub>


 <sub> (loại) </sub>


*) Nếu


 



0


2 <sub>1</sub> , 0;2 , 2;0


2
<i>xy</i>


<i>x y</i> <i>x y</i>


<i>xy</i>



   <sub></sub>  



 <sub> (thỏa mãn) </sub>


Vậy nghiệm nguyên

<i>x y</i>,

của phương trình đã cho là:

0; 2 , 2;0 .

 




<b>2.</b> Cho <i>n</i> *<sub>.Chứng minh rằng nếu </sub>2<i>n</i>1<sub>và </sub>3<i>n</i>1<sub>là các số chính phương thì </sub><i>n</i><sub>chia hết cho</sub>40<sub>.</sub>


Giả sử 2<i>n</i> 1 <i>m</i>2<sub>, </sub>3<i>n</i> 1 <i>k</i>2


*
,


<i>m k N</i> <i><sub>m</sub></i>2


 <sub>là số lẻ </sub> <i>m</i><sub>là số lẻ.</sub>


 



2


2<i>n m</i> 1 <i>m</i> 1 <i>m</i> 1 4


      


, Suy ra: <i>n</i>chẵn, <i>k</i>lẻ


Vì <i>k</i>là số lẻ nên <i>k</i>1,<i>k</i>1 là hai số chẵn liên tiếp và

3, 8

1nên
Từ 3<i>n</i> 1 <i>k</i>2 3<i>n k</i> 21

<i>k</i>1

 

<i>k</i>1 8

  <i>n</i>8 (1)


</div>
<span class='text_page_counter'>(51)</span><div class='page_container' data-page=51>

Nếu <i>n</i> chia cho 5 dư 2<sub> thì 3</sub><i>n</i>1<sub>chia cho 5 dư </sub>2<sub>. (vơ lí)</sub>
Nếu <i>n</i> chia cho 5 dư 3 thì 2<i>n</i>1<sub>chia cho 5 dư </sub>2<sub>. (vơ lí)</sub>
Nếu <i>n</i> chia cho 5 dư 4 thì 3<i>n</i>1<sub>chia cho 5 dư 3. (vơ lí)</sub>
Vậy


5 (2)
<i>n</i>



5, 8

1nên từ (1) và (2) suy ra <i>n</i>chia hết cho 40 .


<b>Bài 92:</b><i>( HSG CẤP THÀNH PHỐ HỒ CHÍ MINH NĂM HỌC 2016– 2017).</i>


a)Từ a − b = 7, b − c = 3  a − c = 10, ta có:


2 2 2 2 2


2 2 2 2


2(a b c ab bc ca) (a b) (b c) (c a) 79


P ... <sub>40</sub>


2(a c 2ab 2bc) 2[(a b) (b c) ]


         


   


     


b) Ta có: (x + y)(x + 2y) = x + 5 x 3xy 2y x 5 02  2   4x 12xy 8y 4x 20 02  2  


2 2 2 2 2


(2x) 2.2x(3y 1) (3y 1) y 6y 21 0 [2x (3y 1)] (y 3) 12


              



(2x 2y 2)(2x 4y 4) 12 (x y 1)(x 2y 2) 3


           


Với x, y nguyên ta có bảng sau:


x + 2y – 2 1 3 –1 –3


x + y + 1 3 1 –3 –1


y – 3 –2 2 2 –2


y 1 5 5 1


x 1 –5 –9 –3


c) Vào thứ Hai, anh ta chơi bóng bàn và hai ngày sau đó anh ta chơi bóng đá.
• Nếu 0 ngày sau đó anh ta chơi bóng đá thì ngày chơi bóng đá là thứ Hai.


• Nếu 1 ngày sau đó anh ta chơi bóng đá thì ngày chơi bóng đá là thứ Ba.


• Đề bài cho hai ngày sau đó anh ta chơi bóng đá nên ngày chơi bóng đá là thứ Tư.


Suy ra 3 ngày chạy chỉ là: 1) thứ Ba, thứ Năm, thứ Bảy. Khi đó ngày chơi cầu lơng là thứ Sáu hoặc
Chủ Nhật (khơng xảy ra vì đều sau ngày chạy)


2) thứ Ba, thứ Năm, Chủ Nhật. Khi đó ngày chơi cầu lơng là thứ Sáu (khơng xảy ra vì sau
ngày chạy thứ Năm) hoặc thứ Bảy (khơng xảy ra vì khi đó sau ngày bơi thứ Sáu)



Hoặc 3) thứ Ba, thứ Sáu, Chủ Nhật. Khi đó ngày chơi cầu lơng là thứ Năm và ngày bơi là thứ Bảy
(thỏa điều kiện của đề bài). Vậy thứ Bảy trong tuần Nam đi bơi.


<b>Bài 93:</b><i>( HSG TỈNH TRÀ VINH NĂM HỌC 2016– 2017).</i>


Chứng minh rằng không tồn tại các số nguyên

<i> x, y, z </i>

thỏa mãn



3 3 3 <sub>2017</sub>


<i>x</i> <i>y</i> <i>z</i>    <i>x y z</i>

<sub> </sub>


Nếu x,y,z chẵn thì x

3

<sub>,y</sub>

3

<sub>,z</sub>

3

<sub> chẵn</sub>


Nếu x,y,z lẻ thì x

3

<sub>,y</sub>

3

<sub>,z</sub>

3

<sub> lẻ</sub>



Suy ra x+y+z và x

3

<sub>+y</sub>

3

<sub>+z</sub>

3

<sub> cùng tính chẵn, lẻ nên (x</sub>

3

<sub>+y</sub>

3

<sub>+z</sub>

3

<sub>)-( x+y+z) ln chẵn</sub>


Do đó (x

3

<sub>+y</sub>

3

<sub>+z</sub>

3

<sub>)-( x+y+z)=2017 là vơ lí</sub>



Vậy khơng tồn tại các số ngun

<i> x, y, z </i>

thỏa mãn

<i>x</i>3<i>y</i>3<i>z</i>3    <i>x y z</i> 2017

<sub> </sub>



<b>Bài 94:</b><i>( HSG TỈNH VĨNH PHÚC NĂM HỌC 2007– 2008).</i>


</div>
<span class='text_page_counter'>(52)</span><div class='page_container' data-page=52>

Trong 8 số ngun liên tiếp ln có 4 số chẵn liên tiếp, giả sử đó là các số



2 ,2<i>k k</i>2,2<i>k</i>4,2<i>k</i>6(<i>k</i> )

<sub>. Ta có </sub>

2 (2<i>k k</i>2)(2<i>k</i>4)(2<i>k</i>6) 16 ( <i>k k</i>1)(<i>k</i>2)(<i>k</i>3)

<i>k k</i>( 1)(<i>k</i>2)(<i>k</i>3) 8

<sub> nên </sub>

2 (2<i>k k</i>2)(2<i>k</i>4)(2<i>k</i>6) 128

<sub>. Từ đó suy ra điều cần chứng</sub>


minh.



b)Ta chứng minh bằng phản chứng. Từ phần a) ta suy ra

<i>m</i>128<i>c</i>5040

<sub>.</sub>



Giả sử

<i>m</i>

có thể phân tích thành tổng của hai số chính phương, tức là tồn tại các số tự




nhiên

<i>a b</i>,

sao cho

2 2


128<i>c</i>5040<i>a</i> <i>b</i>

<sub>(1)</sub>



Vế trái của (1) chia hết cho 4 nên

<i>a b</i>,

cùng là các số chẵn (vì ngược lại, nếu một số chẵn


và một số lẻ thì vế phải (1) là số lẻ, còn nếu hai số đều lẻ thì



2 2 <sub>(2</sub> <sub>1)</sub>2 <sub>(2</sub> <sub>1)</sub>2 <sub>4</sub> <sub>2</sub>


<i>a</i> <i>b</i>  <i>x</i>  <i>y</i>  <i>z</i>

<sub> chia 4 dư 2, vơ lí!)</sub>


Do đó

<i>a</i>2 ,<i>a b</i>1 2 ( ,<i>b a b</i>1 1 1 )



2 2


1 1


(1) 32<i>c</i>1260<i>a</i> <i>b</i> (2)


Lập luận tương tự cho (2), ta có

(2) 8<i>c</i>315<i>a</i>22<i>b a b</i>22,( ,2 2 ) (3)


Lúc này,

8<i>c</i>315 3(mod 4)

còn

<i>a</i>22<i>b</i>22 3(mod 4)

(thật vậy, xét tất cả các khả năng chẵn,



lẻ của

<i>a b</i>2, 2

ta thấy chỉ có ba khả năng xảy ra là



2 2


2 2


2 2



2 2


2 2


2 2


0(mod 4)
1(mod 4)


2(mod 4)


<i>a</i> <i>b</i>
<i>a</i> <i>b</i>
<i>a</i> <i>b</i>


 


 <sub></sub> <sub></sub>


  


<sub> do đó (3) mâu thuẫn, </sub>



suy ra điều phải chứng minh.



<b>Bài 95:</b><i>( HSG TỈNH VĨNH PHÚC NĂM HỌC 2008– 2009).</i>


a)Phương trình đã cho tương đương với


 



2

4 3 2

2 4 3 2


4 3<i>m</i> 3<i>m</i> 1 4<i><sub>n</sub></i> 8<i><sub>n</sub></i> 4<i><sub>n</sub></i> 8<i><sub>n</sub></i> 1 2 3<i>m</i> 1 4<i><sub>n</sub></i> 8<i><sub>n</sub></i> 4<i><sub>n</sub></i> 8<i><sub>n</sub></i> 1


              


Do



2
2 3<i>m</i> 1


 


là số chình phương và


<sub>2</sub><i><sub>n</sub></i>2 <sub>2</sub><i><sub>n</sub></i>

2 <sub>4</sub><i><sub>n</sub></i>4 <sub>8</sub><i><sub>n</sub></i>3 <sub>4</sub><i><sub>n</sub></i>2 <sub>8</sub><i><sub>n</sub></i> <sub>1</sub>

<sub>2</sub><i><sub>n</sub></i>2 <sub>2</sub><i><sub>n</sub></i> <sub>1</sub>

2


        


nên



2 <sub>2</sub> 2


2 3<i>m</i> 1 2<i><sub>n</sub></i> 2<i><sub>n</sub></i> 1 4 (<i><sub>n n</sub></i> 1) 0


       


Suy ra <i>n</i>1 và do đó <i>m</i>1


Thử lại và kết luận


b)Ký hiệu số ghi ở ô (<i>i</i>;<i>j</i>) là <i>ai j</i><sub> và gọi </sub><i>S</i> {( ; ) :<i>i j aij</i>>0}<sub>. Gọi </sub><i>r ci</i>, <i>j</i><sub> là tổng các số ghi trên hàng </sub><i><sub>i</sub></i><sub> và</sub>


cột <i>j</i>. Vậy <i>ri</i> <i>cj</i>  ( ; )<i>i j</i> <i>S</i>


Khi đó ta có ( ; ) ( ; )


<i>ij</i> <i>ij</i>


<i>i j S</i> <i>i</i> <i>i j S</i> <i>j</i>


<i>a</i> <i>a</i>


<i>r</i> <i>c</i>


 






Tính tổng từng vế: ( ; ) 1 1


1


<i>m</i> <i>m</i>
<i>ij</i>


<i>ij</i>


<i>i j S</i> <i>i</i> <i>j</i> <i>j</i> <i>i</i>


<i>a</i>


<i>VT</i> <i>a</i> <i>m</i>


<i>r</i> <i>r</i>


  


<sub></sub>

<sub> </sub>



( ; ) 1 1


1


<i>n</i> <i>n</i>
<i>ij</i>


<i>ij</i>
<i>i j S</i> <i>i</i> <i>i</i> <i>i</i> <i>j</i>


<i>a</i>


<i>VP</i> <i>a</i> <i>n</i>


<i>c</i> <i>c</i>


  



<sub></sub>

<sub> </sub>


Suy ra <i>m</i> = <i>n</i>


</div>
<span class='text_page_counter'>(53)</span><div class='page_container' data-page=53>

+ Nếu <i>m</i> = 1 thì trên mỗi cột có đúng một số dương và tổng các số dương trên mỗi cột này bằng <i>s</i> =
<i>ns</i> (bằng tổng các số dương trên hàng). Do đó <i>n</i> = 1


+ Trong trường hợp tổng quát, gọi <i>r </i> < <i>m</i> là số hàng có tổng bằng <i>s</i>, cịn trên các hàng khác có tổng
khác. Do mỗi cột, mà có giao với <i>r</i> hàng đó tại ơ dương có tổng bằng <i>s</i>, nên giả sử có <i>c</i> cột có tổng
bằng <i>s</i>. Thực hiện việc đánh số lại các hàng, cột sao cho <i>r </i>hàng đầu và <i>c</i> cột đầu có tổng bằng <i>s</i>
(không làm thay đổi bản chất của bảng). Khi đó, những ơ của <i>r </i>hàng đầu, khơng nằm trong <i>c</i> cột đầu
và những ô của <i>c</i> cột đầu không nằm trong <i>r</i> hàng đầu phải chứa số 0. Vậy bảng con <i>r c</i> <sub>(gồm giao </sub>
của <i>r </i>hàng đầu và <i>c </i>cột đầu thỏa mãn). Suy ra <i>r = c</i>. Nhưng, phần còn lại của bảng, sau khi bỏ đi <i>r </i>
hàng đầu và <i>c</i> cột đầu (kích thước (<i>m r</i> ) ( <i>n c</i>)cũng thỏa mãn.


Do đó, bằng quy nạp, được <i>m = n</i>.


<b>Bài 96:</b><i>( HSG TỈNH VĨNH PHÚC NĂM HỌC 2009– 2010).</i>


a)Gọi <i>a</i> là số lẻ lớn nhất mà <i>a</i>2 <i>n</i>.<sub> Khi ấy </sub>


2
2
<i>n</i> <i>a</i>


Nếu <i>a</i>7<sub> thì </sub><i>a</i> 4,<i>a</i> 2,<i>a</i><sub> là các ước lẻ của </sub><i>n</i>.<sub> Để ý rằng, các số này nguyên tố cùng nhau đôi một, </sub>
nên <i>a a</i>

 2

 

<i>a</i> 4 |

<i>n</i>. Suy ra


<sub>2</sub>

 

<sub>4</sub>

<sub>2</sub>

2 3 <sub>7</sub> 2 <sub>4</sub> <sub>4 0</sub> 2

<sub>7</sub>

<sub>4</sub>

<sub>1</sub>

<sub>0</sub>
<i>a a</i> <i>a</i>  <i>n</i> <i>a</i>  <i>a</i>  <i>a</i>  <i>a</i>   <i>a a</i>  <i>a</i> 


. Vơ lý (do <i>a</i>7<sub>). </sub>


Do đó <i>a</i>1<sub> hoặc </sub><i>a</i>3<sub> hoặc </sub><i>a</i>5


- Nếu <i>a</i>1<sub> thì </sub>12  <i>n</i> 32  <i>n</i>

1, 2,3, 4,5,6,7,8



- Nếu <i>a</i>3<sub> thì </sub>32  <i>n</i> 52  <i>n</i>

9,12,15,18, 21, 24

<sub> (do 1,3|</sub><i>n</i><sub>)</sub>
- Nếu <i>a</i>5<sub> thì </sub>52 <i>n</i> 72 <i>n</i>

30, 45

<sub> (do 1,3,5|</sub><i>n</i><sub>)</sub>


Vậy tất cả các số nguyên dương <i>n</i> cần tìm là 1,2,3,4,5,6,7,8,9,12,15,18,21,24,30,45


b)-Trên mỗi hình vng con, kích thước 2 2 <sub> chỉ có khơng q 1 số chia hết cho 2, cũng vậy, có</sub>
khơng q 1 số chia hết cho 3


- Lát kín bảng bởi 25 hình vng, kích thước 2 2 <sub>, có nhiều nhất 25 số chia hết cho 2, có nhiều nhất</sub>
25 số chia hết cho 3. Do đó, có ít nhất 50 số cịn lại không chia hết cho 2, cũng không chia hết cho 3.
Vì vậy, chúng phải là một trong các số 1,5,7.


- Từ đó, theo nguyên lý Dirichlet, có một số xuất hiện ít nhất 17 lần.


<b>Bài 97:</b><i>( HSG TỈNH VĨNH PHÚC NĂM HỌC 2010– 2011).</i>


a)Đặt <i>a x</i> 1; <i>b y</i>  1, phương trình đã cho trở thành: (<i>a</i>1)2<i>b</i>(<i>b</i>1)2<i>a</i>1 (1).
Ta có:


(1) <i>ab a b</i>(  ) 4 <i>ab</i>(<i>a b</i> ) 1  <i>ab a b</i>(  4) ( <i>a b</i> 4) 5  (<i>a b</i> 4)(<i>ab</i>1) 5


Khi đó chỉ xảy ra 4 trường hợp sau:


1 9 3 5


; ; ;



0 2 4 6


       


   


   


   


   


<i>a b</i> <i>a b</i> <i>a b</i> <i>a b</i>


<i>ab</i> <i>ab</i> <i>ab</i> <i>ab</i>


Từ đó tìm ra ( , ) (0,1);(1,0);( 6,1);(1, 6)<i>a b</i>    . <i>(Mỗi trường hợp 0,25 điểm)</i>


Vậy có 4 cặp số ( , )<i>x y</i> cần tìm là: ( , ) (1, 2);(2,1);( 5, 2);(2, 5)<i>x y</i>   


b)- Giả sử trái lại, với mọi cách tô, không tồn tại
hai điểm cùng màu mà có khoảng cách bằng 1.
Xét hai điểm <i>M N MN</i>, :  3 thì tồn tại các điểm


,


<i>P Q</i><sub> sao cho các tam giác </sub><i>MPQ NPQ</i>, <sub> là các tam giác</sub>
đều có độ dài cạnh bằng 1.



</div>
<span class='text_page_counter'>(54)</span><div class='page_container' data-page=54>

cùng một màu, chẳng hạn tô <i>P</i>: Đỏ, <i>Q</i>: Vàng thì <i>M</i>, <i>N</i>:
phải tơ cùng màu Xanh, (Hình vẽ).


- Từ đó, nếu điểm <i>M</i> được tơ màu Xanh, thì mọi điểm nằm trên đường trịn tâm <i>M,</i> bán kính 3 đều
được tơ màu Xanh. Nhưng trên đường trịn này ln có hai điểm mà khoảng cách giữa chúng bằng 1.
Mâu thuẫn với giả thiết phản chứng.


Từ đó suy ra điều phải chứng minh.


<b>Bài 98:</b><i>( HSG TỈNH VĨNH PHÚC NĂM HỌC 2011– 2012).</i>


a)Nhận xét. Nếu <i>x y</i> 1 thì <i>f x</i>

 

<i>f y</i>

 

1.


Thật vậy, ta có


 



 






3
3


3 3


3 3


1


1


1 1


<i>x</i>
<i>x</i>


<i>f x</i> <i>f y</i> <i>f</i> <i>x</i>


<i>x</i> <i>x</i> <i>x</i> <i>x</i>




    


   


suy ra


 

 

 








3
3


3 3



3 3


1


1 1


1 1


<i>x</i>
<i>x</i>


<i>f x</i> <i>f y</i> <i>f x</i> <i>f</i> <i>x</i>


<i>x</i> <i>x</i> <i>x</i> <i>x</i>




      


   


.
Vậy, nhận xét được chứng minh. Ta có


1 1


2 2


<i>f</i> <sub></sub> <sub></sub>



  <sub>.</sub>


Theo nhận xét trên ta có:


1 2011 2 2010


...


2012 2012 2012 2012


1005 1007 1006 1


1005 1005,5


2012 2012 2012 2


<i>A</i> <i>f</i> <i>f</i> <i>f</i> <i>f</i>


<i>f</i> <i>f</i> <i>f</i> <i>f</i>


         


<sub></sub> <sub></sub> <sub></sub> <sub></sub> <sub></sub><sub></sub><sub></sub> <sub></sub> <sub></sub> <sub></sub> <sub></sub><sub></sub> 


       


   


        



    


       


 


       


 


b)Nếu <i>x y</i>  6 <i>x y x</i>   (<i>y</i>6) 1  phương trình vô nghiệm. Do đó <i>x y</i> 6


2 <i>x y</i> <i>y</i> 6 <i>x</i> <i>x</i> 3


         <i>x</i>{1;2}


Với <i>x</i>1<sub> thay vào phương trình ban đầu ta được:</sub>


<i><sub>y</sub></i> <sub>1</sub>

3 <sub>(</sub><i><sub>y</sub></i> <sub>5)</sub>2

<i><sub>y</sub></i> <sub>3</sub>

<i><sub>y</sub></i>2 <sub>5</sub><i><sub>y</sub></i> <sub>8</sub>

<sub>0</sub> <i><sub>y</sub></i> <sub>3</sub>


         


suy ra phương trình có nghiệm

<i>x y</i>;

(1; 3).
Với <i>x</i>2<sub> thay vào phương trình ban đầu ta được:</sub>


<i><sub>y</sub></i> <sub>2</sub>

3 <sub>(</sub><i><sub>y</sub></i> <sub>4)</sub>2 <i><sub>y</sub></i>3 <sub>5</sub><i><sub>y</sub></i>2 <sub>4</sub><i><sub>y</sub></i> <sub>8 0</sub>


       



phương trình này vơ nghiệm do <i>y</i>1.
Vậy phương trình đã cho có nghiệm

<i>x y</i>;

(1; 3).


c)Xét ngũ giác đều <i>ABCDE</i>, ta nhận thấy ba đỉnh bất kì của ngũ giác ln tạo thành một tam giác cân.
Do đó khi tơ 5 đỉnh <i>A, B, C, D, E</i> bằng 3 màu xanh, đỏ và tím sẽ xảy ra hai khả năng sau:


</div>
<span class='text_page_counter'>(55)</span><div class='page_container' data-page=55>

thành một tam giác cân.


+) Nếu tô 5 đỉnh <i>A, B, C, D</i>,<i> E</i> bởi nhiều nhất 2 màu thì có ít nhất 3 đỉnh cùng màu và tạo thành một
tam giác cân.


Vậy, trong mọi trường hợp ln tồn tại ít nhất một tam giác cân, có 3 đỉnh được tơ bởi cùng một
màu hoặc đôi một khác màu.


<b>Bài 99:</b><i>( HSG CẤP THÀNH PHỐ VĨNH YÊN NĂM HỌC 2012– 2013).</i>


a)Ta có



2 2


2 2 2 2 <sub>2</sub> <sub>2</sub>


<i>a</i> <i>b</i> <i>c</i> <i>d</i>  <i>a b</i>  <i>ab</i> <i>c d</i>  <i>cd</i>  <i>ab cd</i>


Mặt khác

<i>a c a d</i>

 

<i>a</i>2

<i>c d a cd</i>

 <i>a</i>2 <i>a a b</i>

<i>ab</i>0 suy ra <i>a c</i> <sub> hoặc </sub><i>a d</i> <sub>. Ta</sub>
xét hai trường hợp sau:


TH1. Nếu <i>a c</i> <sub> thì </sub><i>b d</i> <sub>. Do đó </sub><i>a</i>2013<i>b</i>2013<i>c</i>2013<i>d</i>2013
TH2. Nếu <i>a d</i> <sub> thì </sub><i>b c</i> <sub>. Do đó </sub><i>a</i>2013<i>b</i>2013<i>c</i>2013<i>d</i>2013



b)Nhận xét. Nếu <i>a</i> là một số nguyên dương thì <i>a</i>2 khi chia cho 3 hoặc 4 được số dư là 0 hoặc 1.
+) Nếu trong hai số <i>x y</i>, khơng có số nào chia hết cho 3 thì <i>x</i>2<i>y</i>2 khi chia cho 3 dư là 2 hay <i>z</i>2
khi chia cho 3 dư là 2, mâu thuẫn với nhận xét trên. Vậy trong 2 số <i>x y</i>, phải có một số chia hết cho
xy3. Suy ra <i>xy</i>3


+) Nếu trong hai số <i>x y</i>, khơng có số nào chia hết cho 4 thì <i>x</i>2<i>y</i>2 khi chia cho 4 dư là 2 hay <i>z</i>2
khi chia cho 4 dư là 2, mâu thuẫn với nhận xét trên. Vậy trong 2 số <i>x y</i>, phải có một số chia hết cho
4. Suy ra <i>xy</i>4


Từ chứng minh trên ta được <i>xy</i> chia hết cho 3 và 4 mà (3,4) = 1 nên <i>xy</i>12.


c)+) Nếu <i>x</i>1<sub> thì 2 1 3</sub>  <i>y</i>  <i>y</i>1<sub> hay phương trình đã cho có nghiệm </sub>

<i>x y</i>;

 

1;1


+) Nếu <i>x</i>2<sub> thì 2</sub><i>x</i> 1


 <sub> chia 4 dư 1 suy ra 3</sub><i>y</i><sub> chia 4 dư 1</sub> <i>y</i><sub> là số chẵn, </sub><i>y</i>2<i>z</i>
Từ phương trình ta được :

 

 



2


3<i>z</i> 1 2<i>x</i> 3<i>z</i> 1 3<i>z</i> 1 2<i>x</i>


     


suy ra tồn tại 2 số tự nhiên ,<i>a b</i> sao
cho 3<i>z</i> 1 2 ;3<i>a</i> <i>z</i>1 2 <i>b</i>.


Trừ từng vế 2 đẳng thức trên ta được :





2<i>a</i> 2<i>b</i> 2 2 2<i>b</i> <i>a b</i> 1 2 <i><sub>b</sub></i> 1,<i><sub>a b</sub></i> 1 <i><sub>a</sub></i> 2,<i><sub>b</sub></i> 1


           


. Do đó

<i>x y</i>;

 

 3;2

.
Vậy

<i>x y</i>;

 

1;1 , 3; 2

 



<b>Bài 100:</b><i>( HSG TỈNH YÊN BÁI NĂM HỌC 2003– 2004).</i>


Gọi số nhỏ là ab (a, b  N*, 1 ≤ a, b ≤ 9). Theo giả thiết:


2 2 2 2


2 2


2 2


ab a ab b 10a b a ab b


25a 4b a ab b


(a 5).b (a 5) (a 5)b b


        


 




 



   


      


 




Suy ra: 15a = 5b  3a = b 
b
a


3


hay b <sub></sub> 3  b  {3; 6; 9}
– Với b = 3 thì a = 1 (thỏa mãn)


– Với b = 6 thì a = 2 (loại)
– Với b = 9 thì a = 3 (loại)
Vậy: Hai số phải tìm là 13 và 63


</div>
<span class='text_page_counter'>(56)</span><div class='page_container' data-page=56>

Giả sử ba cạnh của tam giác là n – 1, n, n + 1 (n  Z, n > 4)


Đường cao chia cạnh có độ dài n thành hai đoạn x, y (giả sử x > y). Ta có:
x2<sub> = (n + 1)</sub>2<sub> – h</sub>2<sub> (1)</sub>


y2<sub> = (n – 1)</sub>2<sub> – h</sub>2<sub> (2)</sub>



Lấy (1) trừ (2) ta được: x2 <sub>– y</sub>2<sub> = n</sub>2<sub> + 2n + 1 – n</sub>2<sub> + 2n – 1 = 4n</sub>
 (x + y)(x – y) = 4n, mà x + y = n  x – y = 4


<b>Bài 102:</b><i>( HSG TỈNH YÊN BÁI NĂM HỌC 2011– 2012).</i>


<i><b>* Cách 1: </b></i>


Ta có: x − xy = 7x − 2y − 15  xy − 2y = x − 7x + 15


 y(x − 2) = x − 7x + 15  y = \f(x−7x+15,x−2 = \f(+5,x−2 = + \f(5,x−2
Vì x, y  Z  \f(5,x−2  Z  x − 2  Ư(5)


- Nếu x − 2 = 1  x = 3  y = 3−5 + \f(5,3−2 = 3
- Nếu x − 2 = -1  x = 1  y = 1−5+ \f(5,1−2 = -9
- Nếu x − 2 = 5  x = 7  y = 7−5+ \f(5,7−2 = 3


- Nếu x − 2 = -5  x = -3  y = -3−5+ \f(5,-3−2 = -9
Vậy các cặp số nguyên x, y thỏa mãn phương trình là


x ; y

3 ; 3 ; 1 ; 9 ; 7 ; 3 ; 3 ; 9

 

 

 

 

 



<i><b>* Cách 2:</b></i>


Ta thấy phương trình đã cho tương đương:
x − xy − 7x + 2y + 15 = 0


 (2y − xy) − (2x − x) + (10 − 5x) = -5
 y(2 − x) − x(2 − x) + 5(2 − x) = -5
 (2 − x)(y − x + 5) = -5



 (x − 2)(y − x + 5) = 5


Vì x, y là các số nguyên nên x − 2 và y − x + 5 cũng là các số nguyên
 x − 2 và y − x + 5 là các ước của 5.


</div>

<!--links-->

×